download scanned papers here - Paper 1    Paper II


UPSC PRELIMS 2020 PAPER I -- ANSWER KEYS


BIRD'S EYEVIEW


  • पिछले सात वर्षों में सबसे कठिन पेपर।

  • 2020 की प्रारम्भिक परीक्षा किसी के भी लिए चाहे अवर हो या अनुभवी जिसने यह आशा की थी की परीक्षा का पैटर्न 2019 या उसके पहले की परीक्षणों की तरह होगा, बड़ी चुनौती साबित हो सकती थी. अंततः यह परीक्षा हमें यही सिख देती है की वास्तविक जीवन हो या परीक्षा अनिश्चितता ही निश्चित है.

  • 2017 के पेपर (I और II दोनों) न केवल कठिन थे, बल्कि घुमावदार थे, कुछ प्रश्न अस्पष्ट थे एवं उन्हे हल करने के लिए कई गूढ़ तथ्यों की आवश्यकता थी। इससे कटऑफ प्रभावित हुआ था। 2018 का पेपर तथ्यात्मक रूप से कठिन था, 2017 की तरह विश्लेषणात्मक रूप से नहीं । लेकिन 2020 इन दोनों से पूरी तरह से अलग था। बहुत कम तार्किक अनुप्रयोग, अत्यधिक तथ्यात्मक ।

  • सीख (एक बार फिर): दीर्घकालिक तपस्या, गहराई में जाना, अनुमान लगाने का खेल समाप्त, विवेक रखना, अभ्यास करना।

  • प्रमुख परिवर्तनों का सारांश:
    1. बहुत कठिन प्रश्न-पत्र
    2. अन्यथा आसान विषयों पर कई सूक्ष्म स्तर के तथ्यों पर आधारित प्रश्न
    3. बहुत कम तर्क-आधारित प्रश्न
    4. कुछ सवालों में बेहद कठिन विकल्प
    5. कुछ पुर्णतः गूढ़ प्रश्न, जिनमें से ज्यादातर के बारे में किसी ने कभी नहीं सुना
    6. कुछ प्रश्नो में विषयों के बीच पारस्परिक संबंध उच्च स्तर का था किसीप्रश्न.
    7. कुछ बहुत ही आसान प्रश्न, जिन्हे बिल्कुल भी छोड़ना नहीं चाहिए था
    8. कुल मिलाकर, पिछले सात वर्षों का सबसे कठिन पेपर
    9. चूंकि यह एक सापेक्ष परीक्षण है, अंतत: जिन्होंने इन परिश्थितियों में अपना मानसिक संतुलन बांये रखते हुए प्रदर्शन किया वे सफलता प्राप्त करेंगे
  • विषय-वार विश्लेषण एवं प्रकार:
    1. कृषि - कई सवाल पूछे गए, जिनमें ज्यादातर कठिन थे
    2. प्राचीन / मध्यकालीन इतिहास - कुछ तार्किक प्रश्नो को छोड़कर, बहुत कठिन
    3. समसामयिकी - कठिन और मध्यम प्रश्नो का मिश्रण, कुछ आसान
    4. पर्यावरण एवं पारिस्थितिकी - कुछ बहुत कठिन, कुछ मध्यम, कुछ आसान
    5. भारत की अर्थव्यवस्था - कुछ कठिन, कुछ आसान, लेकिन आम तौर पर अप्रत्याशित प्रवृत्ति
    6. सामान्य विज्ञान - सिर्फ 2 प्रश्न पूछे गए, मुश्किल
    7. भूगोल - केवल 3 प्रश्न, करने योग्य
    8. सरकारी योजनाएं - 7 प्रश्न, कठिन और मध्यम का मिश्रण
    9. आधुनिक इतिहास - बहुत कठिन, कुछ मुश्किल से करने योग्य
    10. राजनीति - कुल मिलाकर आसान, प्रबंधनीय
    11. विज्ञान और प्रौद्योगिकी - एक कठिन संग्रह, कुछ मध्यम
  • इस स्तर पर, हम छात्रों को प्रांरम्भिक परीक्षा के लिए "कट-ऑफ" के बारे में किन्ही भी अटकलों से बचने का सुझाव देना चाहेंगे। पिछले वर्ष/ वर्षों) की तुलना में उनके बहुत कम होने की संभावना है।


विषयवार वर्गीकरण



विस्तृत प्रश्नवार समाधान (Set D)

1. भारत के सांस्कृतिक इतिहास के संदर्भ में, निम्नलिखित युग्मों पर विचार कीजिए :

  1. परिव्राजक - परित्यागी व भ्रमणकारी
  2. श्रमण - उच्च पद प्राप्त पुजारी
  3. उपासक - बौद्ध धर्म का साधारण अनुगामी

उपर्युक्त युग्मों में से कौन-से सही सुमेलित हैं ?

  1. केवल 1 और 2
  2. केवल 1 और 3
  3. केवल 2 और 3
  4. 1, 2 और 3

उत्तर (b) परिव्राजक सत्य के खोजी थे जो किसी एक स्थान पर स्थायी रूप से नहीं रहते थे। वे अपने घरों को छोड़कर गांवों एवं जंगलों में भटकते रहते थे। इसलिए उन्हें परिव्राजक (पथिक) या भिक्षु (भीख मांगने वाले) कहा जाता था। इन परिव्राजकों में, वर्धमान महावीर, गौतम बुद्ध, माख्खाली गोशाला एवं अजीता केशकंबालिन बहुत प्रसिद्ध थे। अतः, कथन 1 सही है।

श्रमण का अर्थ है ‘साधक, जो तपस्या करता है’। जैन, बौद्ध एवं अज्विका धर्मों सहित प्राचीन भारत की कुछ प्राचीन परंपराओं में श्रमण भटकने वाला भिक्षुओं को कहते थे। श्रमण परंपरा ने जैन एवं बौद्ध धर्म को जन्म दिया। श्रमणों ने, मोक्ष प्राप्त करने के लिए, घरेलू जीवन का त्याग करते हुए, वैदिक अनुष्ठानों से अलग मार्ग अपनाया। श्रमण परंपरा संसार (जन्म एवं मृत्यु का चक्र) एवं मोक्ष (उस चक्र से मुक्ति) से संबंधित अवधारणाओं के लिए जिम्मेदार है। श्रमणों ने आमअतः र पर तपस्या, ध्यान एवं संबद्ध सिद्धांअतः ं जैसी तीन प्रकार की गतिविधियों को शामिल किया। इसलिए, कथन 2 सही नहीं है। अतः केवल (इ) सही हो सकता है!

उपासक, ‘परिचर’ के लिए संस्कृत एवं पालि शब्द हैं। यह बौद्ध धर्म के अनुयायियों का शीर्षक है, जो बौद्ध आदेश में भिक्षु या नन नहीं हैं लेकिन वे कुछ प्रण लेते हैं। अतः कथन 3 सही है।

(यह प्रश्न आसान था क्योंकि हम जानते हैं कि श्रमण सन्यासी होते थे अतः तीन विकल्पों को सीधे खारिज किया जा सकता है!)


2. भारतीय हाथियों के संदर्भ में, निम्नलिखित कथनों पर विचार कीजिए:

  1. हाथियों के समूह का नेतृत्व मादा करती है।
  2. हाथी की अधिकतम गर्भावधि 22 माह तक हो सकती हैं
  3. सामान्यतः हाथी में 40 वर्ष की आयु तक ही बच्चे पैदा करने की क्षमता होती है।
  4. भारत के राज्यों में, हाथियों की सर्वाधिक जीवसंख्या केरल में है।

उपर्युक्त कथनों में से कौन-सा/से सही है/हैं?

  1. केवल 1 और 2
  2. केवल 2 और 4
  3. केवल 3
  4. केवल 1, 3 और 4

उत्तर (a) हाथी समूह हमेशा मातृसत्तात्मक समाज होते हैं, जिनमें पुरुष दूर रहते हैं, एवं केवल प्रजनन के लिए ही साथ आते हैं। झुंड का नेतृत्व मादा करती है। अतः, कथन 1 सही है एवं विकल्प (b) एवं (c) सही उत्तर नही हो सकते।

हाथियों के पास किसी भी अन्य जानवर की तुलना में सबसे लंबी गर्भावधि होती है, जो 680 दिनों यानी 22 महीने तक चलती है। अतः, कथन 2 भी सही है। इसलिए, सही उत्तर विकल्प (a) होगा।

हाथी अपनी संतान को 50 या किसी मामले में 60 वर्षों की उम्र तक जन्म दे सकते हैं।

(एक कठिन प्रश्न, लेकिन यदि आप जानते थे कि 1 सही है, तो अनुमान केवल दो विकल्पों तक सीमित हो जाता)

3. निम्नलिखित में से कौन-सा ‘संरक्षित क्षेत्र’ कावेरी बेसिन में स्थित है ?

  1. नागरहोले राष्ट्रीय उद्यान
  2. पापिकोंडा राष्ट्रीय उद्यान
  3. सत्यमंगलम बाघ आरक्षित क्षेत्र
  4. वायनाड वन्यजीव अभयारण्य

नीचे दिए गए कूट का प्रयोग कर सही उत्तर चुनिए:

  1. केवल 1 और 2
  2. केवल 3 और 4
  3. केवल 1, 3 और 4
  4. 1, 2, 3 और 4

उत्तर (C) पापिकोंडा राष्ट्रीय उद्यान गोदावरी बेसिन, आंध्र प्रदेश में स्थित है। अन्य सभी कावेरी बेसिन में हैं।


(प्रश्न का उत्तर देने के लिए सभी संरक्षित क्षेत्रों के सटीक स्थान-ज्ञान की आवश्यकता है)

4.भारत की जैव-विविधता के संदर्भ में, सीलोन फ्राॅगमाऊथ, काॅपरस्मित बार्बेट, ग्रे-चिन्ड मिनिवेट और व्हाइट-थ्रेटेड रेस्टार्ट क्या है ?

  1. पक्षी
  2. प्राइमेट
  3. सरीसृप
  4. अभयचर

उत्तर (a) वे सभी पक्षी हैं। अनुमान लगाना आसान नही था क्योंकि सभी नाम असामान्य हैं।

(इस प्रश्न के लिए पक्षी प्रजातियों के नाम के विशिष्ट ज्ञान की आवश्यकता है)

5. कान्हा राष्ट्रीय उद्यान भारतीय अनूप मृग (बारहसिंगा) की उस उपजाति, जो पक्की भूमि पर फलती-फूलती है और केवल घासभक्षी है, के संरक्षण के लिए निम्नलिखित में से कौन-सा संरक्षित क्षेत्र प्रसिद्ध है ?

  1. कान्हा राष्ट्रीय उद्यान
  2. मानस राष्ट्रीय उद्यान
  3. मुदुमलाई वन्यजीव अभयारण्य
  4. ताल छप्पर वन्यजीव अभयारण्य

उत्तर (a) कान्हा राष्ट्रीय उद्यान - भारतीय अनूप मृग (बारहसिंगा) मध्य प्रदेश का राज्य पशु है। जब कान्हा राष्ट्रीय उद्यान में एक सफल प्रजनन एवं संरक्षण कार्यक्रम से इस प्रजाति को जीवन दान मिला, तब यह विलुप्ती की कगार पर थी।

(यह एक सफल संरक्षण प्रयोग था, इसलिए प्रश्न पूछा गया)

6.इस्पात स्लेग निम्नलिखित में से किसके लिए सामग्री हो सकता है?

  1. आधार-सड़क के निर्माण के लिए
  2. कृषि मृदा के सुधार के लिए
  3. सीमेंट के उत्पादन के लिए

नीचे दिए गए कूट का प्रयोग कर सही उत्तर चुनिए:

  1. केवल 1 और 2
  2. केवल 2
  3. केवल 2 और 3
  4. 1, 2 और 3

उत्तर (d) इस्पात स्लैग, इस्पात बनाने का एक उप-उत्पाद है, जो इस्पात बनाने वाली भट्टियों में अशुद्धियों से पिघले हुए इस्पात के पृथक्करण के दौरान निर्मित होता है। स्लैग तरल होता है एवं सिलिकेट एवं ऑक्साइड का एक जटिल मिश्रण होता है जो ठंडा होने पर जम जाता है।

इसके हाइड्रोलिक गुण एवं अधिक सहन क्षमता के कारण, इस्पात स्लैग का उपयोग आधार-सड़क सामग्री के रूप में किया जाता है। उच्च कण घनत्व एवं कठोरता के साथ, इस्पात स्लैग टूट-फूट का बेहतर प्रतिरोधी है इसलिए इसे डामर कंक्रीट के साथ उपयोग किया जाता है।

स्थिरीकरण सामग्री के रूप में इस्पात स्लैग की उपस्थिति कृषि मृदा की उपजाऊ क्षमता में सुधार करती है।

इस्पात स्लैग का उपयोग ओपीसी क्लिंकर एवं ब्लास्ट फर्नेस स्लैग के साथ पीसकर ऊर्जा की बचत करते हुए सीमेंट का उत्पादन किया जा सकता है। ऊर्जा की बचत वाले इस्पात-स्लैग-सीमेंट में दीर्घकालिक शक्ति विकास, अच्छा स्थायित्व एवं बहुत कम हाईड्रेशन उष्मा होती है। इसलिए सभी तीन उपयोग संभव हैं।

(कठिन प्रश्न)

7. निम्नलिखित में से कौन-से ऐसे सर्वाधिक ऐसे सर्वाधिक संभावनीय स्थान हैं जहाँ कस्तूरी मृत अपने प्राकृतिक आवास में मिल सकता है ?

  1. अस्कोट वन्यजीव अभयारण्य
  2. गंगोत्री राष्ट्रीय उद्यान
  3. किशनपुर वन्यजीव अभयारण्य
  4. मानस राष्ट्रीय उद्यान

नीचे दिए गए कूट का प्रयोग कर सही उत्तर चुनिए:

  1. केवल 1 और 2
  2. केवल 2 और 3
  3. केवल 3 और 4
  4. केवल 1 और 4

उत्तर (a) अस्कोट कस्तूरी मृग अभयारण्य भारत के उत्तराखंड राज्य में अस्कोट के पास पिथौरागढ़ से 54 किमी दूर स्थित है। इस अभयारण्य की स्थापना मुख्य रूप से कस्तूरी मृग (मॉस्कस ल्यूकोसेस्टर) एवं इसके निवास स्थान के संरक्षण के उद्देश्य से की गई है। अतः सही उत्तर (a) या (d) हो सकता है।

गंगोत्री राष्ट्रीय उद्यान हिम तेंदुए का घर है। आज तक, पार्क में 15 स्तनपायी प्रजातियों एवं 150 पक्षियों की प्रजातियों को प्रलेखित किया गया है, जिनमें एशियाई काले भालू (उर्सस थिबेटेनस), भूरे भालू (उर्सस आर्कटोस), कस्तूरी मृग (मॉस्कस क्राइसस्टर), नीली भेड़ (स्यूडोस नयौर), हिमालयन तहर (हेमित्रैगस जेमाहेलिकस), हिमालयन मोनाल (लोफोफोरस इम्पेनेजस), कोकलस (पुकारिया मैक्रोलोफा) एवं हिमालयन स्नोकोक (टेट्राओगैलस हेलासिनेसिस), तीतर, दलिया, कबूतर एवं कबूतर शामिल हैं। इसलिए सही उत्तर विकल्प (a) है।

कस्तूरी मृग एक छोटा हिरण है जो ऊंचाई में लगभग 3 फीट तथा इसका वजन 11-18 किलो होता है। इसका शरीर सफेद भूरे रंग की पट्टी के साथ गले से ठोड़ी तक रेतीले भूरे रंग का होता है। शरीर आगे की ओर ढलान मे होता हैं, तथा आगे के पैर पिछले पैरों की तुलना में एक तिहाई से अधिक लंबे होते हैं। कान बड़े एवं गोल होते हैं। इसके कैनाइन दांत मुंह से बाहर निकले होते हैं। हिरण दो साल की उम्र में प्रजनन कर सकता है एवं आमतौर पर सिर्फ एक बच्चा ही पैदा करता है।

(पूर्णतः आँकड़ों पर आधारित एक कठिन प्रश्न)

8. ग्रामीण सड़क निर्माण में, पर्यावरणीय दीर्घोपयोगिता को सुनिश्चित करने अथवा कार्बन पदचिन्ह को घटाने के लिए निम्नलिखित में से किसके प्रयोग को अधिक प्राथमिकता दी जाती है ?

  1. ताम्र स्लैग
  2. शीत मिश्रित ऐस्फाल्ट प्रौद्योगिकी
  3. जीयोटेक्सटाइल्स
  4. उष्ण मिश्रित ऐस्फाल्ट प्रौद्योगिकी
  5. पोर्टलैंड सीमेंट

नीचे दिए गए कूट का प्रयोग कर सही उत्तर चुनिए:

  1. केवल 1, 2 और 3
  2. केवल 2, 3 और 4
  3. केवल 4 और 5
  4. केवल 1 और 5

उत्तर (a) सरकारी योजनाओं एवं पर्यावरण-पारिस्थितिकी दोनों पर आधारित एक प्रश्न। वैश्विक उष्मण के लगातार खतरे को देखते हुए, प्रधान मंत्री ग्राम सड़क योजना (PMGSY) ‘कार्बन फुटप्रिंट’ को कम करने के लिए हरित प्रौद्योगिकियों पर ध्यान केंद्रित कर रही है। राज्य न्यूनतम 15% सड़कों का चयन किया गया हैं, जिसमें प्लास्टिक के कचरे, फ्लाई ऐश, ताम्र एवं लौह स्लैग, जीयोटेक्सटाइल, शीत मिश्रित ऐस्फाल्ट, पैनल्ड सीमेंट कंक्रीट एवं सेल से भरे कंक्रीट का उपयोग किया गया है। सड़कों के स्थिरीकरण के लिए नैनो प्रौद्योगिकी उत्पादों का भी उपयोग किया जा रहा है। अतः कथन 1, 2 एवं 3 सही हैं।

उष्ण मिश्रित ऐस्फाल्ट (एचएमए) लगभग 95% पत्थर, रेत एवं बजरी का एक संयोजन है, जो कच्चे तेल के उत्पाद डामर सीमेंट के साथ मिश्रित होता है। यह एक हरित विकल्प नहीं है।

पोर्टलैंड सीमेंट हाइड्रोलिक बंधन सामग्री है जिसमें पोर्टलैंड सीमेंट क्लिंकर, 0.5% चूना पत्थर या दानेदार ब्लास्ट फर्नेस स्लैग एवं सीमित मात्रा में जिप्सम शामिल होते है।

(प्रदार्थों एवं सरकारी योजनाओं के सूक्ष्म ज्ञान पर आधारित प्रश्न)

9. निम्नलिखित कथनों पर विचार कीजिए:

  1. कोयले की राख में आर्सनिक, सीसा और पारद अंतर्विष्ट होते हैं।
  2. कोयला संचालित विद्युत् संयंत्र पर्यावरण में सल्फर डाईआक्साइड और नाइट्रोजन के आक्साइड उत्सर्जित करते हैं।

उपर्युक्त कथनों में से कौन-सा/से सही है/हैं?

  1. केवल 1
  2. केवल 2 और 3
  3. केवल 3
  4. 1, 2 और 3

उत्तर (d) तीनों सही हैं। भारत के घरेलू कोयला भंडार में राख की मात्रा 40 से 45 प्रतिशत तक है। कोयला अपने आप में एक विषैला पदार्थ नहीं है। लेकिन जलने के बाद, उसकी राख में सीसा, पारा, कैडमियम, क्रोमियम, आर्सेनिक एवं सेलेनियम शामिल होते हैं जो मानव के अस्वास्थ्यकारी होते हैं। अतः कथन 1 एवं 3 सही हैं। केवल विकल्प (d) संभव है। कोयला आधारित बिजली संयंत्रों पर कई सारे समाचार कवरेज लंबे समय से मौजूद हैं।

(एक किया जा सकने वाला प्रश्न)

10. खेती में बायोचार का क्या उपयोग है ?

  1. बायोचार ऊध्र्वाधर खेती (Vertical farming) में वृद्धिकर माध्यम के अंश के रूप में प्रयुक्त किया जा सकता है।
  2. जब बायोचार वृद्धिकर माध्यम के अंश के रूप में प्रयुक्त किया जाता है, तो वह नाइट्रोजन-यौगिकीकारी सूक्ष्मजीवों की वृद्धि को बढ़ावा देता है।
  3. जब बायोचार वृद्धिकर माध्यम के अंश के रूप में प्रयुक्त किया जाता है, तब वह उस वृद्धिकर माध्यम की जलधारण क्षमता को अधिक लम्बे समय तक बनाए रखने में सहायक होता है।

उपर्युक्त कथनों में से कौन-सा/से सही है/हैं ?

  1. केवल 1 और 2
  2. केवल 2
  3. केवल 1 और 3
  4. 1, 2 और 3

उत्तर (d) तीनों सही हैं। बायोचार एक ठोस कार्बोनेसस उत्पाद है जिसे पौधे एवं/या पशु बायोमास से पायरोलिसिस के के अंर्तगत प्राप्त किया जाता है। पाइरोलिसिस ऑक्सीजन की अनुपस्थिति में रासायनिक पदार्थों को ऊंचे तापमान पर रासायनिक रूप से विघटित करने की प्रक्रिया है।

वृद्धिकर माध्यम के रूप में बायोचार अनुप्रयोग से मृदा के जैविक नाइट्रोजन निर्धारण में 50% से 72% की वृद्धि पाई गई है। मृदा पोषक तत्व एवं जल धारण क्षमता, पीएच, बल्क घनत्व एवं मृदा माइक्रोबियल गतिविधियों की वृद्धि के माध्यम से मिट्टी के भौतिक, रासायनिक एवं जैविक गुणों पर बायोचार का प्रभाव जैव ईंधन में मृदा रोगाणुओं के एकत्रीकरण, छिद्र, सतह क्षेत्र एवं आवास में सुधार के द्वारा होता है। अतः कथन 2 एवं 3 सही हैं एवं उत्तर (d) होना चाहिए।

(नई तकनीकों पर आधारित पर्यावरण एवं पारिस्थितिकी के साथ ही कृषि से संबंधित प्रश्न)


11. भारत में, निमनलिखित में से किन्हें कृषि में सार्वजनिक निवेश माना जा सकता है ?

  1. सभी फ़सलों के कृषि उत्पाद के लिए न्यूनतम समर्थन मूल्य निर्धारित करना
  2. प्राथमिक कृषि साख समितियों का कम्प्यूटरीकरण
  3. सामाजिक पूँजी विकास
  4. कृषकों के निःशुल्क बिजली की आपूर्ति
  5. बैंकिंग प्रणाली द्वारा कृषि ऋण की माफ़ी
  6. सरकारों द्वारा शीतागार सुविधाओं को स्थापित करना

नीचे दिए गए कूट का प्रयोग कर सही उत्तर चुनिए:

  1. केवल 1, 2 और 5
  2. केवल 1, 3, 4 और 5
  3. केवल 2, 3 और 6
  4. 1, 2, 3, 4, 5 और 6

उत्तर (c) अत्यंत आसान प्रश्न। न्यूनतम सर्मथन मूल्य निर्धारित करना, कृषि में ‘सार्वजनिक निवेश’ नहीं है, बल्कि न्यूनतम आय सुनिश्चित करने के लिए किसानों के लिए एक सामाजिक सहायता प्रणाली है। यह सरकार द्वारा सर्मथन मूल्य के रूप में प्रत्यक्ष व्यय है। यदि आप इस बिंदु को ध्यान में रखते हैं, तब विकल्प (a), (b) एवं (d) तुरंत समाप्त हो जाते हैं, एवं (c) उत्तर है। {एक महत्वपूर्ण बिंदु - कथन 1 में लिखा है - ‘सभी फसलों के कृषि उत्पाद के लिए न्यूनतम सर्मथन मूल्य तय करना’। भारत सरकार, वैसे भी सभी फसलों के लिए यूनतम सर्मथन मूल्य प्रदान नहीं करती है, बल्कि 30 से कम फसलों के लिए ऐसा करती है।’

कृषि में सार्वजनिक क्षेत्र का निवेश तीन उप क्षेत्रों में विघटित होता है - कृषि, वानिकी एवं मत्स्य। मद - फसल पालन, मृदा एवं जल संरक्षण, पशुपालन, डेयरी विकास, मछली पालन, वानिकी एवं वन्यजीव, वृक्षारोपण, खाद्य भंडारण एवं भंडारण, कृषि अनुसंधान एवं शिक्षा, सहकारिता, अन्य कृषि कार्यक्रम, ग्रामीण विकास, सिंचाई एवं बाढ़ नियंत्रण। इसलिए 2, 3 एवं 6 वैध हैं।

(यदि आप जानते हैं कि न्यूनतम सर्मथन मूल्य ‘सार्वजनिक निवेश’ नही है तो एक बहुत आसान प्रश्न)

12.भारत में फर्म के ‘‘ब्याज-व्याप्ति अनुपात (Interest Coverage Ratio)’’ पद का क्या महत्व है ?

  1. यह उस फर्म, जिसे बैंक ऋण देने जा रहा है, के वर्तमान जोखिम को समझने में मदद करता है
  2. यह उस फर्म, जिसे बैंक ऋण देने जा रहा है, के आने वाले जोखिक के मूल्यांकन में मदद करता है।
  3. उधार लेने वाली फर्म का ब्याज-व्याप्ति अनुपात जितना अधिक होगा, उसकी ऋण समाशोधन क्षमता उतनी ही खराब होगी।
  4. नीचे दिए गए कूट का प्रयोग कर सही उत्तर चुनिए:

  1. केवल 1 और 2
  2. केवल 2
  3. केवल 1 और 3
  4. 1, 2 और 3

उत्तर (a) कथन 1 एवं 2 सही हैं। ब्याज-व्याप्ति अनुपात - किसी कंपनी की आय (ब्याज एवं करों से पहले) को एक निश्चित अवधि के दौरान कंपनी के ब्याज भुगतान द्वारा उसी अवधि के भीतर विभाजित करके प्राप्त किया जाता है। इससे हमें कई बातें पता चलती हैं। यह एक ऋण एवं लाभप्रदता अनुपात है जिसका उपयोग यह निर्धारित करने के लिए किया जाता है कि कोई कंपनी अपने बकाया ऋण पर कितनी आसानी से ब्याज का भुगतान कर सकती है। ऋणदाता, निवेशक एवं लेनदार अक्सर इस अवधारणा का उपयोग कंपनी के मौजूदा ऋण के सापेक्ष या भविष्य में उधार लेने के जोखिम के मूल्यांकन के लिए करते हैं। एक उच्च कवरेज अनुपात कंपनियों के ऋण से कम जोखिम दिखाता है, एवं बेहतर है। अतः 3 गलत है। इसलिए (c) एवं (d) गलत हैं।

(यदि आप दिए गए शब्द की परिभाषा जानते हैं तो इसका उत्तर दिया जा सकता था)

13. हाल के बीते दिनों में निम्नलिखित में से कौन-से कारक/ कौन-सी नीतियाँ भारत में चावल के मूल्य केा प्रभावित कर रही थीं ?

  1. न्यूनतम समर्थन मूल्य
  2. सरकार द्वारा व्यापार करना
  3. सरकार द्वारा भण्डारण करना
  4. उपभोक्ता सहायिकियाँ (subsidies)

नीचे दिए गए कूट का प्रयोग कर सही उत्तर चुनिए:

  1. केवल 1, 2 और 4
  2. केवल 1, 3 और 4
  3. केवल 2 और 3
  4. 1, 2, 3 और 4

उत्तर (d) किसी भी वस्तु की कीमत मांग के सापेक्ष बाजार में उसकी आपूर्ति पर निर्भर करती है। 2 एवं 3 दोनों ही बाजार में चावल की आपूर्ति को प्रभावित करते हैं, इसलिए कीमत को प्रभावित करते हैं। बाजार में हस्तक्षेप द्वारा सरकार भी कीमतें प्रभावित कर सकती हैं। इसलिए 1 एवं 4 भी सही हैं। इनमें से कुछ कीमतें बढ़ा सकती हैं एवं कुछ कम की सकती हैं।

(कठिन प्रश्न, क्योंकि इसे हल करने के लिए सरकार एवं बाजार स्तर पर फसल के खरीद एवं बिक्री के प्रक्रिया की संपूण जानकारी होना चाहिए)

14.निम्नलिखित कथनों पर विचार कीजिए:

  1. पिछले दशक में भारत-श्रीलंका व्यापार के मूल्य में सतत वृद्धि हुई है
  2. भारत और बांग्लादेश के बीच होने वाले व्यापार में ‘‘कपड़े और कपड़े से बनी चीज़ों’’ का व्यापार प्रमुख है
  3. पिछले पाँच वर्षों में, दक्षिण एशिया में भारत के व्यापार का सबसे बड़ा भागीदार नेपाल रहा है

उपर्युक्त कथनो में से कौन-सा/से सही है/हैं ?

  1. केवल 1 और 2
  2. केवल 2
  3. केवल 3
  4. 1, 2 और 3

उत्तर (a) बांग्लादेश, दक्षिण एशिया में भारत का सबसे बड़ा व्यापारिक भागीदार है, उसके बाद नेपाल, श्रीलंका, पाकिस्तान, भूटान, अफगानिस्तान एवं मालदीव हैं। भारतीय निर्यात का स्तर भी इसी क्रम में है। इसलिए कथन 3 सही नहीं है, उत्तर विकल्प (a) या (b) हो सकता है।

चूंकि भारत एवं श्रीलंका के मध्य एक द्विपक्षीय मुक्त व्यापार समझौते पर हस्ताक्षर किए गए थे एवं यह 2000 में प्रभाव में आया था, 2004 तक इंडो-श्रीलंकाई व्यापार 128% बढ़ा एवं 2006 तक चैगुना होकर यूएस $ 2.6 बिलियन तक पहुंच गया। अतः 1 सही है, एवं (b) सही उत्तर है।

(समसामयिकी आधारित प्रश्न। स्मरणीय है कि बांग्लादेश की अर्थव्यवस्था नेपाल की तुलना में 10 गुना अधिक है। अतः 3 सही उत्तर का हिस्सा नही हो सकता)

15. निम्नलिखित में से किस समूह के सभी चारो देश G20 के सदस्य हैं ?

    1. अर्जेंटीना, मेक्सिको, दक्षिण अफ्रीका एवं तुर्की
    2. आस्ट्रेलिया, कनाडा, मलेशिया एवं न्यूज़ीलैण्ड
    3. ब्राज़ील, ईरान, साऊदी अरब एवं वियतनाम
    4. इंडोनेशिया, जापान, सिंगापुर एवं दक्षिण कोरिया

उत्तर (a) आपको जी 20 समूह के सदस्य राष्ट्रों के नाम याद होना चाहिए!

जी 20 में 19 देश एवं यूरोपीय संघ शामिल हैं। 19 देश अर्जेंटीना, ऑस्ट्रेलिया, ब्राजील, कनाडा, चीन, जर्मनी, फ्रांस, भारत, इंडोनेशिया, इटली, जापान, मैक्सिको, रूस, सऊदी अरब, दक्षिण अफ्रीका, दक्षिण कोरिया, तुर्की, यूनाइटेड किंगडम एवं संयुक्त राज्य अमेरिका हैं। ध्यान दें कि ईरान जी 20 का हिस्सा नहीं है, न ही मलेशिया। सिंगापुर एक शहर-राज्य है।

(यह प्रश्न गलत नहीं होना चाहिए, क्योंकि जी 20 सतत खबरों में हैं)


16. ‘किसान क्रेडिट कार्ड’योजना के अन्तर्गत, निम्नलिखित में से किन-किन उद्देश्यों के लिए कृषकों को अल्पकालीन ऋण समर्थन उपलब्ध कराया जाता है ?

  1. फर्म परिसंपत्तियों के रख-रखाव हेतु कार्यशील पूँजी के लिए
  2. कम्बाइन कटाई मशीनों, ट्रैक्टरों एवं मिनी ट्रकों के क्रय के लिए
  3. फार्म परिवारो की उपभोग आवश्यकताओं के लिए
  4. फ़सल कटाई के बाद के खर्चों के लिए
  5. परिवार के लिए घर निर्माण एवं गाँव में शीतागार सुविधा की स्थापना के लिए

नीचे दिए गए कूट का प्रयोग कर सही उत्तर चुनिए:

  1. केवल 1, 2 और 5
  2. केवल 1, 3 और 4
  3. केवल 2, 3, 4 और 5
  4. 1, 2, 3, 4 और 5

उत्तर (b) कथन 2 एवं 5 दोनों में बताई गई गतिविधियां पूंजी-गहन हैं जिन्हें क्रेडिट कार्ड आधारित प्रणाली द्वारा कवर नहीं किया जा सकता है। किसान क्रेडिट कार्ड योजना का उद्देश्य किसानों को पर्याप्त एवं समय पर ऋण उपलब्ध कराना है। यह फसल के बाद के खर्चों के लिए ऋण, विपणन ऋण का उत्पादन, किसान परिवार की उपभोग आवश्यकताओं, कृषि संपत्तियों के रख-रखाव के लिए कार्यशील पूंजी एवं कृषि से संबद्ध गतिविधियों के लिए निवेश ऋण की आवश्यकता एवं पशु, पक्षी, मछली, झींगा, अन्य जलीय जीव, मछली आदि के पालन के लिए अल्पकालिक ऋण आवश्यकताओं को प्रदान कर सकता है। नोट - कृषि संबद्ध गतिविधियों में डेयरी पशु, अंतर्देशीय मत्स्यपालन, फूलों की खेती, बागवानी आदि शामिल हैं।

(प्रश्न सामान्य ज्ञान द्वारा हल किया जा सकता था)

17.निम्नलिखित कथनों पर विचार कीजिए:

  1. खाद्य वस्तुओं का ‘उपभोक्ता मूल्य सूचकांक’(CPI) में भार (weightage) उनके ‘थोक मूल्य सूचकांक’ (WPI) में दिए गए भार से अधिक है।
  2. WPI, सेवाओं के मूल्यों में होने वाले परिवर्तनों को नहीं पकड़ता, जैसा कि CPI करता है।
  3. भारतीय रिज़र्व बैंक ने अब मुद्रास्फीति के मुख्य मान हेतु तथा प्रमुख नीतिगत दरों के निर्धारण और परिवर्तन हेतु WPI को अपना लिया है।

उपर्युक्त कथनों में से कौन-सा/से सही है/हैं ?

  1. केवल 1 और 2
  2. केवल 2
  3. केवल 3
  4. 1, 2 और 3

उत्तर (a) उपभोक्ता मूल्य सूचकांक एवं थोक मूल्य सूचकांक पर आधारित एक आसान प्र्रश्न!

अप्रैल 2014 में, भारतीय रिज़र्व बैंक ने मुद्रास्फीति के प्रमुख उपाय के रूप में उपभोक्ता मूल्य सूचकांक को अपनाया। संपूर्ण एफआईटी (लचीली मुद्रास्फीति लक्ष्यीकरण) इस पर आधारित है। अतः कथन 3 गलत है। उत्तर (a) या (b) हो सकता है। ‘उपभोक्ता मूल्य सूचकांक’ में खाद्यान्न का भार ‘थोक मूल्य सूचकांक’ की तुलना में अधिक है। अतः सही उत्तर विकल्प (a) है।

(यह प्रश्न गलत नहीं होना चाहिए)

18.निम्नलिखित युग्मों पर विचार कीजिए:

नदी - में जाकर मिलती है

  1. मेकोंग - अण्डमान सागर
  2. थेम्स - आयरिश सागर
  3. वोल्गा - कैस्पियन सागर
  4. ज़म्बेजी - हिन्द महासागर

उपर्युक्त युग्मों में से कौन-सा/से सही समुलित है/हैं ?

  1. केवल 1 और 2
  2. केवल 3
  3. केवल 3 और 4
  4. केवल 1, 2 और 4

उत्तर (c) बुनियादी भौगोलिक विवरणों पर आधारित एक प्रश्न। केवल 3 एवं 4 सही हैं। मेकांग नदी तिब्बती पठार पर शुरू होती है एवं दक्षिण चीन सागर में गिरती है। टेम्स नदी उत्तरी सागर में टेम्स इस्ट्यूरी से होकर बहती है। वोल्गा नदी कैस्पियन सागर में गिरती है। ज़ाम्बिया में कालेनी पहाड़ियों से उभरते हुए, ज़म्बेजी नदी मोजाम्बिक तट पर हिंद महासागर में गिरती है।

(इसका उत्तर सभी उम्मीदवारों को पता होना चाहिए)

19. निम्नलिखित कथनों पर विचार कीजिए:

  1. सभी अनाजों, दालों एवं तिलहनों का ‘न्यूनतम समर्थन मूल्य’ (MSP) पर प्रापण (खरीद) भारत के किसी भी राज्य/केंद्र शासित प्रदेश (यू.टी.) में असीमित होता है।
  2. अनाजों एवं दालों का डैच् किसी किसी भी राज्य/ केन्द्र-शासित प्रदेश में उस स्तर पर निर्धारित किया जाता है जिस स्तर पर बाज़ार मूल्य कभी नहीं पहुँच पाते।

उपर्युक्त कथनों में से कौन-सा/से सही है/हैं ?

  1. केवल 1
  2. केवल 2
  3. 1 और 2 दोनो
  4. न तो 1, न ही 2

उत्तर (d) दोनों ही कथन सही नही हैं। नई PM-AASHA योजना के तहत, मौजूदा मूल्य समर्थन योजना (PSS) दलहन, तिलहन एवं खोपरा (लेकिन अनाज नहीं) के लिए जारी रहेगी। NAFED एवं भारतीय खाद्य निगम (केंद्रीय एजेंसियों) जब भी उपज की बाजार मूल्य, न्यूनतम सर्मथन मूल्य से नीचे जाएगा, इसकी खरीद जारी रखेंगी। अतः कथन 1 गलत है।

PM-AASHA - प्रधान मंत्री अन्नदाता आय सरंक्षण अभियान

कृषि लागत एवं मूल्य आयोग (CACP) की सिफारिशों के आधार पर केंद्र द्वारा 23 कृषि जिंसों के लिए न्यूनतम सर्मथन मूल्य तय किया गया है। कथन 2 भी गलत है।

(सामान्य बोध से हल किया जा सकता था क्योंकि कथन 2 निश्चित गलत है)

20. भारतीय अर्थव्यवस्था के संदर्भ में, निम्नलिखित कथनों पर विचार कीजिए:

  1. ’वाणिज्यिक पत्र (Commercial Paper)’ अल्पकालीन प्रतिभूति-रहित वचन-पत्र है।
  2. जमा प्रमाण-पत्र (certificate of deposit)’ भारतीय रिज़र्व बैंक द्वारा किसी निगम को निर्गत किया जाने वाला दीर्घकालीन प्रपत्र है।
  3. शीघ्र-कूप बाण्ड (Zero-Coupon Bonds) अनुसूचित व्यापारिक बैंको द्वारा निगमों को निर्गत किए जाने वाले ब्याज सहित अल्पकालीन बाण्ड हैं।

उपर्युक्त कथनों में से कौन-सा/से सही है/हैं ?

  1. केवल 1 और 2
  2. केवल 4
  3. केवल 1 और 3
  4. केवल 2, 3 और 4

उत्तर (c) कथन 1 एवं 3 सही हैं। वाणिज्यिक पत्र - कॉरपोरेशनों द्वारा जारी किया जाने वाला एक सामान्य प्रकार का असुरक्षित, अल्पकालिक ऋण साधन है। इसका उपयोग पेरोल के वित्तपोषण, देय खातों एवं इन्वेंटरी के लिए किया जाता है। कथन 1 सही है। उत्तर (a) या (c) हो सकता है।

जमा प्रमाण पत्र - एक प्रकार का मुद्रा बाजार साधन है, जो किसी निवेशक द्वारा बैंक के साथ जमा किए गए धन के विरूद्ध एक विशिष्ट संक्षिप्त अवधि के लिए विमुद्रीकृत रूप में जारी किया जाता है। यह भारिबैं द्वारा विनियमित है। कथन 2 सही नहीं है। अतः उत्तर विकल्प (c) है।

शीघ्रावधि द्रव्य - एक अल्पकालिक, ब्याज देने वाला ऋण है जो एक वित्तीय संस्था से दूसरे वित्तीय संस्थान को दिया जाता है।

शून्य-कूपन बॉन्ड - एक ऋण सुरक्षा है जो ब्याज का भुगतान नहीं करती है, बल्कि एक गहरी छूट पर ट्रेड करती है, जब बांड को उसके पूर्ण मूल्य के लिए भुनाया जाता है तब परिपक्वता पर लाभ प्रदान करती है।

(प्रश्न को हल करने के लिए बुनियादी परिभाषाएं याद करने की आवश्यकता है जो आप को याद होना चाहिए! एवं 4 निश्चित गलत है इसलिए दो विकल्प समाप्त हो जाते हैं)


21. यदि किसी पौधे की विशिष्ट जाति को वन्यजीव सुरक्षा अधिनियम, 1972 की अनुसूची टप् में रखा गया है, तो इसका क्या तात्पर्य है ?

  1. उस पौधे की खेती करने के लिए लायसेंस की आवश्यकता है।
  2. ऐसे पौधे की खेती किसी भी परिस्थिति में नहीं हो सकती।
  3. यह एक अनुवंशिकतः रूपांतरित फ़सली पौधा है ।
  4. ऐसा पौधा आक्रामक होता है और पारितंत्र के लिए हानिकाकर होता है

उत्तर (b) वाइल्ड लाइफ प्रोटेक्शन एक्ट, 1972 भारत की संसद का एक अधिनियम है जो पौधों एवं प्रणियों की प्रजातियों के संरक्षण के लिए लागू किया गया है। 1972 से पहले, भारत में केवल पांच नामित राष्ट्रीय उद्यान थे। अन्य सुधारों के बीच, अधिनियम ने संरक्षित पौधे एवं प्राणि प्रजातियों के कार्यक्रम स्थापित किए। इन प्रजातियों का शिकार या कटाई बड़े पैमाने पर गैरकानूनी थी।

इसमें छह अनुसूचियां हैं जो सुरक्षा के अलग-अलग अंश प्रदान करती हैं। अनुसूची I एवं अनुसूची II के भाग II में सूचीबद्ध प्रजातियों को पूर्ण सुरक्षा मिलती है - इनके तहत किए गए अपराध पर उच्चतम दंड का प्रावधान हैं। अनुसूची III एवं अनुसूची IV में सूचीबद्ध प्रजातियां भी संरक्षित हैं, लेकिन दंड बहुत कम हैं। अनुसूची V में वे प्राणि शामिल हैं जिनका शिकार किया जा सकता है। अनुसूची VI में पौधों को खेती एवं रोपण से प्रतिबंधित किया गया है।

अधिनियम जंगली जानवरों, पक्षियों एवं पौधों को आकस्मिक चिकित्सा से संबंधित मामलों में सुरक्षा प्रदान करता है। यह संपूर्ण भारत में वैध है। इसमें छह अनुसूचियां हैं जो सुरक्षा के अलग-अलग अंश प्रदान करती हैं। अनुसूची VI में निर्दिष्ट स्थानिक पौधों को खेती एवं रोपण से प्रतिबंधित किया गया है। अद्यतन, पौधों की सूची है - 1. बेडोमेड्स साइकैड (साइकस बेडडोमि), 2. ब्लू वांडा (वांडा ेवमतनसमब), 3. कुथ (सौसुरिया लप्पा), 4. महिलाओं के चप्पल ऑर्किड (पापीओपेडिलम एसपीपी।), 5. पिचर प्लांट (नेपेंथेस खसियाना), 6. रेड वांडा (रैननथेरा इनकूटियाना)।

(यह एक अच्छा, तार्किक प्रश्न था जिसमें वांछित नियम के बारे में मुलभुत जानकारी की आवश्यकता थी)

22. प्राचीन भारतीय गुप्त राजवंश के समय के संदर्भ में, नगर घंटाशाला, कदूरा तथा चौल किस लिए विख्यात थे ?

  1. विदेशी व्यापार करने वाले बंदरगाह
  2. शक्तिशाली राज्यों की राजधानियाँ
  3. उत्कृष्ट प्रस्तर कला तथा स्थापत्य से संबंधित स्थान
  4. बौद्ध धर्म के महत्वपूर्ण तीर्थस्थल

उत्तर (a) सभी स्थान बंदरगाह थे। एक कठिन प्रश्न क्योंकि दिए गए नाम सामान्य नही थे।

गुप्त साम्राज्य के दौरान, भूमि एवं समुद्र दोनों के माध्यम से, आंतरिक एवं विदेशी व्यापार फला-फूला। व्यापार के मुख्यतः कपड़े, खाद्यान्न, मसाले, नमक एवं कीमती पत्थरों का किया जाता था। भारत से मोती, कीमती पत्थरों, कपड़े, इत्र, मसाले, नील, दवाईयों, नारियल एवं हाथी दांत के सामानों का निर्यात तथा सोने, चांदी, टिन, सीसा, रेशम एवं घोड़े का आयात किया जाता था।

स्त्रोत -https://www.nodc.noaa.gov/archive/arc0001/9900162/2.2/data/0-data/jgofscd/htdocs/marinearc/ancientwreck.html से हमें यह मिला -

भारत में जहाज निर्माण, बंदरगाह एवं गोदामों की स्थापना के प्रमाण हड़प्पा संस्कृति के समय से पाए गए हैं। महत्वपूर्ण हड़प्पा एवं पूर्व-हड़प्पा बंदरगाह लोथल, लखावल, किंडरखेड़ा, कुंतसी, मेघम, प्रभास, टोडियो, अमरा इत्यादि थे। इन स्थलों की खुदाई में बहरीन द्वीप, फारसी खाड़ी, मिस्र एवं मेसोपोटामिया शहरों की प्राचीन वस्तुएँ प्राप्त हुई हैं। बुद्ध, मौर्य, गुप्त एवं बाद के काल में भी यही सामुद्रिक परंपराएँ चलती रहीं। ऐतिहासिक अवधि के दौरान भारत का मिस्र, रोम, यूनानियों, अरबों, चीन एवं सभी सबसे दक्षिण पूर्व एशियाई देशों के साथ व्यापार एवं सांस्कृतिक संपर्क था। भारत के दोनों तटों कई बंदरगाहों से जुड़े हुए थे। पश्चिमी तट पर स्थित बंदरगाहों में बरगया, सुप्पारा, कलीना, सेम्यला, मंडागोर, पलापेटमे, मालीज़िगारा, आॅर्नाेबोबस, बैज़न्टाईन, नौरा, टायंडिस, मुजिरिस एवं नेल्सींडा थे एवं पूर्वी तट पर ताम्रलिप्ति, चारित्रपुर, पलामू, पीथुंडा, सोपतामा, घंटासला, पोडुका, पुहार, कोरकाई एवं कैमारा, तट थे। पश्चिमी तट पर मालवन, सोपारा, एलीफेंटा द्वीप, चायल, उदयवारा, होन्नावर, गोपाकपटना एवं भारत के पूर्वी तट पर पूम्पुहार एवं ट्रेंक्यूबार के आसपास के क्षेत्र मणिकपाटना, कलिंगपत्ना में समुद्रतटीय अन्वेषण किए गए हैं।

(कठिन, तथ्यात्मक प्रश्न)

23. कृषि में शून्य-जुताई (zero-tillage) का/के क्या लाभ है/हैं?

  1. पिछली फ़सल के अवशेषों को जलाए बिना गेहूँ की बुआई संभव है
  2. चावल की नई पौध की नर्सरी बनाए बिना, धान के बीजों का नम मृदा में सीधे रोपण संभव है।
  3. मृदा में कार्बन पृथक्करण संभव है

नीचे दिए गए कूट का प्रयोग कर सही उत्तर चुनिए :

  1. केवल 1 और 2
  2. केवल 2 और 3
  3. केवल 3
  4. 1, 2 और 3

उत्तर (d) तीनों कथन सही हैं। जुताई - यांत्रिक माध्यम से कृषि भूमि की तैयारी है जिसमें खुदाई, घुमाना एवं पलटना शामिल है। शून्य-जुताई कृषि, जुताई का उपयोग किए बिना फसल उगाने की तकनीक है। सैद्धांतिक रूप से, यह मिट्टी को होने वाली क्रमिक हानि से बचाने में मदद करती है।

शून्य-जुताई कृषि में, फसल बीज को बिना भूमि की तैयारी के एवं पूर्ववर्ती फसल के अवशेषों को हटाए बिना ड्रिलर्स के माध्यम से बोया जाता है। शून्य-जुताई खेती की लागत को कम करती है एवं मिट्टी के कटाव, फसल की अवधि एवं सिंचाई आवश्यकताओं को भी कम करती है। भारत में गंगा के मैदानी इलाकों में शून्य-जुताई प्रणाली का पालन किया जा रहा है जहां चावल-गेहूं की फसलें ली जाती है। चावल की फसल के बाद गेहूं को बिना जुताई किए लगाया जाता है।

शून्य जुताई के लाभ

1. भूमि की तैयारी के लिए आदानों की लागत में कमी।

2. शुष्क पदार्थ एवं कार्बनिक पदार्थ मिट्टी में मिल जाते हैं।

3. अवशिष्ट नमी का उपयोग करके सिंचाई को कम किया जा सकता है।

4. फसल की अवधि में कमी एवं इससे जल्दी फसल लेकर उच्च पैदावार प्राप्त की जा सकती है।

5. पर्यावरण की दृष्टि से सुरक्षित - ग्रीनहाउस प्रभाव कार्बन अनुक्रम के कारण कम हो जाता है।

6. मिट्टी का संघनन कम नहीं होता एवं अपवाह द्वारा पानी की कमी को कम किया जा सकता है एवं मिट्टी का कटाव रूकता है।

(एक अच्छा प्रश्न है, लेकिन बहुत कठिन नहीं है। यदि आप जानते हैं कि अवशेष जलाना नहीं है, तो 1 सही है एवं विकल्प (b) एवं (c) समाप्त हो जाते हैं। एवं 3 को सही होना ही होगा।)

24. भारत की जैव-ईंधन की राष्ट्रीय नीति के अनुसार जैव-ईंधन के उत्पादन के लिए निम्नलिखित में से किनका उपयोग कच्चे माल के रूप में हो सकता है ?

  1. कसावा
  2. क्षतिग्रस्त गेहूँ के दाने
  3. मूँगफली के बीज
  4. कुथली (भ्वतेम हतंउ)
  5. सड़ा आलू
  6. चुकंदर

नीचे दिए गए कूट का प्रयोग कर सही उत्तर चुनिए :

  1. केवल 1, 2, 5 और 6
  2. केवल 1, 2, 3 और 6
  3. केवल 2, 3, 4 और 5
  4. 1, 2, 3, 4, 5 और 6

उत्तर (a) जैव ईंधन पर भारत की राष्ट्रीय नीति के अनुसार, गन्ने का रस, चीनी युक्त पदार्थ जैसे चुकंदर, मीठा सोरघम, स्टार्च युक्त सामग्री जैसे मकई, कसावा, क्षतिग्रस्त अनाज जैसे गेहूं, टूटे चावल, सड़े हुए आलू / वे आलू जो मानव उपभोग के लिए सही नही है का उपयोग इथेनॉल उत्पादन के लिए किया जा सकता है।

संपूर्ण जानकारी - http://petroleum.nic.in/sites/default/files/biofuelpolicy2018_1.pdf

(एक कठिन प्रश्न क्योंकि आपको सभी कच्चे फीडस्टॉक संभावनाओं को याद रखने की आवश्यकता है)

25. निम्नलिखित में से कोन-सा कथन कार्बन के सामाजिक मूल्य’पद का सर्वोत्तम रूप है ?

आर्थिक मूल्य के रूप में यह निम्नलिखित में से किसका माप है ?

  1. प्रदत्त वर्ष में एक टन CO2 के उत्सर्जन से होने वाली दीर्घकालीन क्षति।
  2. किसी देश की जीवाश्म ईंधनों की आवश्यकता, जिन्हें जलाकर देश अपने नागरिकों को वस्तुएँ और सेवाएँ प्रदान करता है
  3. किसी जलवायु शरणार्थी (Climate refugee) द्वारा किसी नए स्थान के प्रति अनुकूलित होने हेतु किए गए प्रयास
  4. पृथ्वी ग्रह पर किसी व्यक्ति विशेष द्वारा अंशदत कार्बन पदचिन्ह

उत्तर (a) एक उत्कृष्ट प्रश्न। कार्बन के सामाजिक मूल्य (SCC) की अवधारणा मौद्रिक संदर्भ में CO2 के एक अतिरिक्त टन उत्सर्जन की सभी मात्रात्मक लागतों एवं लाभों को जोड़ने की कोशिश करती है। इसका उपयोग उत्सर्जन में कटौती की लागत के मुकाबले उष्मण के कम होने पर होन वाले लाभों को की तुलना के लिए किया जा सकता है।

ईपीए एवं अन्य संघीय एजेंसियां, नियम-निर्धारण के जलवायु प्रभावों को महत्व देने के लिए कार्बन (SC-2 ) की सामाजिक लागत का अनुमान लगाती हैं। (CO2 ) एक वर्ष में कार्बन डाइऑक्साइड (CO2 ) उत्सर्जन के एक टन द्वारा किए गए दीर्घकालिक क्षति का माप है।

संदर्भ स्रोत https://www.carbonbrief.org/qa-social-cost-carbon

(पर्यावरण एवं पारिस्थितिकी का बेहतर तरिके से बनाया गया गया प्रश्न)

26. भारत में दालों के उत्पादन के संदर्भ में, निम्नलिखित कथनों पर विचार कीजिए :

  1. उड़द की खेती खरीफ़ ओर रबी दोनो फ़सलों में की जा सकती है ।
  2. कुल दाल उत्पादन का लगभग आधा भाग केवल मूँग का होता है ।
  3. पिछले तीन दशको में, जहाँ खरीफ़ दालों का उत्पादन बढ़ा है, वहीं रबी दालों का उत्पादन घटा है ।

उपर्युक्त कथनों में से कौन-सा/से सही है/हैं ?

  1. केवल 1
  2. केवल 2 और 3
  3. केवल 2
  4. 1, 2 और 3

उत्तर (a) भारत में पिछले कुछ वर्षों में दालों के उत्पादन में बदलाव देखा गया है। आज, भारत सबसे बड़ा उत्पादक (वैश्विक उत्पादन का 25%), उपभोक्ता (विश्व उपभोग का 27%) एवं दुनिया में दालों का आयातक (14%) है। मध्यप्रदेश, महाराष्ट्र, राजस्थान, उत्तर प्रदेश एवं कर्नाटक शीर्ष पांच दलहन उत्पादक राज्य हैं।

उड़द - खरीफ के दौरान, पूरे देश में इसकी खेती की जाती है। यह भारत के दक्षिणी एवं दक्षिण-पूर्वी हिस्सों में रबी के दौरान चावल की फलियों के लिए सबसे उपयुक्त है। कथन 1 सही है, इसलिए उत्तर (a) या (d) हो सकता है। उड़द (सभी प्रकार के काले, हरे इत्यादि को मिलाकर) कुल दाल उत्पादन में लगभग 40 प्रतिशत हिस्सेदारी के साथ सबसे प्रमुख दाल है, इसके बाद तुअर/अरहर 15 से 20 प्रतिशत एवं मूंग लगभग 8-10 प्रतिशत है। अतः कथन 2 सही नहीं है। इसलिए, (b) एवं (c) गलत हैं। सही उत्तर (a) है।

(दालों के उत्पादन एवं खपत पर बहुत सटीक तथ्यों की आवश्यकता)

27. "यह फ़सल उपोष्ण प्रकृति की है। उसके लिए कठोर पाला हानिकारक है। विकास के लिए उसे कम-से-कम 210 पाला-रहित दिवसों और 50-100 सेंटीमीटर वर्षा की आवश्यकता पड़ती है। हल्की सुअपवाहित मृदा जिसमें नमी धारण करने की क्षमता है उसकी खेती के लिए आदर्श रूप से अनुकूल है।" यह फ़सल/ निम्नलिखित में से कौन-सी है?

  1. कपास
  2. जूट
  3. गन्ना
  4. चाय

उत्तर (a) उत्तर ‘कपास’ है। कपास एक उपोष्ण-कटिबंधीय पौधा है, जिसके लिए लंबी ठंड से मुक्त अवधि, धूप एवं मध्यम बारिश (आमतौर पर 60 से 120 सेमी) की आवश्यकता होती है। मिट्टी को आमतौर पर काफी भारी होना चाहिए, हालांकि पोषक तत्वों के स्तर को असाधारण होने की आवश्यकता नहीं है। ये स्थिति उत्तरी एवं दक्षिणी गोलार्ध में मौसमी रूप से शुष्क उष्ण कटिबंध एवं उपोष्णकटिबंध के भीतर मिलती हैं। दुनिया का अधिकांश कपास अमेरिका, उज्बेकिस्तान, चीन एवं भारत में उगाया जाता है।

जूट उगाने के लिए उपयुक्त जलवायु उष्ण एवं आर्द्र है, जो कि मानसून जलवायु में मानसून के दौरान पाई जाती है। 25° सेल्सियस से अधिक तापमान एवं 70-90ः के सापेक्ष आर्द्रता सफल खेती के लिए अनुकूल हैं।

गन्ना एक उष्णकटिबंधीय पौधा है, इसलिए परिपक्वता तक पहुंचने के लिए एक गर्म मौसम की आवश्यकता होती है। 20° से 26° सेल्सियस तक तापमान एवं 150 सेमी औसत वर्षा वाले क्षेत्र इसकी खेती के लिए उपयुक्त हैं। ... गन्ना जलोढ़ मिट्टी एवं हल्की मिट्टी में अच्छी तरह से बढ़ता है।

अपने प्राकृतिक रूप में चाय (उष्णकटिबंधीय एवं उपोष्णकटिबंधीय) उन क्षेत्रों में सबसे अच्छी तरह से बढ़ती है जो एक वर्ष में कम से कम 100 सेंटीमीटर की वर्षा के साथ एक गर्म, आर्द्र जलवायु का आनंद लेते हैं। आदर्श रूप से, यह गहरी, हल्की, अम्लीय एवं अच्छी तरह से सूखी मिट्टी पसंद करती है।

(फसलों एवं उनके लिए उपयुक्त परिस्थतीयों का विस्तृत ज्ञान आवश्यक है)

28. सौर जल पंपो के संदर्भ में, निम्नलिखित कथनों पर विचार कीजिए

  1. सौर ऊर्जा का प्रयोग पृष्ठीय पंपो के चलने के लिए हो सकता है और निमज्जनी (submersible) पंपो के लिए नहीं।
  2. सौर ऊर्जा का प्रयोग अपकेन्द्री पंपो को चलाने के लिए हो सकता है और पिस्टन वालो के लिए नहीं।

उपर्युक्त कथनों में से कौन-सा/से सही है/हैं ?

  1. केवल 1
  2. केवल 2
  3. 1 और 2 दोनो
  4. न तो 1, न ही 2

उत्तर (d) दोनों कथन असत्य हैं। सौर पीवी अभी भी सौर ऊर्जा पर पानी के पंप चलाने का सबसे अच्छा तरीका है। चाहे आप निम्मजनी या सतह के पंपों का उपयोग करें, आप इसका उपयोग दिन में या रात को किसी भी मौसम की स्थिति में कर सकते हैं। सौर-संचालित पंपों को यातोे सकारात्मक विस्थापन पंपों (जैसे डायाफ्राम, पिस्टन, या पेचदार रोटर) या केन्द्रापसारक पंपों के रूप में चित्रित किया जाता है। सकारात्मक विस्थापन पंपों का उपयोग आमतौर पर तब किया जाता है जब टीडीएच अधिक होता है एवं प्रवाह दर (प्रति मिनट गैलन में मापा जाता है) कम होती है। इसके विपरीत, केन्द्रापसारक पंपों का उपयोग आमतौर पर कम टीडीएच एवं उच्च प्रवाह दर के लिए किया जाता है।

सौर पीवी में हाल के प्रौद्योगिकी विकास एवं लागत में गिरावट ने सौर पर आपके मौजूदा निम्मजनी पंप को चलाने के लिए एक आकर्षक प्रस्ताव दिया है। बैटरी की कोई आवश्यकता नहीं है एवं यह एक चर आवृत्ति ड्राइव (VFD) के माध्यम से सीधे सौर पैनल को पनडुब्बी पंप इनपुट से जोड़ सकता है। VFD एक इलेक्ट्रॉनिक उपकरण है जो इन्वर्टर एवं MPPT (अधिकतम पावर प्वाइंट ट्रैकिंग) से लैस है। सौर विकिरण की भिन्नता आवृत्ति मॉड्यूलेशन के माध्यम से देखी जाती है एवं सुबह से शाम तक सूर्य के प्रकाश की एक विस्तृत श्रृंखला के लिए निरंतर टॉर्क पर सबमर्सिबल पंप चलाएगी।

(कठिन प्रश्न)

29.भारत में गन्ने की खेती में वर्तमान प्रवृत्तियों के संदर्भ में, निम्नलिखित कथनों पर विचार कीजिए :

  1. जब ‘बड चिप सैटलिंग्स (bud chip settlings)’ को नर्सरी में उगाकर मुख्य कृषि भूमि में प्रतिरेपित किया जाता है, तब बीज सामग्री में बड़ी बचत होती है।
  2. जब सैट्स का सीधे रोपण किया जाता है, तब एक-कलिका (Single budded) सैट्स का अंकुरण प्रतिशत कई-कलिका (many budded) सैट्स की तुलना में बेहतर होता है।
  3. खराब मौसम की दशा में यदि सैट्स का सीधे रोपण होता है, तब एक-कलिका सैट्स का जीवित बचना बड़े सैट्स की तुलना में बेहतर होता है।
  4. गन्ने की खेती, ऊतक संवर्धन से तैयार की गई सैटलिंग से की जा सकती है।

उपर्युक्त कथनों में से कौन-सा/से सही है/हैं ?

  1. केवल 1 और 2
  2. केवल 3
  3. केवल 1 और 4
  4. केवल 2, 3 और 4

उत्तर (c) कथन 1 एवं 4 सही हैं। गन्ना एक वानस्पतिक रूप से प्रचारित फसल है। उपयोग की जाने वाली बीज सामग्री स्टेम कटिंग ‘सेट’ के रूप में जानी जाती है एवं प्रत्येक में एक या कई कलियां हो सकती हैं। स्वस्थ बीज का उपयोग सफल गन्ने की खेती का एक महत्वपूर्ण पहलू है क्योंकि गन्ने के अधिकांश रोग बीज जनित होते हैं एवं बीज सेट के माध्यम से प्रेषित होते हैं।

रोग मुक्त रोपण सामग्री के तेजी से गुणन के लिए उत्तक संर्वधन तकनीक का विकास गन्ने में पर्याप्त उत्पादन वृद्धि के लिए एक महत्वपूर्ण कदम है। एशिया-प्रशांत क्षेत्र में ऑस्ट्रेलिया, भारत एवं फिलीपींस ने व्यावसायिक बीज उत्पादन के लिए इस तकनीक को पहले ही लागू कर दिया है एवं तेजी से प्रसार एवं उत्पादन वृद्धि से इसके लाभ स्पष्ट हो गए हैं।

शोधकर्ताओं ने पाया कि द्वि-कलिका सैट बेहतर उपज के साथ लगभग 65 से 70ः अंकुरण दे रहे हैं। खराब मौसम के तहत बड़े सैट्स बेहतर अस्तित्व में रहते हैं, लेकिन एक-कलिका सैट भी रासायनिक उपचार से संरक्षित होने पर 70ः अंकुरण देते हैं। अतः कथन 3 गलत है। इसलिए विकल्प (c) सही है।

स्त्रोत - https://www.researchgate.net/publication/322976767_SUGARCANE_PLANTING_TECHNIQUES_A_REVIEW

(गन्ना उगाने की तकनीकों को सूक्ष्म ज्ञान आवश्यक)

30.भारत के संदर्भ में, निम्नलिखित में से किस/किन पद्धति/यों को परितंत्र-अनुकूली कृषि माना जाता है?

  1. फ़सल विधिरूपण
  2. शिंब आधिक्य (Legume intensification)
  3. टेंसियोमीटर का प्रयोग
  4. ऊर्ध्वाधर कृषि (Vertical farming)

नीचे दिए गए कूट का प्रयोग कर सही उत्तर चुनिए :

  1. केवल 1, 2 और 3
  2. केवल 3
  3. केवल 4
  4. 1, 2, 3 और 4

उत्तर (d) दी गई सभी पद्धतियों का उपयोग भारत में पारितंत्र-अनुकूलि कृषि के लिए किया जा सकता है। फसल विविधरूपण मिट्टी में पोषक तत्वों को बनाए रखने में मदद करता है। शिंब आधिक्य फसल उत्पादन गहनता का एक स्थायी तरीका है जिससे किसान संसाधनों को केंद्रित करते हैं एवं भूमि के एक छोटे से टुकड़े पर ऊर्जा खर्च करते हैं जिससे श्रम की मांग कम होती है एवं परिणामस्वरूप कम निवेश से उच्च उत्पादकता होती है। किसानों को सिंचाई की योजना कें अंर्तगत पानी डालने के समय को बताने के लिए टेन्सियोमीटर का उपयोग किया जाता है। उध्र्वाधर कृषि - खड़ी परतो में फसल उगाने का अभ्यास है। इससे भूमि एवं मिट्टी पर कृषि का बोझ कम हो सकता है, एवं पानी की काफी कम खपत होती है। अतः चारों सही हैं।

(कथन 4 पर कुछ तर्क लगाने से उत्तर प्राप्त किया जा सकता है)


31. भारत में प्रत्यक्ष विदेशी निवेश के संदर्भ में, निम्नलिखित में से कोन-सी उसकी प्रमुख विशेषता मानी जाती है?

  1. यह मूलतः किसी सूचीबद्ध कम्पनी में पूँजीगत साधनों द्वारा किया जाने वाला निवेश है।
  2. यह मुख्यतः ऋण सृजित न करने वाला पूँजी प्रवाह है।
  3. यह ऐसा निवेश है जिससे ऋण-समाशोधन अपेक्षित होता है।
  4. यह विदेशी संस्थागत निवेशकों द्वारा सरकारी प्रतिभूतियों में किया जाने वाला निवेश है।

उत्तर (b) प्रत्यक्ष विदेशी निवेश हमेशा स्वागतयोग्य है क्योंकि यह ऋण दायित्वों का निर्माण नहीं करता है। इसलिए (c) एवं (d) गलत हैं। विदेशी प्रत्यक्ष निवेश (एफडीआई) देश में किसी अन्य देश में स्थित इकाई द्वारा किसी व्यवसाय में नियंत्रित स्वामित्व के रूप में किया गया निवेश है। इस प्रकार यह प्रत्यक्ष नियंत्रण की धारणा द्वारा विदेशी पोर्टफोलियो निवेश (एफपीआई) से अलग है। मोटेतौर पर, प्रत्यक्ष विदेशी निवेश में ‘विलय एवं अधिग्रहण, नई सुविधाओं के निर्माण, विदेशी परिचालनों से अर्जित मुनाफे को सुदृढ़ करना, एवं अंतर्राज्यीय ऋण’ शामिल हैं। प्रत्यक्ष विदेशी निवेश, इक्विटी पूंजी, लंबी अवधि की पूंजी एवं अल्पकालिक पूंजी का योग है जैसा कि भुगतान संतुलन में दिखाया गया है। प्रत्यक्ष विदेशी निवेश में आमतौर पर प्रबंधन, संयुक्त-उद्यम, प्रौद्योगिकी एवं विशेषज्ञता के हस्तांतरण में भागीदारी शामिल होती है।

प्रत्यक्ष विदेशी निवेश का भंडार किसी भी अवधि के लिए शुद्ध (यानी, आउटवर्ड एफडीआई एवं इनवर्ड एफडीआई का अंतर) संचयी प्रत्यक्ष विदेशी निवेश है। प्रत्यक्ष निवेश शेयरों की खरीद के माध्यम से निवेश को बाहर करता है (यदि वह खरीदी जाने वाली कंपनी के 10ः से कम निवेशक को नियंत्रित करता है)। इन सभी को गैर-ऋण सृजन पूंजी प्रवाह माना जाता है। इसलिए उत्तर (b)।

(यदि आप एफडीआई एवं एफपीआई के बीच बुनियादी अंतर को जानते थे तो एक किया जा सकने वाला प्रश्न)

32. वर्तमान में भारत के अंतर्राष्ट्रीय व्यापार के संदर्भ में, निम्नलिखित में से कौन-सा/से कथन सही है/हैं ?

  1. भारत के माल का नियर्ज्ञत, माल के आयात से कम है
  2. भारत के लोहे व इस्पात, रसायनों, उर्वरको और मशीनों के आयात में हाल के वर्षों में कमी आयी है
  3. भारत की सेवाओं का निर्यात, सेवाओं के आयात से अधिक है।
  4. भारत को कुल मिलाकर व्यापार/चालू खाते का घाटा हो रहा है।

नीचे दिए गए कूट का प्रयोग कर सही उत्तर चुनिए :

  1. केवल 1 और 2
  2. केवल 2 और 4
  3. केवल 3
  4. केवल 1, 3 और 4

उत्तर (d) 2018-19 में भारत का माल निर्यात $331 बिलियन था। 2018-19 में भारत का माल आयात $514 बिलियन था। अतः कथन 1 सही है। उत्तर (a) या (d) हो सकता है। भारत में सेवाओं का निर्यात 2018-19 में 207 बिलियन अमेरिकी डॉलर था। भारत में सेवाओं का आयात 2018-19 में 176.58 बिलियन अमेरिकी डॉलर था। अतः कथन 3 सही है। सही उत्तर विकल्प (d) है।

दिलचस्प बात यह है कि इस प्रश्न का उत्तर देने के लिए 2020-21 की स्थिति भिन्न हो सकती है, क्योंकि भारत में चालू खाते का अधिशेष रहेगा!

(प्रश्न कठिन प्रतित होता हैं लेकिन यदि आपको यह पता हैं कि 1 सही है, तो यह आसान है)

33. कभी-कभी समाचारो में पाया जाने वाला पद’वेस्ट टेक्सास इंटरमीडिएट (West Texas Intermediate), निम्नलिखित में से किस एक पदार्थ की श्रेणी से संबंधित है ?

  1. कच्चे तेल की
  2. बहुमूल्य-धातु (Buillion) की
  3. दुर्लभ मृदा तत्वों की
  4. यूरेनियम की

उत्तर (a) वेस्ट टेक्सास इंटरमीडिएट कच्चे तेल के मिश्रण के एक ग्रेड एवं/या उस तेल के वर्तमान मूल्य, वायदा मूल्य, या निर्धारित मूल्य का उल्लेख कर सकता है।

(शायद पूरे पेपर में सबसे आसान प्रश्न!)

34. भारतीय अर्थव्यवस्था के संदर्भ में, निम्नलिखित में से कोन-सा/से ग़ैर-वित्तीय ऋण में सम्मिलित है/हैं?

  1. परिवारों का बकाया गृह ऋण
  2. क्रेडिट कार्डों पर बकाया राशि
  3. राजकोष बिल (Treasury bills)

नीचे दिए गए कूट का प्रयोग कर सही उत्तर चुनिए :

  1. केवल 1
  2. केवल 1 और 2
  3. केवल 3
  4. 1, 2 और 3

उत्तर (d) गैर-वित्तीय ऋण का अर्थ है सरकारों, परिवारों एवं गैर-वित्तीय क्षेत्र की कंपनियों द्वारा लिया गया ऋण। वित्तीय कंपनियों में वाणिज्यिक एवं निवेश बैंक, बीमा कंपनियां, वित्त कंपनियां, बंधक ऋणदाता एवं निवेश फर्म शामिल हैं। गैर-वित्तीय कंपनियों के उदाहरण विनिर्माण कंपनियों, सेवा कंपनियों, सरकारी संस्थाओं एवं घरों से हैं। वे गैर-वित्तीय ऋण की श्रेणी में आएंगे करेंगे। इसलिए 1 एवं 3 प्रकृति में स्पष्ट रूप से गैर-वित्तीय हैं - परिवारों के बकाया गृह ऋण एवं राजकोष बिल। अतः एकमात्र संभावित विकल्प (d) बचता है, जो ‘1, 2 एवं 3’ है। हमें 2 के बारे में संदेह है लेकिन ऐसा कोई विकल्प नहीं है (केवल 1 एवं 3)।

(एक अलग तरह का प्रश्न जिसे सामान्य ज्ञान से हल किया जा सकता है।)

35. भारत में, क्यों कुछ परमाणु रिऐक्टर’आई.ए.ई.ए. सुरक्षा उपायों के अधीन रखे जाते हैं जबकि अन्य इस सुरक्षा के अधीन नहीं रखे जाते हैं ?

  1. कुछ यूरेनियम का प्रयोग करते हैं और अन्य थोरियम का
  2. कुछ आयातित यूरेनियम का प्रयोग करते हैं और अन्य घरेलू आपूर्ति का
  3. कुछ विदेशी उद्यमों द्वारा संचालित होते हैं और अन्य घरेलू उद्यमों द्वारा
  4. कुछ सरकारी स्वामित्व वाले होते हैं और अन्य निजी स्वामित्व वाले

उत्तर (b) भारत में, कौन से रिएक्टर आईएईए सुरक्षा के तहत होंगे एवं कौनसे नही यह यूरेनियम के स्रोत पर निर्भर करता है। यदि रिएक्टर को विदेश से खरीदे गए यूरेनियम द्वारा ईंधन दिया जाता है तो यह आईएईए सुरक्षा के तहत आता है अन्यथा नहीं। आप विकल्प (d) एवं (a) को सीधे समाप्त कर सकते थे।

(एक बहुत अच्छी तरह से तैयार किया गया प्रश्न जिसमें कुछ मूलभूत समझ की आवश्यकता थी)

(यदि आप जानते हैं कि सेवाएं ज्त्प्डै के अंतर्गत नहीं आती हैं, तो प्रश्न तेजी से हल किया जा सकता है)

36.व्यापार-संबंधित निवेश उपायो’(TRIMS) के संदर्भ में, निम्नलिखित कथनों में से कौन-सा/से सही है/हैं?

  1. विदेशी निवेशकों द्वारा किए जाने वाले आयात पर परिमाणात्मक निर्बंधन’ प्रषिद्ध होते हैं।
  2. ये वस्तुओं एवं सेवाओं दोनो के व्यापार से संबंधित निवेश उपायों पर लागू होते हैं।
  3. ये विदेशी निवेश के नियमन से संबंधित नहीं है।

नीचे दिए गए कूट का प्रयोग कर सही उत्तर चुनिए :

  1. केवल 1 और 2
  2. केवल 2
  3. केवल 1 और 3
  4. 1, 2 और 3

उत्तर (c) डब्ल्यूटीओ पर एक प्रश्न, वर्तमान परिस्थितियों के कारण प्रासंगिक। ट्रेड-रिलेटेड इन्वेस्टमेंट मेजर्स (TRIMS) का समझौता मानता है कि कुछ निश्चित निवेश उपाय व्यापार को प्रतिबंधित एवं विकृत कर सकते हैं। इसमें कहा गया है कि डब्ल्यूटीओ के सदस्य विदेशी उत्पादों के खिलाफ भेदभाव करने वाले किसी भी उपाय को लागू नहीं कर सकते जो परिमाणात्मक निर्बंधन की ओर जाते है। दोनों बुनियादी डब्ल्यूटीओ सिद्धांत का उल्लंघन करते हैं। इसलिए कथन 1 सही है। उत्तर (a), (c) या (d) हो सकता है। समझौते की कवरेज को अनुच्छेद 1 में परिभाषित किया गया है, जिसमें कहा गया है कि समझौते केवल वस्तुओं में व्यापार से संबंधित निवेश उपायों पर लागू होते हैं। इस प्रकार, TRIMs समझौता सेवाओं पर लागू नहीं होता है। अतः कथन 2 गलत है। अतः तीन विकल्पों को एक शॉट में खारिज किया जाता है, एवं उत्तर (b) है।

37.यदि आर.बी.आई. प्रसारवादी मौद्रिक नीति का अनुसरण करने का निर्णय लेता है, तो वह निम्नलिखित में से क्या नहीं करेगा ?

  1. वैधानिक तरलता अनुपात को घटाकर उसे अनुकूलित करना।
  2. सीमान्त स्थायी सुविधा दर को बढ़ाना
  3. बैंक दर को घटाना तथा रेपो दर को भी घटना

नीचे दिए गए कूट का प्रयोग कर सही उत्तर चुनिए :

  1. केवल 1 और 2
  2. केवल 2
  3. केवल 1 और 3
  4. 1, 2 और 3

उत्तर (b) एक सीधा एवं आसान प्रश्न लेकिन प्रश्न में दिए गए ‘नही’ पर ध्यान दिजीए!

प्रसारवादी मौद्रिक नीति सामान्य से अधिक तेजी से पैसे की आपूर्ति का प्रसार करके या अल्पकालिक ब्याज दरों को कम करके काम करती है। यह केंद्रीय बैंकों द्वारा अधिनियमित किया जाता है एवं खुले बाजार संचालन, आरक्षित आवश्यकताओं एवं ब्याज दरों को निर्धारित करने के माध्यम से आता है। सीमांत स्थायी सुविधा (MSF) दर उस दर को संदर्भित करती है, जिस पर सरकारी प्रतिभूतियों के विरुद्ध अनुसूचित बैंक रातोंरात भारिबै से धनराशि उधार ले सकते हैं। MSF अनुसूचित वाणिज्यिक बैंकों के लिए एक बहुत ही अल्पकालिक उधार योजना है। इसलिए यदि भारिबै एक विस्तारवादी मौद्रिक नीति अपनाने का फैसला करता है तो यह MSF में वृद्धि नहीं करेगा बल्कि इसे कम करेगा। तो 2 गलत है, इसलिए एक शॉट में तीन विकल्पों को खारिज किया जाता है! इसलिए (b) सही उत्तर है।

(यदि आप विषय को आमतौर पर समझते हैं, तो प्रश्न बहुत तेजी से हल हो सकता है)

38. 1991 के आर्थिक उदारीकरण के बाद की भारतीय अर्थव्यवस्था के संबंध में, निम्नलिखित कथनों पर विचार कीजिए :

  1. शहरी क्षेत्रों में श्रमिक उत्पादकता (2004-05 की कीमतों पर प्रति श्रमिक रू.) में वृद्धि हुई जबकि ग्रामीण क्षेत्रों में इसमें कमी हुई।
  2. कार्यबल में ग्रामीण क्षेत्रों की प्रतिशत हिस्सोदारी में सतत वृद्धि हुई।
  3. ग्रामीण क्षेत्रो में, गैर-कृषि अर्थव्यवस्था में वृद्धि हुई।
  4. ग्रामीण रोज़गार की वृद्धि दर में कमी आई।

उपर्युक्त कथनों में से कौन-सा/से सही है/हैं ?

  1. केवल 1 और 2
  2. केवल 3 और 4
  3. केवल 3
  4. केवल 1, 2 और 4

उत्तर (b) 1991 के बाद कृषि क्षेत्र में श्रम बल की प्रतिशत हिस्सेदारी में गिरावट एवं सेवा क्षेत्र में वृद्धि के साथ व्यावसायिक संरचना में बदलाव हुआ है। अतः कथन 2 गलत है। उत्तर (b) या (c) हो सकता है। 2004-05 के बाद, ग्रामीण क्षेत्रों में उत्पादन में उच्च वृद्धि के बावजूद रोजगार में नकारात्मक वृद्धि देखी गई है। अतः कथन 4 सही है।


(भारत में कार्यबल संरचनाओं की गहन जानकारी की आवश्यकता है)

39. निम्नलिखित कथनों पर विचार कीजिए :

  1. कृषि क्षेत्र को अल्पकालीन साख परिदान करने के संदर्भ में, जिला केन्द्रीय सहकारी बैंक (DCCBs)’‘अनुसूचित वाणिज्यिक बैंको’ एवं ’क्षेत्रीय ग्रामीण बैंको’ की तुलना में अधिक ऋण देते हैं।
  2. डी.सी.सी.बी. (DCCBs) का एक सबसे प्रमुख कार्य 'प्राथमिक कृषि साख समितियों’को निधि उपलब्ध कराना है।

उपर्युक्त में से कौन-सा/से कथन सही है/हैं?

  1. केवल 1
  2. ये केवल 2
  3. 1 और 2 दोनो
  4. न तो 1, न ही 2

उत्तर (b) ‘अनुसूचित वाणिज्यिक बैंकों’ ने कृषि एवं संबद्ध ऋण में प्रमुख हिस्सेदारी (78 - 80 प्रतिशत) का योगदान दिया। सहकारी संस्थाएं भी कृषि ऋण का विस्तार करने में महत्वपूर्ण भूमिका निभाती हैं एवं सभी सहकारी बैंकों/संस्थानों (यानी StCBs] DCCBs एवं PACS  को एक साथ रखा गया) की हिस्सेदारी 15-16 प्रतिशत है। आरआरबी ने कृषि ऋण के शेष 5 प्रतिशत का योगदान दिया। इसलिए कथन 1 सही नहीं है।

जिला केंद्रीय सहकारी बैंकों के प्रमुख कार्य - सदस्य समाजों की क्रेडिट आवश्यकताओं को पूरा करना, बैंकिंग व्यवसाय करना, प्राथमिक कृषि साख समितियों (PACS)  के अधिशेष धन को डायवर्ट को कमी का सामना करने वाले संस्थानों की ओर डायवर्ट करके संतुलन केंद्र के रूप में कार्य करना, गैर-ऋण गतिविधियों को करना। अतः कथन 2 सही है।

स्त्रोत - https://www.rbi.org.in/Scripts/PublicationReportDetails.aspx?UrlPage=&ID=942

(थोड़ा कठिन प्रश्न)

40. भारत में, किसी व्यक्ति के साइबर बीमा कराने पर, निधि की हानि की भरपाई एवं अन्य लाभों के अतिरिक्त, सामान्यतः निम्नलिखित में से कौन-कौन से लाभ दिए जाते हैं ?

  1. यदि कोई मैलवेयर कम्प्यूटर तक उसकी पहुँच बाधित कर देता है, तो कम्प्यूटर प्रणाली को पुनः प्रचालित करने में लगने वाली लागत।
  2. यदि यह प्रमाणित हो जाता है कि किसी शरारती तत्व द्वारा जान-बूझकर कम्प्यूटर को नुकसान पहुँचाया गया है, तो नए कम्प्यूटर की लागत
  3. यदि साइबर लागत्-ग्रहण होता है तो इस हानि को न्यूनतम करने के लिए विशेषज्ञ परामर्शदाता की सेवाएँ लेने पर लगने वाली लागत
  4. यदि कोई तीसरा पख मुक़दमा दायर करता है तो न्यायालय में बचाव करने में लगने वाली लागत।

नीचे दिए गए कूट का प्रयोग कर सही उत्तर चुनिए :

  1. केवल 1, 2 और 4
  2. केवल 1, 3, और 4
  3. केवल 2 और 3
  4. 1, 2, 3 और 4

उत्तर (d) प्रथम-पक्ष का कवरेज पॉलिसीधारक के अपने डेटा, डेटा ब्रीच के कारण खोई हुई आय या साइबर-हमले या रैंसमवेयर हमले के लिए बिमीत करता है। प्रथम-पक्ष कवरेज डेटा भंग या सुरक्षा विफलता का जवाब देने के लिए बीमित व्यवसाय की प्रत्यक्ष लागत पर लागू होता है।

थर्ड-पार्टी कवरेज डेटा ब्रीच, साइबर-हमले या रैंसमवेयर हमले के परिणामस्वरूप तीसरे पक्ष (ग्राहकों एवं सरकारी संस्थाओं सहित) के प्रति दायित्व के विरुद्ध है। तृतीय-पक्ष कवरेज तब लागू होता है जब व्यक्ति बीमित व्यवसाय के खिलाफ मुकदमा दायर करते हैं या दावा करते हैं, या सरकारें या नियामक बीमाधारक व्यवसाय से जानकारी की मांग करते हैं।

प्रथम-पक्ष कवरेज - व्यावसायिक रुकावट एवं खोई हुई आय - जब एक साइबर घटना किसी इकाई के नेटवर्क को नुकसान पहुंचाती है या व्यवसाय की निरंतरता एवं संचालन को बाधित करने वाले डेटा के नुकसान का कारण बनती है, तो खोई हुई आय एवं बढ़े हुए परिचालन खर्चों को कवर करता है।

कंप्यूटर डेटा हानि एवं नेटवर्क बहाली - एक इकाई के कंप्यूटर सिस्टम को शारीरिक क्षति एवं क्षतिग्रस्त या चोरी हुए डेटा, हार्डवेयर एवं सॉफ्टवेयर को पुनः प्राप्त करने एवं पुनर्स्थापित करने की लागत को कवर करता है।

फोरेंसिक जांच सेवाएँ - साइबर-घटना का आकलन करने एवं रोकने के लिए तकनीकी, कानूनी या अन्य विशेषज्ञ सेवाओं के लिए लागत एवं खर्च को कवर करती है।

अधिसूचना लागत - ब्रीड रीमेडिएशन को नियंत्रित करने वाले कानूनों एवं विनियमों के बारे में कानूनी सलाह शामिल है, जिसमें सभी पीड़ितों को सूचित करने के लिए लागतें शामिल हैं, जिसमें ग्राहक एवं कर्मचारी, साइबर घटना एवं संभावित पहचान या क्रेडिट कार्ड चोरी शामिल हैं।

संकट प्रबंधन एवं जनसंपर्क - ग्राहक सहयोग, कॉल सेंटर, क्रेडिट मॉनिटरिंग एवं अन्य खर्चों को साइबर ब्रीच घटना के पीड़ितों एवं व्यावसायिक संस्थाओं की प्रतिक्रिया के साथ-साथ जनसंपर्क क्षति से बचाने के लिए परामर्श शुल्क शामिल करता है।

एक्सटॉर्शन एवं रैनसमवेयर - साइबर-हमलों एवं हमलों के खतरों की जांच के लिए लागत के साथ-साथ एक्सटॉर्शनिस्ट को भुगतान के लिए लागत को कवर करता है।

इलेक्ट्रॉनिक चोरी - एक व्यावसायिक इकाई के पैसे को कवर करता है जो नेटवर्क के उल्लंघन एवं इलेक्ट्रॉनिक फंड के धोखाधड़ी हस्तांतरण के परिणामस्वरूप चोरी हो जाता है।

तृतीय-पक्ष कवरेज -

मुकदमेबाजी - मुकदमों की रक्षा के लिए लागतों को शामिल करता है, जिसमें कक्षा की कार्रवाई शामिल है, जिसमें एक कंप्यूटर हमले, वायरस के प्रसार को रोकने या कम करने, गोपनीय जानकारी के अनधिकृत उपयोग/गोपनीय जानकारी के उपयोग या सिस्टम सुरक्षा की विफलता को रोकने में विफलता शामिल है।

सरकारी एवं नियामक - सरकारी जाँच या कार्यवाही के साथ-साथ साइबर घटना से संबंधित जुर्माना एवं जुर्माने के भुगतान का जवाब देने या बचाव करने के लिए लागत को कवर करता है।

क्रेडिट एवं धोखाधड़ी की निगरानी - ग्राहक क्रेडिट निगरानी, पहचान की चोरी से सुरक्षा सेवाओं एवं साइबर-घटना के बाद धोखाधड़ी की निगरानी के लिए लागत को कवर करता है।

मल्टीमीडिया - ऑनलाइन मानहानि, कॉपीराइट एवं ट्रेडमार्क उल्लंघन के दावों से संबंधित लागतों को शामिल करता है।

कवरेज के बाहर क्या है? अन्य प्रकार की बीमा पॉलिसियों के समान, साइबर इंश्योरेंस पॉलिसी अक्सर कवरेज से कुछ नुकसानों का बीमा करती हैं। विशिष्ट बहिष्करण में युद्ध से उत्पन्न होने वाले दावे, अनुबंध का उल्लंघन, व्यापार रहस्य की चोरी, अनुचित व्यापार व्यवहार, एवं रोजगार प्रथाएँ शामिल हैं। साइबर बीमा पॉलिसियां आमतौर पर बीमाधारक के इरादतन, जानबूझकर, दुर्भावनापूर्ण, धोखाधड़ी, बेईमानी या आपराधिक कृत्यों या चूक के लिए कवरेज का बीमा करती हैं।

स्रोत - https://www.sgrlaw.com/client-alerts/cyber-insurance-frequently-asked-questions/

(एक बहुत कठिन प्रश्न)

41. कृषि में फर्टीगेशन (fertigation) के क्या लाभ हैं ?

  1. सिंचाई जल की क्षारीयता का नियंत्रण संभव है।
  2. रॉक फॉस्फेट और सभी अन्य फॉस्फेटिक उर्वरकों का सफलता के साथ अनुप्रयोग संभव है।
  3. पौधों के लिए पोषक बढ़ी हुई मात्रा में सुलभ किए जा सकते हैं।
  4. रासायनिक पौषकों के निक्षालन में कमी संभव है।

नीचे दिए गए कूट का प्रयोग कर सही उत्तर चुनिए :

  1. केवल 1, 2 और 3
  2. केवल 1, 2, और 4
  3. केवल 1, 3 और 4
  4. केवल 2, 3 और 4

उत्तर (c) कथन 2 गलत है, लेकिन 1, 3 एवं 4 सही हैं।

‘फर्टीगेशन’ तकनीक में उर्वरकों को सिंचाई प्रणाली में विघटित करके फसलों को आपूर्ति की जाती है। प्रसारण एवं बैंड निषेचन पर इसके कुछ विशिष्ट फायदे हैं -

  1. पोषक तत्वों की लगातार आपूर्ति मिट्टी में पोषक तत्वों की सांद्रता के उतार-चढ़ाव को कम करती है।
  2. पौधों द्वारा पोषक अवशोषण में वृद्धि।
  3. पोषक तत्वों की सटीक नियुक्ति - फसल की पोषण संबंधी आवश्यकताओं के अनुसार पोषक तत्वों का कुशल उपयोग एवं सटीक अनुप्रयोग है। जहां पानी जाता है पोषक तत्व भी चला जाता है।
  4. उर्वरकों को सिंचित मिट्टी की मात्रा में लगाया जाता है।
  5. पोषक तत्वों को मिट्टी पर तब लागू किया जा सकता है जब मिट्टी या फसल की स्थिति अन्यथा पारंपरिक उपकरणों के साथ क्षेत्र में प्रवेश पर रोक लगा देगी।
  6. ‘सूक्ष्म खुराक’ की क्षमता, पौधों को पर्याप्त मात्रा में खिलाना ताकि पोषक तत्वों को अवशोषित किया जा सके एवं अगली बार बारिश होने पर तूफान के पानी से धोया नहीं जा सके।
  7. उर्वरक, रसायन, एवं पानी की कमी।
  8. पानी की आपूर्ति में रसायनों के लीचिंग को कम करना।
  9. पौधे की जड़ के द्रव्यमान में पानी को फंसाने एवं धारण करने की क्षमता के कारण पानी की खपत कम होना।
  10. पोषक तत्वों के अनुप्रयोग को सटीक समय एवं आवश्यक दर पर नियंत्रित किया जा सकता है।
  11. दूषित मिट्टी के माध्यम से मृदा जनित रोगों से जूझने वाली जड़ों का न्यूनतम जोखिम।
  12. मिट्टी के कटाव के मुद्दों को कम करना क्योंकि पानी ड्रिप प्रणाली के माध्यम से पोषक तत्वों को पंप किया जाता है।
  13. प्रजनन क्षमता को नियोजित करने के लिए प्रयुक्त विधियों के माध्यम से लीचिंग को अक्सर कम किया जाता है।
  14. पानी की क्षारीयता को फॉस्फोरिक, सल्फ्यूरिक, नाइट्रिक एवं साइट्रिक जैसे एसिड का उपयोग करके प्रजनन के माध्यम से नियंत्रित किया जा सकता है।

अब, पानी एवं साइट्रेट अघुलनशील फॉस्फेट उर्वरकों पर ध्यान दें - इनमें फॉस्फोरस होता है, जो पानी के साथ-साथ साइट्रिक एसिड में भी अघुलनशील होता है। वे अत्यधिक एसिड युक्त मिट्टी या जैविक मिट्टी में उपयुक्त हैं, एवं हरी खाद वाले खेतों में दिए जाते हैं। क्रिया में, फास्फोरस बहुत धीरे-धीरे रोगाणुओं द्वारा छोड़ा जाता है एवं लंबे समय तक मिट्टी में रहता है। अतः फर्टीगेशन का राॅक फाॅस्फेट और सभी अन्य फाॅस्फेटिक उर्वरकों का सफलता के अनुप्रयोग संभव नही है।

(यह एक बहुत ही कठिन प्रश्न है, एवं पूरी तरह से तथ्यात्मक है।)

42. निम्नलिखित खनिजों पर विचार कीजिए :

  1. बेंटोनाइट
  2. क्रोमाइट
  3. कायनाइट
  4. सिलीमेनाइट

भारत में, उपर्युक्त में से कौन-सा/से अधिकारिक रूप से नामित प्रमुख खनिज (major minerals) है/हैं ?

  1. केवल 1 और 2
  2. केवल 4
  3. केवल 1 और 3
  4. केवल 2, 3 और 4

उत्तर (d) प्रमुख खनिज वे हैं, जो खान एवं खनिज (विकास एवं विनियमन) अधिनियम, 1957 (एमएमडीआर अधिनियम 1957) में संलग्न प्रथम अनुसूची में निर्दिष्ट हैं एवं आम प्रमुख खनिज हैं लिग्नाइट, कोयला, यूरेनियम, लौह अयस्क, सोना आदि। MMDR एक्ट 1957 में क्रोमाइट, किनाइट, सिलिमेनाइट नाम के खनिज भी शामिल हैं लेकिन बेंटोनाइट नहीं।

क्रोमाइट एक खनिज है जो एक लौह क्रोमियम ऑक्साइड है। इसमें FeCr2O4 का रासायनिक सूत्र है।

केनाइट एक नीला एल्युमिनोसिलिकेट खनिज है, जो एल्यूमीनियम-समृद्ध मेटामॉर्फिक पेगमाटाइट्स एवं/या तलछटी चट्टान में पाया जाता है।

Sillimanite रासायनिक सूत्र Al2SiO5 के साथ एक aluminosilicate खनिज है।

इसलिए दिए गए चार नामों में से केवल ‘बेंटोनाइट’ खनिजों की प्रमुख खनिजों सूची में शामिल नहीं है, एवं उत्तर विकल्प (d) है। परीक्षक वास्तव में आपसे खनिजों की पूरी सूची को याद रखने की उम्मीद कर रहा है!

(एक बहुत कठिन प्रश्न, पूरी तरह से तथ्यात्मक, तार्किक अनुमान की कोई गुंजाइश नहीं है)

43. महासागर औसत तापमान (Ocean Mean Temperature/ OMT) के संदर्भ में, निम्नलिखित में से कौन-सा/से कथन सही है/हैं ?

  1. OMT को 26oC समताप रेखा की गहराई तक मापा जाता है जो जनवरी-मार्च में हिन्द महासागर के दक्षिण-पश्चिम में 129 मीटर पर होती है।
  2. OMT को जनवरी-मार्च में एकत्रित किया जाता है उसे यह निर्धारित करने के लिए प्रयोग किया जा सकता है कि मानसून में वर्षा की मात्रा एक निश्चित दीर्घकालीन औसत वर्ष से कम होगी या अधिक।

नीचे दिए गए कूट का प्रयोग कर सही उत्तर चुनिए :

  1. केवल 1
  2. केवल 2
  3. 1 और 2 दोनो
  4. न तो 1, न ही 2

उत्तर (उत्तर (b) भारतीय उष्णकटिबंधीय मौसम विज्ञान संस्थान, पुणे के वैज्ञानिकों ने पाया कि वर्तमान में उपयोग किए जाने वाले समुद्री सतह तापमान (एसएसटी) विधि की तुलना में, मानसून के मौसम में भारत को मिलने वाली वर्षा की भविष्यवाणी करने के लिए महासागर औसत तापमान (ओएमटी) एक बेहतर क्षमता है। एसएसटी की तुलना में, जिसमें 60% सफलता दर है, ओएमटी में 80% सफलता दर है।

ओएमटी का विश्लेषण जनवरी से मार्च की अवधि के दौरान महासागर की ऊष्मीय ऊर्जा को मापने के लिए किया जाता है ताकि मानसून के बारे में जानकारी यह हो सके कि कम या ज्यादा अप्रैल की शुरुआत में उपलब्ध कराया जा सकता है, दो महीने पहले दक्षिण-पश्चिम मानसून में स्थापित हो सकता है। ओएमटी को दक्षिण-पश्चिमी हिंद महासागर क्षेत्र (50° पूर्व - 70° पूर्व एवं 10° उत्तर - 0° उत्तर आयताकार बॉक्स द्वारा सीमांकित) में मापा जाता है। महासागर का औसत तापमान 26° सेल्सियस आइसोथर्म की गहराई तक मापा जाता है। 26° सेल्सियस आइसोथर्म 50-100 मीटर से भिन्न गहराई में देखा जाता है। इसलिए कथन 1 आंशिक रूप से सही है, पूरी तरह से सही नहीं है। इसलिए, विकल्प (b) सही है।

(पर्यावरण एवं पारिस्थितिकी आधारित विज्ञान से एक कठिन प्रश्न।)

44.भारत में रासायनिक उर्वरकों के संदर्भ में, निम्नलिखित कथनों पर विचार कीजिए :

डिजिटल हस्ताक्षर

  1. वर्तमान में रासायनिक उर्वरकों का खुदरा मूल्य बाज़ार-संचालित है और यह सरकार द्वारा नियंत्रित नहीं है।
  2. अमोनिया जो यूरिया बनाने में काम आता है, वह प्राकृतिक गैस से उत्पन्न होता है।
  3. सल्फर, जो फॉस्फोरिक अम्ल उर्वरक के लिए कच्चा माल है, वह तेल शोधन कारखानों का उपोत्पाद है।

उपर्युक्त में से कौन-सा/से कथन सही है/हैं?

  1. केवल 1
  2. केवल 2 और 3
  3. केवल 2
  4. 1, 2 और 3

उत्तर (बी) अप्रैल 2010 में गैर-यूरिया उर्वरकों की खुदरा कीमतें पूरी तरह से नियंत्रण मुक्त की गईं है। यूरिया, जो सबसे प्रमुख उर्वरक, को सरकारी नियंत्रण में रखा गया है। अतः कथन 1 गलत है एवं उत्तर (b) या (c) होगा।

एक विशिष्ट आधुनिक अमोनिया उत्पादक संयंत्र पहले प्राकृतिक गैस (यानी, मीथेन) या एलपीजी (प्रोपेन एवं ब्यूटेन जैसी तरलीकृत पेट्रोलियम गैसों) या पेट्रोलियम नेफ्था को गैसीय हाइड्रोजन में परिवर्तित करता है। हाइड्रोजन को तब नाइट्रोजन के साथ मिलाकर हैबर-बॉश प्रक्रिया के माध्यम से अमोनिया का उत्पादन किया जाता है।

सल्फर तेल शोधन एवं गैस प्रसंस्करण का एक प्रमुख उत्पाद है। अतः उत्तर विकल्प (b) है।

(चूंकि यूरिया का मूल्य सरकार नियंत्रित है। कथन 1 गलत है, इसलिए विकल्प (a) एवं (d) से इनकार किया गया है। आपको इसके बाद केवल कथन 3 के बारे में सुनिश्चित होना चाहिए। हाल के दिनों में उर्वरक सब्सिडी आदि पर बहुत बहस के कारण यह प्रश्न पूछा गया था।)

45. भारत के 'मरू राष्ट्रीय उद्यान' के संदर्भ में, निम्नलिखित में से कौन-से कथन सही हैं ?

  1. यह दो जिलों में विस्तृत है।
  2. उद्यान के अन्दर कोई मानव वास स्थल नहीं है।
  3. यह ‘ग्रेट इंडियन बस्टर्ड’के प्राकृतिक आवासों में से एक है।

नीचे दिए गए कूट का प्रयोग कर सही उत्तर चुनिए :

  1. केवल 1 और 2
  2. केवल 2 और 3
  3. केवल 1 और 3
  4. 1, 2 और 3

उत्तर (c) मरू राष्ट्रीय उद्यान (DNP) राजस्थान, भारत, राजस्थान में जैसलमेर एवं बाड़मेर जिलों के पास स्थित है। यह भारत के सबसे बड़े राष्ट्रीय उद्यानों में से एक है, एवं एक संरक्षित अभयारण्य भी है। लुप्तप्राय ग्रेट इंडियन बस्टर्ड एक शानदार पक्षी है जो अपेक्षाकृत अधिक संख्या में पाया जाता है। यह विभिन्न मौसमों में स्थानीय रूप से पलायन करता है। DNP में 73 गाँव हैं, जहाँ हजारों लोग रहते हैं, इसलिए कथन 2 गलत है। राजस्थान में अन्य एनपी हैं सरिस्का एनपी (1982), रणथंभौर एनपी (1980), मुकुंदरा हिल्स (दर्रा) एनपी (2006), एवं केवलादेव घाना एनपी (1981)।

(यह एक सीधा एवं आसान प्रश्न था। भारत-चीन एलएसी झड़प के बारे में विशाल कवरेज को देखते हुए, आपसे यह सुनिश्चित करने की उम्मीद की गई थी।)


46. सियाचिन हिमनद कहाँ स्थित है ?

  1. अक्साई चिन के पूर्व में
  2. लेह के पूर्व में
  3. गिलगिट के उत्तर में
  4. नुब्रा घाटी के उत्तर में

उत्तर (d) सियाचिन हिमनद हिमालय में पूर्वी काराकोरम रेंज में स्थित है, जो लगभग 35-421226° N 77-109540° E है। जैसा कि मानचित्र में दिखाया गया है कि यह नुब्रा घाटी के उत्तर में है। यह स्पष्ट है कि यह अक्साई चीन का ‘पूर्व’ में नहीं हो सकता है (जो चीन में है), एवं न ही यह गिलगित के ‘उत्तर’ में हो सकता है।


(यह एक सीधा एवं आसान प्रश्न था। भारत-चीन एलएसी झड़प के बारे में विशाल कवरेज को देखते हुए, आपसे यह सुनिश्चित करने की उम्मीद की गई थी।)

47.भारत के इतिहास के संदर्भ में, निम्नलिखित युग्मों पर विचार कीजिए :

प्रसिद्ध स्थल वर्तमान राज्य

  1. भीलसा - मध्य प्रदेश
  2. द्वारसमुद्र - महाराष्ट्र
  3. गिरी नगर - गुजरात
  4. स्थानेश्वर - उत्तर प्रदेश

उपर्युक्त में से कौन-से युग्म सही सुमेलित है ?

  1. केवल 1 और 3
  2. केवल 1 और 4
  3. केवल 2 और 3
  4. केवल 2 और 4

उत्तर (a) जोड़े 2 एवं 4 गलत रूप से मेल खाते हैं।

भीलसा परमार साम्राज्य के दौरान एक शहर था जिस पर तेरहवीं शताब्दी में अलाउद्दीन खिलजी ने हमला किया था। वर्तमान में इसे विदिशा के नाम से जाना जाता है एवं यह मप्र का एक जिला है। सही उत्तर (a) या (b हो सकता है ।

भगवान शिव को समर्पित प्राचीन स्थनेश्वर महादेव मंदिर, भारत के हरियाणा के कुरुक्षेत्र जिले के एक पवित्र शहर थानेसर में स्थित है। अतः सही उत्तर (a) है।

गिरिनगर (एक पहाड़ की चोटी पर स्थित शहर) जिसे गिरनार भी कहा जाता है, एवं जूनागढ़, गुजरात में है। यह भगवान दत्तात्रेय से जुड़ी कहानियों वाला एक प्राचीन शहर है।

हलीबिदु (पहले द्वारसमुद्र) 12 वीं शताब्दी में कर्नाटक के होयसल साम्राज्य की राजधानी थी।

(प्राचीन एवं मध्यकालीन इतिहास से एक मानक प्रश्न, स्थानों के ज्ञान की आवश्यकता)

(प्राचीन एवं मध्यकालीन इतिहास से एक मानक प्रश्न, स्थानों के ज्ञान की आवश्यकता)

48.निम्नलिखित कथनों पर विचार कीजिए :

  1. केन्द्रीय भूमि जल प्राधिकरण (CGWA) ने भारत के 36% जिलों को "अतिशोषित" (overexploited) अथवा ‘‘संकटपूर्ण" (critical) वर्गीकृत किया हुआ है
  2. CGWA का निर्माण ‘पर्यावरण (संरक्षण) अधिनियम’ के अंतर्गत हुआ।
  3. विश्व में भूजल सिंचाई के अन्तर्गत सबसे अधिक क्षेत्र भारत में है।

उपर्युक्त कथनों में से कौन-सा/से सही है/हैं?

  1. केवल 1
  2. केवल 2 और 3
  3. केवल 2
  4. केवल 1 और 3

उत्तर (b) देश में भूजल विकास एवं प्रबंधन के नियमन एवं नियंत्रण के लिए पर्यावरण (संरक्षण) अधिनियम, 1986 के तहत केंद्रीय भूजल प्राधिकरण (CGWA) का गठन किया गया था। अतः उत्तर (b) या (c) हो सकता है। 39 मिलियन हेक्टेयर (कुल भूजल सिंचाई का 67%) के साथ, भारत में दुनिया का सबसे बड़ा भूजल सिंचाई क्षेत्र है (चीन 19 माह के साथ दूसरे स्थान पर है, 17 एमए के साथ संयुक्त राज्य अमेरिका तीसरे स्थान पर है)। उत्तर (b) है।

दिशानिर्देशों को परिभाषित करने के लिए वार्षिक पुनःपूर्ति योग्य भूजल संसाधन उपलब्धता एक महत्वपूर्ण भूमिका निभाती है। वार्षिक पुनःपूर्ति योग्य भूजल संसाधन का अनुमान 431 बिलियन क्यूबिक मीटर (bcm) लगाया गया है। शुद्ध भूजल उपलब्धता 396 सेमी है एवं देश के भूजल विकास का समग्र चरण 61% है। ‘अति शोषित’ एवं ‘महत्वपूर्ण’ इकाइयाँ 36% नहीं हैं, लेकिन उससे कम हैं। यह याद रखना एक कठिन तथ्य है।

(यह स्पष्ट है कि 3 निश्चित रूप से सही है, एवं यही कारण है कि पानी के बढ़ते तनाव का कारण है)

49. निम्नलिखित कथनों पर विचार कीजिए :

  1. जेट प्रवाह केवल उत्तरी गोलार्ध में होते है।
  2. केवल कुछ चक्रवात ही केंद्र में वाताक्षि उत्पन्न करते हैं।
  3. चक्रवात की वाताक्षि के अन्दर का तापमान आसपास के तापमान से लगभग 10OC कम होता है।

उपर्युक्त कथन में से कौन-सा/से सही है/हैं?

  1. केवल 1
  2. केवल 2 और 3
  3. केवल 2
  4. केवल 1 और 3

उत्तर (ए) जेट स्ट्रीम उत्तरी एवं दक्षिणी गोलार्ध दोनों में होती हैं। अतः कथन 1 गलत है, एवं उत्तर (b) या (c) हो सकता है। अतः कथन 2 निश्चित रूप से सही है। उष्णकटिबंधीय चक्रवातों के केंद्र वाताक्षि ज्यादातर एक शांत क्षेत्र होता है। एक चक्रवात के केंद्र में वाताक्षि लगभग एक गोलाकार क्षेत्र है, आमतौर पर इसका व्यास 30-65 किलोमीटर होता है। यह वाताक्षि गोलक से घिरा हुआ होता है, जो कि सबसे भयंकर मौसम एवं उच्चतम हवाएं होती है, जहां तेज आंधी का एक चक्र होता है। चक्रवात का सबसे कम बैरोमीटर वाताक्षि में होता है एवं यह तूफान के बाहर के दबाव से 15 प्रतिशत कम हो सकता है।

जैसा कि हवा कम हो जाती है, यह थोड़ा सिकुड़ता हैं एवं गर्म हो जाता है, ताकि उष्णकटिबंधीय चक्रवात के केंद्र में तापमान कुछ 5.5° से (10° एफ) तूफान के अन्य क्षेत्रों की तुलना में अधिक हो। क्योंकि संक्षेपण होने से पहले गर्म हवा अधिक नमी धारण कर सकती है, चक्रवात की वाताक्षि आमतौर पर बादलों से मुक्त होती हैं। अतः कथन 3 गलत है। एवं हमारा उत्तर है (c)

(एक मानक प्रश्न जिसमें कुछ मुश्किल कथन दिए गए है। थोड़ा दिमाग लगाने से मदद मिल सकती थी)

50.निम्नलिखित बाघ आरक्षित क्षेत्रों में ‘‘क्रांतिक बाघ आवास (Critical Tiger Habitat)’’ के अंतर्गत सबसे बड़ा क्षेत्र किसके पास है।

  1. कॉबेट
  2. रणथम्बौर
  3. नागार्जुनसागर-श्रीसैलम
  4. सुंदरबन

उत्तर (c) इसका उत्तर नागार्जुनसागर-श्रीसैलम (आंध्रा प्रदेश) है।

विकल्प (a) - उत्तराखंड में कॉर्बेट नेशनल पार्क में 821 वर्ग किमी का क्रिटिकल टाइगर हैबिटेट है। बफर क्षेत्र 466 वर्ग किमी है।

विकल्प (b) - राजस्थान में रणथंभौर में 1113 वर्ग किमी का क्रिटिकल टाइगर हैबिटेट है। बफर क्षेत्र 297 वर्ग किमी है। विकल्प

विकल्प (c) - एपी में नागार्जुनसागर श्रीशैलम में 2597 वर्ग किमी का क्रिटिकल टाइगर हैबिटेट है। बफर क्षेत्र 700 वर्ग किमी है।

विकल्प (d) - पश्चिम बंगाल के सुंदरबन में 1699 वर्ग किमी का क्रिटिकल टाइगर हैबिटेट है। बफर क्षेत्र 885 वर्ग किमी है। दिए गए विकल्पों में से, (c) में सबसे बड़ा CTH है।

क्रांतिक बाघ आवास (CTH) , वन्य जीवन संरक्षण अधिनियम (WLPA) 1972 के तहत वैज्ञानिक प्रमाणों के आधार पर पहचाने जाने वाले बाघ अभ्यारण्यों के मुख्य क्षेत्र हैं, ‘बाघ संरक्षण के उद्देश्य से अनुसूचित जनजातियों या ऐसे अन्य वनवासियों के अधिकारों को प्रभावित किए बिना ऐसे क्षेत्रों को संरक्षित रखा जाना आवश्यक है’। CTH की अधिसूचना राज्य सरकार द्वारा की जाती है। महत्वपूर्ण ‘वन्यजीवों के आवास (सीडब्ल्यूएलएच) केवल वन अधिकार अधिनियम, 2006 में परिभाषित किए गए हैं।

नवीनतम रिपोर्ट के पृष्ठ 44-45-46 पर पूरी सूची देखें -

https://ntca.gov.in/assets/uploads/Reports/Annual_Reports/Annual_Report_English_2018-19.pdf


51.निम्नलिखित कथनों पर विचार कीजिए :

  1. ‘आधार’मेटाडेटा को तीन महीने से अधिक संग्रहित नहीं रखा जा सकता है।
  2. राज्य निजी निगमों (Corporations) से ‘आधार’ डेटा को साझा करने के लिए कोई अनुबंध नहीं कर सकता।
  3. ‘आधार’बीमा उत्पादों को प्राप्त कने के लिए अनिवार्य है।
  4. ‘आधार’भारत की संचित निधि से हितलाभ प्राप्त करने के लिए अनिवार्य है।

उपर्युक्त कथनों में से कौन-सा/से सही है/हैं?

  1. केवल 1 और 4
  2. केवल 2 और 4
  3. केवल 3
  4. केवल 1, 2 और 3

उत्तर (b) केवल 2 एवं 4 सत्य हैं।

आधार का व्यापक उपयोग 2017 तक एक बड़ी समस्या बन गया, जब निजता के अधिकार से जुड़े मुद्दे सुप्रीम कोर्ट के सामने आए। अलग से, आधार अधिनियम की संवैधानिकता को भी चुनौती दी गई थी, एवं उच्चतम न्यायालय ने उस पर भी अलग से निर्णय दिया।

आधार अब बैंक खाते, मोबाइल कनेक्शन, स्कूलों/परीक्षाओं के लिए, बीमा उत्पाद खरीदने के लिए अनिवार्य नहीं है। पैन, आई-टी रिटर्न एवं कल्याण योजनाओं के लिए यह अभी भी अनिवार्य है। आधार की संवैधानिक वैधता पर सुप्रीम कोर्ट के फैसले के अनुसार, मेटाडेटा को छह महीने से अधिक के लिए संग्रहीत नहीं किया जा सकता है। इसलिए, 1 गलत है, इसलिए विकल्प (a) एवं (d) गलत हैं। साथ ही, 3 गलत है, इसलिए (c) भी गलत है।

निजी संस्थाओं को संबंधित व्यक्ति के साथ अनुबंध के आधार पर, प्रमाणीकरण के उद्देश्य से आधार संख्या का उपयोग करने की अनुमति नहीं है, क्योंकि यह निजी संस्थाओं द्वारा किसी व्यक्ति की बायोमेट्रिक एवं जनसांख्यिकीय जानकारी का व्यावसायिक शोषण करने में सक्षम होगा।

राज्य आधार मेटाडेटा को अनुबंध के आधार पर निजी निकायों के साथ साझा नहीं कर सकते। एवं भारत के समेकित कोष (सीएफआई) से वित्त पोषित किसी भी कल्याणकारी लाभ के लिए आधार कार्ड की आवश्यकता है।

(इस को हल करने के लिए महीन जानकारी की आवश्यकता है)

52.राज्य सभा की लोक सभा के समान शक्तियाँ किस क्षेत्र में है?

  1. नई अखिल भारतीय सेवाएँ गठित करने के विषय में
  2. संविधान में संशोधन करने के विषय में।
  3. सरकार को हटाने के विषय में
  4. कटौती प्रस्ताव प्रस्तुत करने के विषय में।

उत्तर (b). राज्य सभा, राज्य परिषद के रूप में, केंद्रीय विधायिका में राज्यों की आवाज है। इसे राष्ट्रपति के महाभियोग, उप-राष्ट्रपति को हटाने, संवैधानिक संशोधनों, एवं उच्चतम न्यायालय एवं उच्च न्यायालयों के न्यायाधीशों को हटाने जैसे मामलों में लोकसभा के समान शक्तियाँ प्राप्त है। अखिल भारतीय सेवाएँ बनाने के मामले में, राज्य सभा को विशेष शक्तियाँ प्राप्त होती हैं (समान नहीं)। जब सरकार लोकसभा में धन विधेयक लाती है, तब राज्य सभा इसको नही रोक सकती है। उस मार्ग का उपयोग करके कई कानूनों को बनाया गया है।

लोकसभा के विपरीत, राज्य सभा का सदस्य सरकार के खिलाफ अविश्वास प्रस्ताव नहीं ला सकता है। इसलिए (c) गलत है।

इसके अलावा, चूंकि लोकसभा (543) की तुलना में राज्य सभा एक छोटा सदन (245 सदस्य) है, इसलिए किसी भी संयुक्त बैठक में इसकी शक्ति अपने आप कम हो जाती है।

कट मोशन लोकसभा के सदस्यों को सरकार द्वारा चर्चा किए गए वित्तीय विधेयक में एक मांग का विरोध करने के लिए दी गई शक्ति है। यदि संसद द्वारा एक कट मोशन अपनाया जाता है एवं सरकार के पास संख्या नहीं है, अतः वह लोकसभा के नियमों के अनुसार इस्तीफा देने के लिए बाध्य है। राज्य सभा की इसमें कोई भूमिका नहीं है। इसलिए (d) गलत है।

(राजनीति एवं संविधान से एक मौलिक प्रश्न!)

53.संसद सदस्य स्थानीय क्षेत्र विकास योजना (MPLADS) के अन्तर्गत निधियों के संदर्भ में, निम्नलिखित कथनों में से कौन-से सही है ?

  1. MPLADS निधियाँ टिकाऊ परिसंपत्तियों जैसे स्वास्थ्य, शिक्षा, आदि की भौतिक आधारभूत संरचनाओं के निर्माण में ही प्रयुक्त हो सकती है।
  2. प्रत्येक सांसद की निधि का एक निश्चित अंश अनुसूचित जाति/ जनजाति जनसंख्या के लाभार्थ प्रयुक्त होना आवश्यक है।
  3. MPLADS निधियाँ वार्षिक आधार पर स्वीकृत की जाती है और अप्रयुक्त निधि को अगले वर्ष के लिए अग्रेनीत नहीं किया जा सकता।
  4. कार्यान्वित हो रहे सभी कार्यों में से कम-से-कम 10OC कार्यों का जिला प्राधिकारी द्वारा प्रति वर्ष निरीक्षण अनिवार्य है।

नीचे दिए गए कूट का प्रयोग कर सही उत्तर चुनिए :

  1. केवल 1 और 2
  2. केवल 3 और 4
  3. केवल 1, 2 और 3
  4. केवल 1, 2 और 4

उत्तर (d). ‘संसद सदस्य स्थानीय क्षेत्र विकास योजना’ जिसे MPLADS के रूप में जाना जाता है, 1993 दिसंबर में शुरू की गई एक सरकारी योजना है। केंद्रीय क्षेत्र की यह योजना संसद सदस्यों को स्थानीय स्तर पर महसूस की गई जरूरतों के आधार पर अपने निर्वाचन क्षेत्रों में विकासात्मक कार्यों की अनुशंसा करने में सक्षम बनाने के लिए विकसित की गई थी। ये विकासात्मक कार्य मुख्य रूप से राष्ट्रीय प्राथमिकताओं के क्षेत्रों जैसे कि पेयजल, शिक्षा, सार्वजनिक स्वास्थ्य, स्वच्छता, सड़क आदि पर केंद्रित हैं, इसलिए 1 सही है।

यह सरकार द्वारा वित्त पोषित योजना है जहाँ प्रत्येक सांसद के निर्वाचन क्षेत्र के लिए वार्षिक 5 करोड़ रूपये का धन प्रदान किया जाता हैं। सांसदों द्वारा सिफारिश प्रतिवर्ष की जानी चाहिए, जिसमें अनुसूचित जाति की आबादी वाले क्षेत्रों में कम से कम 15 प्रतिशत एवं अनुसूचित जाति की आबादी के निवास वाले क्षेत्रों के लिए 7.5 प्रतिशत राशि खर्च की जानी चाहिए। अतः, 2 सही है।

आदिवासी लोगों की भलाई के लिए ट्रस्टों एवं समाजों को प्रोत्साहित करने के लिए योजना के दिशानिर्देशों के अनुसार ट्रस्टों एवं समाजों द्वारा संपत्ति के निर्माण के लिए 75 लाख रुपये प्रदान किए जाते हैं।

यह फंड अग्रेनीत हैं। अर्थात बचा हुआ धन अगले वर्ष कार्य में लिया जा सकता है। अतः, 3 गलत है। इसलिए (b) एवं (c) गलत हैं।

इस योजना का उपयोग 100 प्रतिशत से अधिक हो सकता है। इसकी गणना सरकार द्वारा जारी राशि एवं खर्च की गई राशि के अनुपात में की जाती है। योजना के अंर्तगत, मूल रूप से तीन प्रकार की राशि होती है - सरकार द्वारा जारी की गई, जिला अधिकारियों द्वारा स्वीकृत राशि एवं खर्च की गई राशि। स्वीकृत राशि जारी की गई राशि से अधिक हो सकती है क्योंकि इसमें भारत सरकार द्वारा जारी की गई राशि पर अर्जित ब्याज एवं पिछले वर्षों से आगे बढ़ाई गई राशि (यदि हो तो) शामिल है।

जिला प्राधिकरण जिला स्तर पर योजना के तहत कामों के समग्र समन्वय एवं पर्यवेक्षण के लिए जिम्मेदार होगा एवं हर साल कार्यान्वयन के तहत कम से कम 10ः कार्यों का निरीक्षण करेगा। जिला प्राधिकरण को परियोजनाओं के निरीक्षण में सांसदों को संभव सीमा तक शामिल करना चाहिए। अतः 4 सही है।

इसलिए, उत्तर विकल्प (d) है।

(मोदी सरकार द्वारा महामारी के दौरान MPLADS फंड के निलंबन के कारण यह प्रश्न प्रासंगिक था। लेकिन यदि आपको पता हो कि 3 गलत है तो प्रश्न सीधे हल हो सकता था)

54.निम्नलिखित मूल अधिकारों के किस संवर्ग में अस्पृश्यता के रूप में किए गए विभेदन के विरूद्ध संरक्षण समाविष्ट है ?

  1. शोषण के विरूद्ध अधिकार
  2. स्वतंत्रता का अधिकार
  3. सांविधानिक उपचार का अधिकार
  4. समता का अधिकार

. उत्तर (d). यह सब समानता के बारे में है!

संविधान द्वारा मान्यता प्राप्त सात मौलिक अधिकार हैं -

  1. >समानता का अधिकार, कानून के समक्ष समानता, धर्म, जाति, जाति, लिंग या जन्म स्थान के आधार पर भेदभाव का निषेध, एवं रोजगार के मामलों में अवसर की समानता, अस्पृश्यता का उन्मूलन एवं शीर्षकों का उन्मूलन।
  2. स्वतंत्रता का अधिकार जिसमें भाषण एवं अभिव्यक्ति, विधानसभा, संघ या संघ या सहकारिता, आंदोलन, निवास एवं किसी भी पेशे या व्यवसाय का अभ्यास करने का अधिकार शामिल है (इनमें से कुछ अधिकार राज्य की सुरक्षा के अधीन हैं, विदेशी देशों के साथ मैत्रीपूर्ण संबंध, सार्वजनिक आदेश, शालीनता या नैतिकता), जीवन एवं स्वतंत्रता का अधिकार, शिक्षा का अधिकार, अपराधों में सजा के संबंध में संरक्षण एवं कुछ मामलों में गिरफ्तारी एवं हिरासत के खिलाफ संरक्षण।
  3. शोषण के खिलाफ अधिकार, मानव में सभी प्रकार के श्रम, बाल श्रम एवं तस्करी पर प्रतिबंध।
  4. धर्म की स्वतंत्रता, जिसमें अंतरात्मा की स्वतंत्रता एवं मुक्त पेशा, अभ्यास, एवं धर्म का प्रचार, धार्मिक मामलों का प्रबंधन करने की स्वतंत्रता, कुछ करों से मुक्ति एवं कुछ शैक्षिक संस्थानों में धार्मिक निर्देशों से स्वतंत्रता शामिल है।
  5. अपनी पसंद के शैक्षिक संस्थानों की स्थापना एवं प्रशासन के लिए उनकी संस्कृति, भाषा या लिपि एवं अल्पसंख्यकों के अधिकार के संरक्षण के लिए नागरिकों के किसी भी वर्ग के अधिकार को संरक्षित करने वाले सांस्कृतिक एवं शैक्षिक अधिकार।
  6. मौलिक अधिकारों के प्रवर्तन के लिए संवैधानिक उपचार का अधिकार।
  7. शिक्षा का अधिकार जो यह सुनिश्चित करता है कि 14 वर्ष की आयु तक के बच्चे शिक्षा प्राप्त करें। यह मुफ्त भी हो सकती है।

एक सीधा आसान प्रश्न!)

55.भारत में, न्यायपालिका का कार्यपालिका से पृथक्करण, किससे व्यादेशित है ?

  1. संविधान की उद्देशिका द्वारा
  2. राज्य की नीति के निदेशक द्वारा
  3. सातवीं अनुसूची द्वारा
  4. परम्परागत व्यवहार द्वारा

. उत्तर (b). भारतीय संविधान का अनुच्छेद 50 राज्य को न्यायपालिका को कार्यपालिका से अलग करने के लिए एक दायित्व देता है, एवं चूंकि यह राज्य के नीति निर्देशक तत्वों (डीपीएसपी) के अंतर्गत आता है, इसलिए यह कानूनी रूप बाध्यकारी नहीं है।

अनुच्छेद 50 -

न्यायपालिका को कार्यपालिका से अलग करना - राज्य राज्य की सार्वजनिक सेवाओं में न्यायपालिका को कार्यपालिका से अलग करने के लिए कदम उठाना

(यदि आपको अनुच्छेद 50 याद है तो प्रश्न आसान था)

56.वित्त मंत्री संसद में बजट प्रस्तुत करते हुए उसके साथ अन्य प्रलेख भी प्रस्तुत करते हैं जिनमें ‘वृहद् आर्थिक रूपरेखा विवरण (The Macro Economic Framework Statement)’ भी सम्मिलित रहता है। यह पूर्वाक्त प्रलेख निम्न आदेशन के कारण प्रस्तुत किया जाता है :

  1. चिरकालिक संसदीय परम्परा के कारण
  2. भारत के संविधान के अनुच्छेद 112 तथा अनुच्छेद 110(1) के कारण
  3. भारत के संविधान के अनुच्छेद 113 के कारण
  4. राजकोषीय उत्तरदायित्व एवं बजट प्रबन्धन अधिनियम, 2003 के प्रावधानों के कारण

उत्तर (d). इस परीक्षा में सबसे आसान प्रश्नों में से एक!

वृहद आर्थिक रूपरेखा विवरण, जो राजकोषीय उत्तरदायित्व एवं बजट प्रबंधन अधिनियम (FRBM), 2003 के अनुसार संसद में प्रस्तुत किया गया है। FRBM सरकार को विशिष्ट अंतर्निहित धारणाओं के साथ अर्थव्यवस्था की विकास संभावनाओं का आकलन करने का निर्देश देता है। रूपरेखा स्टेटमेंट में जीडीपी विकास दर, केंद्र सरकार का राजकोषीय संतुलन एवं अर्थव्यवस्था के बाहरी क्षेत्र के संतुलन के बारे में मूल्यांकन शामिल है।

(आप इसे गलत नहीं कर सकते!)

57.परिभाषा से, संवैधानिक सरकार का अर्थ है

  1. विधान मंडल द्वारा सरकार
  2. लोकप्रिय सरकार
  3. बहु-दलीय सरकार
  4. सीमित सरकार

उत्तर (d). सीमित सरकार एक विचार है कि एक सरकार को कानून बनाने एवं उन्हें लागू करने के लिए असीमित अधिकार नहीं दिए जा सकते। इसलिए हमें इसे परिभाषित करने के लिए एक संविधान की आवश्यकता है। 1789 का अमेरिकी संविधान एवं 1793 का फ्रांसीसी संविधान दोनों को सीमित सरकार बनाने के लिए तैयार किया गया था। इसे प्राप्त करने के लिए एक प्रमुख उपकरण सरकार के अंगों एवं संघीय आधार पर शक्तियों का पृथक्करण है।

स्कॉट गॉर्डन के अनुसार, एक राजनीतिक संगठन इस हद तक संवैधानिक है कि इसमें ‘उन लोगों के हित’ की सुरक्षा एवं नागरिकता के स्वतंत्रता के संरक्षण के लिए संस्थागत तंत्र शामिल हैं, जिनमें अल्पसंख्यक शामिल हो सकते हैं’।

(एक आसान प्रश्न है!)

58.मूल अधिकारों के अतिरिक्त, भारत के संविधान का निम्नलिखित में से कौन-सा/से भाग मानव अधिकारों की सार्वभौम घोषणा 1948 (Universal Declaration of Human Rights 1948) के सिद्धांतो एवं प्रावधानों को प्रतिबिंबित करता /करते है/हैं ?

  1. उद्देशिका
  2. राज्य की नीति के निदेशक तत्व
  3. मूल कर्तव्य

नीचे दिए गए कूट का प्रयोग कर सही उत्तर चुनिए :

  1. केवल 1 और 2
  2. केवल 2
  3. केवल 1 और 3
  4. 1, 2 और 3

उत्तर (d). सार्वभौम घोषणा के अनुच्छेद 1 में कहा गया है कि “सभी मनुष्य सम्मान एवं अधिकारों के लिए स्वतंत्र एवं समान पैदा होते हैं। उन्हें तर्क एवं विवेक से संपन्न होना चाहिए एवं भाईचारे की भावना से एक-दूसरे के प्रति कार्य करना चाहिए। भारत मानवाधिकारों की सार्वभौम घोषणा का एक हस्ताक्षरकर्ता था। भारतीय संविधान के भाग प्प्प् में व्यक्तियों को दिए गए कई मौलिक अधिकार मानव अधिकारों की सार्वभौमिक घोषणा के प्रावधानों के समान हैं।

संविधान की उद्धेशिका भारत को एक संप्रभु, समाजवादी, धर्मनिरपेक्ष एवं लोकतांत्रिक गणराज्य घोषित करती है। ‘लोकतांत्रिक’ शब्द का अर्थ है कि सरकार को लोगों की इच्छा से उसका अधिकार प्राप्त है। यह महसूस करता है कि वे सभी ‘जाति, धर्म, भाषा, लिंग एवं संस्कृति के बावजूद’ समान हैं। अतः, 1 सही है।

संविधान के भाग प्ट में राज्य नीति के न्यायिक सिद्धांत (न्यायिक रूप से गैर-प्रवर्तनीय अधिकार) मुख्यतः आर्थिक एवं सामाजिक चरित्र के हैं। हालाँकि, अनुच्छेद 37 यह स्पष्ट करता है कि उनकी न्यायिक गैर-प्रवर्तनीयता कानून बनाने में उन्हें लागू करने के लिए राज्य के कर्तव्य को कमजोर नहीं करती है, क्योंकि वे काउंटी के शासन में फिर भी मौलिक हैं। इसके अतिरिक्त, सर्वोच्च न्यायालय के अभिनव न्यायशास्त्र ने अब अनुच्छेद 21 (जीवन एवं व्यक्तिगत स्वतंत्रता के अधिकार) को इन सिद्धांतों में से कईयों को न्यायालय में चुनौति देनेयोग्य बनाया है। अतः, 2 सही है।

संविधान का भाग IV (।) प्रत्येक भारतीय नागरिक के अनुच्छेद मौलिक कर्तव्यों को लागू करता है (अनुच्छेद 51-ए)। छियासीवें संवैधानिक संशोधन 2002 ने अनुच्छेद 51 (a) में एक नया खंड (k) सम्मिलित किया, जिसमें निर्देश दिया गया था कि ‘माता-पिता या अभिभावक को अपने 6 से 14 वर्ष तके बच्चों को की शिक्षा के लिए अवसर प्रदान करें।’ यहाँ तक 3 भी सत्य है।

(एक कठिन प्रश्न जिसके लिए गहन चिंतन और समझ की आवश्यकता होती है)

59.भारत में विधिक सेवा प्रदान करने वाले प्राधिकरण (Legal Services Authorities), निम्नलिखित में से किस प्रकार के नागरिकों को निःशुल्क विधिक सेवाएं प्रदान करते हैं ?

  1. रू. 1,00,000 से कम वार्षिक आय वाले व्यक्ति को
  2. रू. 2,00,000 से कम वार्षिक आय वाले ट्रांसजेंडर को
  3. रू. 3,00,000 से कम वार्षिक आय वाले अन्य पिछड़े वर्ग (OBC) के सदस्यों को
  4. सभी वरिष्ठ नागरिकों को

नीचे दिए गए कूट का प्रयोग कर सही उत्तर चुनिए :

  1. केवल 1 और 2
  2. केवल 3 और 4
  3. केवल 2 और 3
  4. केवल 1 और 4

उत्तर (a). यहां, 4 गलत है, इसलिए विकल्प (b) एवं (d) से इनकार किया जा सकता है।

वे व्यक्ति जो विधिक सेवा प्राधिकरण अधिनियम, 1987 के तहत मुफ्त कानूनी सहायता पाने के हकदार हैं -

  1. अनुसूचित जनजाति के एक अनुसूचित जाति के सदस्य,
  2. संविधान के अनुच्छेद 23 में उल्लिखित मानव या तस्करी का शिकार व्यक्ति,
  3. एक महिला या एक बच्चा
  4. विकलांगता (समान अवसर, अधिकारों का संरक्षण एवं पूर्ण भागीदारी), अधिनियम, 1995 के खंड 2 के खंड (I) में परिभाषित विकलांगता वाला व्यक्ति
  5. कोई व्यक्ति जो सामूहिक आपदा, जातीय हिंसा, जातिगत अत्याचार, बाढ़, सूखा, भूकंप या औद्योगिक आपदा या ऐसी ही किसी और दुर्घटना का शिकार हा जिसमें मदद की आवश्यकता है, या या एक औद्योगिक कर्मकार, या हिरासत में कोई व्यक्ति, अनैतिक तस्करी (रोकथाम) अधिनियम, 1956 (1956 का 104) की धारा 2 के खंड (छ) के अर्थ में एक सुरक्षात्मक हिरासत सहित, या किशोर न्याय अधिनियम, 1986 (1986 का 53) की धारा 2, खंड (जे) के भीतर एक किशोर गृह में, या मानसिक स्वास्थ्य अधिनियम, 1987 (1987 का 14) की धारा 2 के खंड (छ) के अर्थ के भीतर एक मनोरोग अस्पताल या मनोरोग नर्सिंग होम में या राज्य सरकार द्वारा निर्धारित नौ हजार रुपये या उससे अधिक की वार्षिक आय से कम वार्षिक आय प्राप्त करने पर, यदि मामला सर्वोच्च न्यायालय के अलावा किसी अन्य के समक्ष है, एवं रुपये बारह हजार से कम या ऐसी अन्य उच्च राशि के रूप में हो सकता है (यदि मामला उच्चतम न्यायालय के समक्ष है तो केंद्र सरकार द्वारा निर्धारित किया जाता है)
  6. “प्रत्येक वरिष्ठ नागरिक” को निःशुल्क विधिक सेवाएँ प्रदान नहीं की जाती है, इसलिए कथन 4 सही नहीं है। इसलिए उत्तर ;ंद्ध या ;बद्ध होगा। लेकिन यह 1,00,000 रुपये से कम वार्षिक आय वाले व्यक्ति को प्रदान किया जाता है। अतः सही उत्तर विकल्प (a) होगा।

स्त्रोत - http://www.legalserviceindia.com/article/l206-Legal-Aid.html

(कठिन प्रश्न, स्थैतिक एवं समसामयिक सामान्य ज्ञान पर आधारित)

60.निम्नलिखित युग्मों पर विचार कीजिए :

अंतर्राष्ट्रीय समझौता/संगठन विषय

  1. अल्मा-आटा घोषणा - लोगों के स्वास्थ्य की देखभाल
  2. हेग समझौता - जैविक एवं रासायनिक शास्त्र
  3. तलानोआ संवाद - वैश्विक जलवायु परिवर्तन
  4. अंडर2 गठबंधन - बाल अधिकार

उपर्युक्त में से कौन-सा/से युग्म सही सुमेलित है/हैं?

  1. केवल 1 और 2
  2. केवल 4
  3. केवल 1 ओर 3
  4. केवल 2, 3 और 4

उत्तर (c). अल्मा-आटा घोषणा को प्राथमिक स्वास्थ्य देखभाल के लिए, अल्माटी, कजाकिस्तान में 6 से 12 सितंबर 1978 को अंतर्राष्ट्रीय सम्मेलन में अपनाया गया था। इसलिए, कथन 1 सही है। उत्तर (a) या (c) हो सकता है। लेकिन द हेग कन्वेंशन इंटरनेशनल चाइल्ड अपहरण के नागरिक पहलुओं पर था। अतः सही उत्तर (c) है।

तलानोआ संवाद एक प्रक्रिया है जो देशों को अपने औद्योगिक स्तर पर औसत वैश्विक तापमान में वृद्धि को 2° से तक बढ़ाने एवं 2020 तक 1.5 ° से तक वृद्धि को सीमित करने के प्रयासों को आगे बढ़ाने के लिए अपने राष्ट्रीय स्तर पर निर्धारित योगदान को लागू करने एवं बढ़ाने में मदद करने के लिए डिजाइन की गई है।

अंडर2 गठबंधन राज्य एवं क्षेत्रीय सरकारों का एक वैश्विक समुदाय है जो महत्वाकांक्षी जलवायु शमन परिणामों के लिए प्रतिबद्ध है।

(एक कठिन प्रश्न जो, स्थैतिक सामान्य ज्ञान पर आधारित है)


61.भारत के इतिहास के संदर्भ में निम्नलिखित युग्मों पर विचार कीजिए :

  1. औरंग - राजकोष का प्रभारी
  2. बेनियान - ईस्ट इंडिया कंपनी का भारतीय एजेंट
  3. मिरासिदार - राज्य का नामित राज्सव दाता

उपर्युक्त युग्मों में से कौन-सा/से सही सुमेलित है/हैं?

  1. केवल 1 और 2
  2. केवल 2 और 3
  3. केवल 3
  4. 1, 2 और 3

उत्तर (c). 1 सही नही है क्योंकि औरंग, गोदाम के लिए फारसी शब्द है (एक ऐसी जगह जहां सामान बेचा जाने से पहले एकत्र किया जाता है)। शब्द का अर्थ भी तैयार माल के लिए एक कार्यशाला को संदर्भित करता है। बनिया का अर्थ एक भारतीय व्यापारी, खजांची या मनी चेंजर से है। अतः 2 भी सही नहीं है। अतः उत्तर होना चाहिए (c)।

भारत में (विशेषकर दक्षिण भारत में), (मिरासीदार ’एक वंशानुगत स्वामी है, विशेषकर भूमि का, विरासत में मिली भूमि के एक मालिक को कानून द्वारा प्रतिबंधित किया गया है, एवं इसकी खेती के बदले में मामूली शुल्क के लिए किरायेदारों के बजाय पट्टे पर दिया गया है। मुट्ठी भर गाँवों के निवासी जिन्हें मीरासीदार कहा जाता है, अक्सर ब्राह्मण एवं अन्य उच्च जातियों के होते हैं। उनके तहत किसानों का एक समूह था, जो यातो गाँव की जमीन पर खेती करने के लिए स्थायी अधिकार रखते थे या अन्य गाँवों से संबंधित अस्थायी किरायेदार थे। रैयतवारी बंदोबस्त प्रणाली के तहत, सरकार ने मीरासीदारों को जमीन के एकमात्र मालिक के रूप में मान्यता दी, किरायेदारों के अधिकारों को पूरी तरह से खारिज कर दिया। केवल उन गाँवों में जहाँ कोई मीरासीदर प्रणाली मौजूद नहीं थी, वे ग्रामीण थे, जिनके पास भू-राजस्व के भुगतान के लिए जिम्मेदार भू-स्वामियों के रूप में मान्यता प्राप्त स्थायी अधिभोग अधिकार थे।

(कठिन प्रश्न, प्राचीन इतिहास पर आधारित)

62.भारत के धार्मिक इतिहास के संर्दी में, निम्नलिखित कथनों पर विचार कीजिए :

  1. स्थाविरवादी महायान बौद्ध धर्म से संबद्ध है।
  2. लोकोत्तरवादी संप्रदाय बौद्ध धर्म के महासंघिक संप्रदाय की एक शाखा थी।
  3. महासंघिको द्वारा बुद्ध के देवत्वारोपण ने महायान बौद्ध धर्म को प्रोत्साहित किया।

उपर्युक्त कथनों में से कौन-सा/से कथन है/हैं?

  1. केवल 1 और 2
  2. केवल 2 और 3
  3. केवल 3
  4. 1, 2 और 3

उत्तर (b). बौद्ध समुदाय में पहला विभाजन दूसरी परिषद के परिणामस्वरूप हुआ था, कहा जाता है कि बुद्ध की मृत्यु के सौ वर्ष बाद, वैशाली (बिहार राज्य) में, जब अकारियावादिंस (पारंपरिक शिक्षण के अनुयायी) स्थाविरवाद से अलग हो गए थे (बड़ों के मार्ग के अनुयायी) एवं अपने स्वयं के स्कूल का गठन किया, जिसे महासंघिका के रूप में जाना जाता है।

बुद्ध एवं अरहत (संत) की प्रकृति पर महासंघिकों के विचारों ने बौद्ध धर्म के महायान रूप के विकास को रेखांकित किया। अतः कथन 1 गलत है एवं 3 सही है। उत्तर (b) या (c) हो सकता है।

अगले सात शताब्दियों में महासंघिकों के आगे के उपखंडों में लोकोत्तरावादी, एकलव्यहारीक एवं कौकुतिकास शामिल थे। अतः 2 सही है।

(एक कठिन एक, प्राचीन भारतीय इतिहास के मिनट के विवरण पर आधारित)

63.निम्नलिखित में से कौन-सा कथन औद्योगिक क्रान्ति के द्वारा उन्नीसवीं शताब्दी के पूर्वार्ध में भारत पर पड़े प्रभाव की सही व्याख्या करता है ?

  1. भारतीय दस्तकारी-उद्योग नष्ट हो गए थे।
  2. भारत के वस्त्र-उद्योग में मशीनों का बड़ी संख्या में प्रवेश हुआ था।
  3. भारत के अनेक भागों में रेलवे लाइनें बिछाई गई थी।
  4. ब्रिटिश उत्पादन के आयात पर भारी शुल्क लगाया गया था।

उत्तर (a). भारत में पहली रेल लाइन 16 अप्रैल, 1853 को (मुंबई से ठाणे) व्यवहार में आई थी। इसलिए विकल्प (ब) सही नहीं है। आधुनिक कपड़ा उद्योग ने उन्नीसवीं सदी की शुरुआत में भारत में जन्म लिया जब देश में पहली कपड़ा मिल 1818 में कलकत्ता के पास फोर्ट ग्लस्टर में स्थापित की गई थी, लेकिन सूती कपड़ा उद्योग ने 1850 के दशक में बॉम्बे में अपनी वास्तविक शुरुआत की। बंबई की पहली सूती कपड़ा मिल 1854 में एक पारसी सूती व्यापारी द्वारा स्थापित की गई थी, जो विदेशी और आंतरिक व्यापार में लगा था। अहमदाबाद में पहली कपड़ा मिल 1861 में स्थापित की गई थी। सूती कपड़ा उद्योग ने उन्नीसवीं सदी के उत्तरार्ध में तेजी से प्रगति की और सदी के अंत तक 178 कपास कपड़ा मिलें थीं। इसलिए विकल्प (b) गलत है। विकल्प (d) स्पष्ट रूप से गलत है।

इसलिए, भारतीय हस्तशिल्प बर्बाद हो गए, इसका सही उत्तर है।

(कठिन प्रश्न, अन्य विकल्प भ्रमित कर सकते हैं)

64.भारत के इतिहास में निम्नलिखित घटनाओं पर विचार कीजिए :

  1. राजा भोज के अधीन प्रतिहारों का उदय
  2. महेन्द्रवर्मन-I के अधीन पल्लव सत्ता की स्थापना
  3. परान्तक-I द्वारा चोल सत्ता की स्थापना
  4. गोपाल द्वारा पाल राजवंश की स्थापना

उपर्युक्त घटनाओं का, प्राचीन काल से आरम्भ कर, सही कालानुक्रम क्या है ?

  1. 2-1-4-3
  2. 3-1-4-2
  3. 2-4-1-3
  4. 3-4-1-2

उत्तर (c). कथन 1 - गुर्जर-प्रतिहार, या केवल, प्रतिहार (आठवीं शताब्दी ईस्वी - ग्याहरवीं शताब्दी ईस्वी)। भोज या मिहिर भोज (836-885 इस्वी) इस राजवंश के सबसे प्रसिद्ध राजा थे।

कथन 2 - पल्लव वंश (275-897 ईस्वी)। महेंद्रवर्मन I (571-630 ईस्वी)।

कथन 3 - चोल राजवंश दक्षिणी भारत का एक तमिल थैलासोक्रेटिक साम्राज्य (नौसैनिक शक्ति पर निर्भर) था, जो दुनिया के इतिहास में सबसे लंबे समय तक राज करने वाले राजवंशों में से एक था। चोल का सबसे पुराना ज्वलनशील संदर्भ मौर्य साम्राज्य (अशोक मेजर रॉक एडिट नं. 13) के अशोक द्वारा छोड़ी गई तीसरी शताब्दी ईसा पूर्व के शिलालेखों में है। परांतक I लगभग 907-950 ईस्वी के बीच था।

कथन 4 - गोपाल (750-770 ईस्वी) बंगाल क्षेत्र के पाल राजवंश के संस्थापक थे।

(यह प्रश्न बिल्कुल भी आसान नही था)

65.निम्नलिखित में से कौन-सा उपवाक्य, उत्तर-हर्ष-कालीन स्त्रोतों में प्रायः उल्लिखित ‘हुंडी' के स्वरूप की परिभाषा बताता है ?

  1. राजा द्वारा अपने अधीनस्थों को दिया गया परामर्श।
  2. प्रतिदिन का लेखा-जोखा अंकित करने वाली बही
  3. विनिमय पत्र
  4. सामन्त द्वारा अपने अधीनस्थों को दिया गया आदेश।

उत्तर (c). हुंडी एक वित्तीय साधन है जो व्यापार एवं ऋण लेनदेन में उपयोग के लिए मध्यकालीन भारत में विकसित हुआ है। हुंडियों का उपयोग प्रेषण साधन के रूप में एक स्थान से दूसरे स्थान पर धन हस्तांतरण के लिए किया जाता है, एक साधन के रूप में या उधार लेने के लिए एवं व्यापार लेनदेन में विनिमय के बिल के रूप में भारतीय रिजर्व बैंक ने हुंडी का वर्णन ‘एक व्यक्ति द्वारा लिखित आदेश में एक निश्चित राशि का भुगतान करने के लिए किसी अन्य व्यक्ति को निर्देश देने के बिना शर्त आदेश के रूप में किया है।’

औपनिवेशिक काल के दौरान, ब्रिटिश सरकार ने हुंडी प्रणाली को स्वदेशी या पारंपरिक माना, लेकिन अनौपचारिक नहीं। हुंडी के प्रकार - सहयोग हुंडी, दर्शनी हुंडी, मुदाती हुंडी, नाम-जोग हुंडी, फुरमान-जोग हुंडी, धानी-जोग हुंडी, जोकिम-हुंडी, जवाबी हुंडी, खाका हुंडी, खोती हुंडी आदि।

(यह आसान था क्योंकि केवल चार विकल्पों में से एक वित्त एवं धन से संबंधित था)

66.स्वतन्त्रता संग्राम के समय लिखी गई सखाराम गणेश देउस्कर की पुस्तक ‘देशेर कथा’के संदर्भ में, निम्नलिखित कथनों पर विचार कीजिए :

  1. इस पुस्तक ने औपनिवेशिक राज्य द्वारा मस्तिष्क की सम्मोहक विजय के विरोध में चेतावनी दी।
  2. इस पुस्तक ने स्वदेशी नुक्कड़ नाटकों का तथा लोग गीतों को प्रेरित किया।
  3. देउस्कर द्वारा ‘देश’ शब्द का प्रयोग, बंगाल क्षेत्र के विशिष्ट संदर्भ में किया गया था।

उपर्युक्त कथनों में से कौन-से सही है ?

  1. केवल 1 और 2
  2. केवल 2 और 3
  3. केवल 1 और 3
  4. 1, 2 और 3

उत्तर (पक्के तौर पर नही कह सकते). सखाराम गणेश देउस्कर (1869 - 1912) क्रांतिकारी लेखक, इतिहासकार तथा पत्रकार थे। वे भारतीय जन-जागरण के ऐसे विचारक थे जिनके चिंतन एवं लेखन में स्थानीयता एवं अखिल बांग्ला तथा चिंतन-मनन का क्षेत्र इतिहास, अर्थशास्त्र, समाज एवं साहित्य था। वे महाराष्ट्र एवं बंगाल के नवजागरण के बीच सेतु के समान हैं। उनका प्रेरणा-स्रोत महाराष्ट्र है, पर वे लिखते बांग्ला में हैं। अरविंद घोष ने लिखा है कि ‘स्वराज्य’ शब्द के पहले प्रयोग का श्रेय देउस्कर को ही जाता है। पत्रकार के अतः र पर जीवन की शुरुआत करने वाले देउस्कर की इतिहास, साहित्य एवं राजनीति में विशेष रूप से रुचि थी। उन्होंने बांग्ला की अधिकांश क्रांतिकारी पत्रिकाओं में सतत लेखन किया। युगांतर पत्रिका के वे नियमित लेखक थे। युगांतर के अलावा उन्होंने साहित्य, भारती, धरनी, साहित्य-संहिता, प्रदीप, बंग-दर्शन, आर्यावर्त्त, वेद व्यास, प्रतिभा आदि पत्रिकाओं में भारत के इतिहास, संस्कृति, साहित्य आदि से संबंधित बहुत सारे लेख लिखे, जिनका उद्देश्य भारतीय जनता को अपने अतीत एवं वर्तमान का ज्ञान कराना था। इतिहास, साहित्य एवं राजनीति उनके प्रिय विषय थे।

देउस्कर की जिस एक रचना ने नवजागरण काल के प्रबुद्धवर्ग को सर्वाधिक प्रभावित किया, वह थी 1904 में प्रकाशित कृति ‘देशेर कथा’। इसका हिंदी-अनुवाद ‘देश की बात’ (1910) नाम से बाबूराव विष्णु पराड़कर ने किया। विलियम डिग्बी, दादाभाई नौरोजी एवं रमेश चंद्र दत्त ने भारतीय अर्थव्यवस्था के जिस विदेशी शोषण के बारे में लिखा था, सखाराम देउस्कर ने मुख्यतः उसी आधार पर इस ऐतिहासिक कृति की रचना की। हिंदुस्तान के उद्योग-धंधों की बर्बादी का चित्रण करती देउस्कर की यह कृति ब्रिटिश साम्राज्यवाद की जंजीरों में जकड़ी एवं शोषण के तले जीती-मरती भारतीय जनता के रुदन का दस्तावेज है। मात्र पांच वर्षों में इसके पांच संस्करण की तेरह हजार प्रतियों के प्रकाशन की सूचना से भयभीत अंग्रेजों ने सन् 1910 में इस पुस्तक पर पाबंदी लगा दी। भारतीय अर्थव्यवस्था को तबाह करने के लिए यहां की कृषि व्यवस्था कारीगरी एवं उद्योग-धंधों को तहस-नहस करने एवं भारतीय नागरिक के संबंध में अवमानना भरे वाक्यों का व्यवहार करने की घटनाओं का प्रमाणिक चित्र यहां उपस्थित है।

67.गाँधी-इरविन समझौते में निम्नलिखित में से क्या सम्मिलित था/थे ?

  1. राउंड टेबल कॉन्फ्रेंस में भाग लेने के लिए काँग्रेस को आमन्त्रित करना।
  2. असहयोग आंदोलन के संबंध में जारी किए गए अध्यादेशों को वापस लेना।
  3. पुलिस की ज़्यादतियों की जाँच करने हेतु गाँधीजी के सुझाव की स्वीकृति
  4. केवल उन्हीं कैदियों की रिहाई जिन पर हिंसा का अभियोग नहीं था।

नीचे दिए गए कूट का प्रयोग कर सही उत्तर चुनिए :

  1. केवल 1
  2. केवल 1, 2 और 4
  3. केवल 3
  4. केवल 2, 3 और 4

उत्तर (b). गांधी-इरविन समझौता लंदन में दूसरे गोलमेज सम्मेलन से पहले 1931 में भारत के वायसराय महात्मा गांधी एवं लॉर्ड इरविन द्वारा हस्ताक्षरित एक राजनीतिक समझौता था। नीचे प्रस्तावित शर्तें हैं -

  1. भारतीय राष्ट्रीय कांग्रेस द्वारा नमक मार्च को बंद करना।
  2. दूसरे गोलमेज सम्मेलन में भारतीय राष्ट्रीय कांग्रेस द्वारा भागीदारी।
  3. भारतीय राष्ट्रीय कांग्रेस की गतिविधियों पर भारत सरकार द्वारा जारी सभी अध्यादेशों को वापस लेना।
  4. हिंसा को छोड़कर सभी प्रकार के अपराधों से संबंधित सभी अभियोगों की वापसी।
  5. नमक सत्याग्रह में भाग लेने वाले गिरफ्तार कैदियों की रिहाई।
  6. नमक पर कर को हटाना, जिसने भारतीयों को कानूनी रूप से एवं अपने निजी उपयोग के लिए नमक का उत्पादन, व्यापार एवं बिक्री करने की अनुमति दी।

जवाब में, ब्रिटिश सरकार ने सहमति व्यक्त की -

  1. सभी अध्यादेशों को वापस लेना एवं अभियोजन समाप्त करना।
  2. हिंसा के दोषी लोगों को छोड़कर सभी राजनीतिक कैदियों को रिहा करना।
  3. शराब एवं विदेशी कपड़ा दुकानों की शांतिपूर्ण पिकेटिंग की अनुमति।
  4. सत्याग्रहियों की जब्त संपत्तियों को बहाल करना।
  5. समुद्री तट के पास व्यक्तियों द्वारा मुफ्त संग्रह या नमक के निर्माण की अनुमति।
  6. कांग्रेस पर पाबंदी हटाना।

(एक कठिन प्रश्न जो प्रसिद्ध ऐतिहासिक घटनाओं की सूक्ष्म जानकारियों पर आधारित है)

68.अस्पृश्य समुदाय के लोगों को लक्षित कर, प्रथम मासिक पत्रिका विटाल-विध्वंसक किसके द्वारा प्रकाशित की गई थी?

  1. गोपाल बाबा वलंगकर
  2. ज्योतिबा फुले
  3. मोहनदास करमचन्द गाँधी
  4. भीमराव रामजी अम्बेडकर

उत्तर (a). गोपाल बाबा वलंगकर (1840-1900) भारत के अछूत लोगों के लिए काम करने वाले एक कार्यकर्ता का एक प्रारंभिक उदाहरण है। उन्होंने उत्पीड़न को समझाने के लिए एक नस्लीय सिद्धांत विकसित किया एवं अछूत लोगों पर लक्षित पहली पत्रिका भी प्रकाशित की। आर्य आक्रमण के सिद्धांत, खारिज होने के बाद से, यह स समय प्रचलन में था। वालंगकर ने इस नस्लीय सिद्धांत के फुले के संस्करण को बढ़ाया, कि भारत के अछूत लोग स्वदेशी निवासी थे एवं ब्राह्मण लोग आर्यों के वंशज थे जिन्होंने देश पर आक्रमण किया था। वालंगकर ने दावा किया कि ‘दक्षिण के उच्च जाति के लोग ‘ऑस्ट्रेलियाई-सेमेटिक गैर-आर्य एवं अफ्रीकी नीग्रो थे, एवं चितपावन ब्राह्मण ‘बर्बरी यहूदी’ थे, एवं यह कि उच्च जाति के मराठा ‘अग्रदूत’ थे।’ 1888 में, वालंगकर ने विटाल-विध्वंशक (ब्राह्मण या कर्मकांड प्रदुषण का विनाशक) शीर्षक से मासिक पत्रिका प्रकाशित करना शुरू किया, जो हरिजनों के लिए पहली पढ़ने योग्य सामग्री थी। उन्होंने सुधारक एवं दीनबंधु नाम के मराठी अखबारों में लेख भी लिखें, उन्होंने मराठी भाषा में दोहे भी लिखें जिससे कई लोग पे्ररित हुए।

(एक गुढ़ प्रश्न, अत्यंत कठिन)

69.भारत के इतिहास के संदर्भ में, "कुल्यावाप" तथा "द्रोणवाप" शब्द क्या निर्दिष्ट करते हैं ?

  1. भू-माप
  2. विभिन्न मौद्रिक मूल्यों के सिक्के
  3. नगर की भूमि का वर्गीकरण
  4. धार्मिक अनुष्ठान

उत्तर (a). एपिग्राफिक साक्ष्यों के अनुसार, हमारे पास भूमि की माप इकाइयों के संदर्भ हैं, जैसे - ‘कुल्यावाप’, ‘द्रोणवाप’, ‘अद्वैप’ और ‘पटाका’ जो बंगाल में उपयोग की जाती थी, निवरत्न और ‘भूमि’ जो मध्य भारत से थी एवं ‘निवार्ताना’ तथा ‘पदावर्त’ जो पश्चिमी भारत से थी।

(तथ्य आधारित प्रश्न)

70.निम्नलिखित में से किस शासक ने अपनी प्रजा को इस अभिलेख के माध्यम से परामर्श दिया ?

‘‘कोई भी व्यक्ति जो अपने संप्रदायको महिमा-मंडित करने की दृष्टि से अपने धार्मिक संप्रदाय की प्रशंसा करता है या अपने संप्रदाय के प्रति अत्यधिक भक्ति के कारण अन्य संप्रदायों की निन्दा करता है, वह अपितु अपने संप्रदाय को गंभीर रूप से हानि पहुँचाता है।’’

  1. अशोक
  2. समुद्रगुप्त
  3. हर्षवर्धन
  4. कृष्णदेव राय

उत्तर (a). अशोक के शिलालेख - शिलालेख ग्प्प् - मीना तालीम की व्याख्या के अनुसार गिरनार संपादन का मूल पाठ। कोई व्यक्ति जो अपने संप्रदाय को महिमा-मंडित करने की द्रष्टि से .....।

71.संसदीय व्यवस्था वाली सरकार वह होती है जिसमें

  1. संसद के सभी राजनीतिक दलों का सरकार में प्रतिनिधित्व होता है।
  2. सरकार संसद के प्रति उत्तरदायी होती है और उसके द्वारा हटाई जा सकती हैं
  3. सरकार लोगों के द्वारा निर्वाचित होती है और उनके द्वारा हटाई जा सकती है।
  4. सरकार संसद के द्वारा चुनी जाती है किंतु निर्धारित समयावधि के पूर्ण होने के पूर्व हटाई नहीं जा सकती।

उत्तर (b). संसदीय व्यवस्था वाली सरकार में, बहुमत वाली पार्टी को सरकार (राजनीतिक कार्यकारी) बनाने का मौका मिलता है। भारत में, निर्वाचक संसद के निचले सदन के सदस्यों का चयन करते हैं और ये सांसद सरकार बनाते हैं। यह सरकार संसद के प्रति उत्तरदायी रहती है और संसद इसे अविश्वास प्रस्ताव (यदि पारित हो) के माध्यम से हटा सकती है। विकल्प (a) स्पष्ट रूप से गलत है क्योंकि ऐसा कोई मानक नहीं है। विकल्प (c) गलत है क्योंकि लोग सरकार को हटा नहीं सकते हैं। विकल्प (d) गलत है क्योंकि सरकारें अपना कार्यकाल पूरा होने से पहले गिरती हैं (अविश्वास प्रस्ताव के कारण, आदि)। विकल्प (a) और (c) को सीधे अलग किया जा सकता है,

(तथ्य आधारित प्रश्न)

72.भारत के संविधान का कौन-सा भाग कल्याणकारी राज्य के आदर्श की घोषणा करता है ?

  1. राज्य की नीति के निदेशक तत्व
  2. मूल अधिकार
  3. उद्देशिका
  4. सातवीं अनुसूची

उत्तर (a). संविधान का वह हिस्सा जो कल्याणकारी राज्य के आदर्शों की घोषणा करता है, वह है राज्य नीति के निदेशक सिद्धांत। ये सिद्धांत नागरिक कल्याण सुनिश्चित करने के लिए वांछनीय कार्यों के बारे में देश की सरकारों (केंद्र एवं राज्य) को दिशा-निर्देश देते हैं। ये कानूनों को तैयार करने में भी मदद करते हैं एवं उच्च न्यायपालिका द्वारा बड़े पैमाने पर फैसले देने में उपयोग किए गए हैं।

(एक बहुत ही सीधा एवं सरल प्रश्न)

73.निम्नलिखित कथनों पर विचार कीजिए :

  1. भारत का संविधान अपने ‘मूल ढाँचे’ को संघवाद, पंथनिरपेक्षता, मूल अधिकारों तथा लोकतंत्र के रूप में परिभाषित करता है
  2. भारत का संविधान, नागरिकों की स्वतंत्रता तथा उन आदर्शों जिन पर संविधान आधारित है, की सुरक्षा हेतु ‘न्यायिक पुनरवलोकन’ की व्यवस्था करता है।

उपर्युक्त कथनों में से कौन-सा/से सही है/हैं?

  1. केवल 1
  2. केवल 2
  3. 1 और 2 दोनो
  4. न तो 1, न ही 2

उत्तर (b). केवल कथन 2 सही है, 1 नहीं। बुनियादी ढाँचा सिद्धांत एक भारतीय न्यायिक सिद्धांत है जो बहुत बाद में विकसित हुआ, विशेष रूप से न्यायमूर्ति हंस राज खन्ना द्वारा प्रतिपादित, कि भारत के संविधान में कुछ बुनियादी विशेषताएं हैं, जिन्हें भारत की संसद द्वारा संशोधनों के माध्यम से परिवर्तित या नष्ट नहीं किया जा सकता है। ‘मूल’ के रूप में करार दी गई कुछ विशेषताएं हैं - संविधान की सर्वोच्चता, कानून का नियम, शक्तियों का पृथक्करण का सिद्धांत, प्रस्तावना में भारत के संविधान में निर्दिष्ट उद्देश्य, न्यायिक समीक्षा, अनुच्छेद 32 एवं 226, संघीयवाद (वित्तीय स्वतंत्रता सहित) अनुच्छेद 282 एवं 293 के तहत राज्य), धर्मनिरपेक्षता, संप्रभुता, लोकतांत्रिक, गणतंत्रीय संरचना, व्यक्ति की स्वतंत्रता एवं गरिमा, राष्ट्र की एकता एवं अखंडता, समानता का सिद्धांत, समानता की प्रत्येक विशेषता नहीं है, लेकिन समान न्याय की सर्वोत्कृष्टता है, भाग III में लिखीत अन्य मौलिक अधिकारों, न्याय के लिए प्रभावी पहुंच, अनुच्छेद 32, 136, 141, 142 के तहत सर्वोच्च न्यायालय की शक्तियां एवं कानून, जो एक अधिनियम के तहत बनाए गए मध्यस्थता न्यायाधिकरण द्वारा गठित राज्य की न्यायिक शक्ति के अभ्यास में किए गए आदेशों को रद्द करने की शक्ति रखता है। इन्हें संविधान में मूल संरचना नहीं कहा जाता है या कहा जाता है, इसलिए कथन 1 गलत है।

न्यायिक समीक्षा को नागरिक व्यक्तियों की स्वतंत्रता एवं अधिकारों की रक्षा के लिए एक सभ्यता की आवश्यक एवं बुनियादी आवश्यकता के रूप में मान्यता प्राप्त है। न्यायिक समीक्षा की शक्ति उच्च न्यायालयों एवं भारत के सर्वोच्च न्यायालय में निहित है। भारतीय संविधान के अनुच्छेद 13 (मौलिक अधिकार, भाग III) के तहत, न्यायिक समीक्षा की बाध्यता स्पष्ट हो जाती है।

अनुच्छेद 13 - (2) - राज्य इस भाग द्वारा प्रदत्त अधिकारों को किसी भी तरह का कानून नहीं बना सकता है एवं न ही निरस्त कर सकता है एवं इस खंड के उल्लंघन में बनाया गया कोई भी कानून, उल्लंघन की सीमा तक, शून्य हो जाएगा।

न्यायिक समीक्षा की शक्तियां संविधान के अनुच्छेद 226 एवं 227 में शामिल हैं क्योंकि उच्च न्यायालय संबंधित हैं, एवं अनुच्छेद 32 एवं 136 जहां तक सर्वोच्च न्यायालय का संबंध है।

(एक कठिन प्रश्न क्योंकि कथन 1 को भ्रामक तरीके से लिखा गया है)

74.गाँधीवाद और मार्क्सवाद के बीच एक समान सहमति पाई जाती है। यह निम्नलिखित में से कौन-सी है ?

  1. एक अंतिम लक्ष्य के रूप में राज्यविहीन समाज
  2. वर्ग संघर्ष
  3. निजी संपत्ति की समाप्ति
  4. आर्थिक नियतिवाद

उत्तर (a). आदर्श समाज को लेकर गाँधी एवं र्माक्स के विचारों में समानता। दोनों का अंतिम उद्देश्य एक राज्य एवं वर्गहीन समाज की स्थापना था, लेकिन इस उद्देश्य को प्राप्त करने के साधन दोनों के लिए अलग-अलग थे। महात्मा गांधी अहिंसा के माध्यम से इसे हासिल करना चाहते थे लेकिन र्माक्स ने भविष्यवाणी की कि यह एक गंभीर वर्ग संघर्ष के बिना संभव नहीं हो सकता। लगभग सभी अन्य क्षेत्रों में, उनके विचार मेल नहीं खाते थे।

आर्थिक नियतिवाद एक सामाजिक आर्थिक सिद्धांत है जिसके अनुसार आर्थिक संबंध वह आधार है जिस पर समाज में अन्य सामाजिक एवं राजनीतिक व्यवस्था आधारित है। गांधी के लिए, आध्यात्मिकता एवं स्वयं की खोज ने एक प्रमुख भूमिका निभाई।

गांधी ने लिखा - “पश्चिम का समाजवाद एवं साम्यवाद कुछ विशिष्ट धारणाओं पर आधारित है जो मूल रूप से हमसे अलग हैं। इस तरह की एक धारणा मानव प्रकृति के आवश्यक स्वार्थ में उनका विश्वास है। मैं इससे सहमत नहीं क्योंकि मुझे पता है कि आदमी एवं जानवर के बीच आवश्यक अंतर यह है कि मनुष्य अंतरआत्मा की आवाज सुन सकता है। इसलिए, हमारे समाजवाद या साम्यवाद को अहिंसा एवं श्रम एवं पूंजी, जमींदार एवं किरायेदार के सामंजस्यपूर्ण सहयोग पर आधारित होना चाहिए।’

एवं उन्होंने लिखा - “मुझे यह स्वीकार करना चाहिए कि मैं अभी तक बोल्शेविज्म के अर्थ को पूरी तरह से समझ नहीं पाया हूँ। यह सब मुझे पता है कि इसका उद्देश्य निजी संपत्ति के संस्थान को खत्म करना है। यह अर्थशास्त्र के दायरे में गैर-कब्जे के नैतिक आदर्श का प्रयोग लगता है एवं यदि लोगों इसे अपने आदर्शों के बल पर शांतिपूर्ण अनुनय के माध्यम से स्वीकार करे तो इससे अच्छा और कुछ नही। लेकिन मुझे जो बोल्शेविज्म के बारे में पता है, वह न केवल बल के उपयोग को नही रोकता, बल्कि निजी संपत्ति के निष्कासन एवं उसी के सामूहिक राज्य के स्वामित्व को बनाए रखने के लिए स्वतंत्र रूप से उसके उपयोग का भी हिमायती है।’

(एक कठिन प्रश्न लेकिन कुछ तर्क के साथ हल किया जा सकता है)

75.भारत के संदर्भ में, नौकरशाही का निम्नलिखित में से कौन-सा उपयुक्त चरित्र-चित्रण है ?

  1. संसदीय लोकतंत्र की व्याप्ति को विस्तार देने वाले अभिकरण।
  2. संघीय ढाँचे को सुदृढ़ करने वाला अभिकरण
  3. राजनीतिक स्थायित्व और आर्थिक वृद्धि के सुलभ बनाने वाला अभिकरण
  4. लोक नीति को कार्यान्वित करने वाला अभिरण

उत्तर (d). सिविल सेवा या नौकरशाही उन पेशेवरों का गठन करती है जो स्थायी एवं वेतनभोगी कर्मचारी हैं जो सरकार के कार्यकारी अंग का हिस्सा हैं। वे राजनीतिक रूप से तटस्थ होते हैं एवं उनका मुख्य काम विभिन्न सरकारी विभागों एवं नीति कार्यान्वयन को सुविधाजनक बनाना है। हालांकि, वे मंत्रियों के नियंत्रण एवं नेतृत्व में काम करते हैं।

निर्वाचित सरकार के किसी नीति पर निर्णय लेने के बाद उन नीतियों को लागू करवाना नौकरशाही की जिम्मेदारी एवं कर्तव्य है। नौकरशाही की अन्य प्रमुख जिम्मेदारियां हैं, दिन प्रतिदिन का प्रशासन प्रभावी ढंग से चलाना, नीतिगत निर्णयों पर राजनीतिक अधिकारियों को सलाह देना, कानूनों के निर्माण में अप्रत्यक्ष समर्थन देना, सरकार का वित्तीय प्रशासन संभालना, आम जनता के साथ संबंध बनाना एवं सरकारी रिकॉर्ड बनाए रखना। इसके अलावा, नौकरशाही को गैरकानूनी आदेशों को मानने से इंकार करना चाहिए, जो स्थापित कानून एवं संविधान की भावना के विपरीत है।

(a), (b) एवं (c) को राजनीतिक लहजे से देखते हुए कुछ तर्क लागू करें)


76.भारत के संविधान की उद्देशिका

  1. संविधान का भाग है किंतु कोई विधिक प्रभाव नहीं रखती।
  2. संविधान का भाग नहीं है और कोई विधिक प्रभाव भी नहीं रखती।
  3. संविधान का भाग है और वैसा ही विधिक प्रभाव रखती है जैसा कि उसका कोई अन्य भाग
  4. संविधान का भाग है किंतु उसके अन्य भागों से स्वतंत्र होकर उसका कोई विधिक प्रभाव नहीं है।

उत्तर (d). एक उद्धेशिका, किसी दस्तावेज का परिचयात्मक विवरण है जो दस्तावेज के दर्शन एवं उद्देश्यों को बताता है। संविधान में, यह अपने निर्माणकर्ताओं, अपनी रचना के पीछे के इतिहास एवं राष्ट्र के मूल मूल्यों एवं सिद्धांतों को प्रस्तुत करती है। प्रस्तावना मूल रूप से संविधान के स्रोत, भारतीय राज्य की प्रकृति, उसके उद्देश्यों का विवरण एवं उसके लागु होने की तारीख का विचार देती है।

हालांकि अदालत में इसे चुनौती नहीं दी जा सकती है लेकिन उद्धेशिका संविधान के उद्देश्यों को बताता है, एवं जब भाषा अस्पष्ट पाई जाती है, अतः लेख की व्याख्या के दौरान सहायता के रूप में कार्य करता है। संविधान के एक भाग के रूप में प्रस्तावना पर सर्वोच्च न्यायालय में कई बार चर्चा हुई। इसका उपयोग बेरुबरी केस एवं केशवानंद भारती केस में एक संदर्भ के रूप में किया गया था। इसलिए, यह निष्कर्ष निकाला जा सकता है कि उद्धेशिका संविधान के परिचयात्मक भाग का हिस्सा है।

एलआईसी आॅफ इंडिया बनाम केंद्र सरकार, 1995 के मामले में, सर्वोच्च न्यायालय ने एक बार फिर माना है कि उद्धेशिका संविधान का अभिन्न अंग है लेकिन भारत में न्याय की अदालत में प्रत्यक्ष रूप से लागू नहीं है।

(a) एवं (d) के बीच का अंतर यह है कि (a) स्पष्ट रूप से उद्धेशिका को शक्तिहीन बनाता है, लेकिन (d) संविधान के अन्य वर्गों के साथ जुड़ने पर इसके महत्व को समझाता है)

77."स्वर्ण-ट्रान्श" (रिज़र्व ट्रान्श) निर्दिष्ट करता है

  1. विश्व बैंक की ऋण व्यवस्था
  2. केन्द्रीय बैंक की किसी एक क्रिया को
  3. WTO द्वारा इसके सदस्यों को प्रदत्त एक साख प्रणाली
  4. IMF द्वारा इसके सदस्यों को प्रदत्त एक साख प्रणाली को

उत्तर (d). स्वर्ण-ट्रांश - सोने की वह मात्रा जो अंतर्राष्ट्रीय मुद्रा कोष (आईएमएफ) का प्रत्येक सदस्य देश निधि के लिए अपनी सदस्यता दायित्वों के हिस्से के रूप में जमा कराता है, एवं आर्थिक कठिनाइयों के समय इसके आधार पर आसानी से उधार ले सकता है।

1973 में ब्रेटन वुड्स की फिक्स्ड एक्सचेंज दरों की प्रणाली के टूटने तक गोल्ड ने अंतरराष्ट्रीय मौद्रिक प्रणाली में एक केंद्रीय भूमिका निभाई। तब से, इसकी भूमिका कम हो गई है। लेकिन यह कई देशों के रिजर्व होल्डिंग्स में एक महत्वपूर्ण संपत्ति है, एवं आईएमएफ अभी भी सोने की दुनिया के सबसे बड़े आधिकारिक धारकों में से एक है। अप्रैल 2008 में निधि के लिए नए आय मॉडल के अनुरूप, सीमित बिक्री के लिए सीमित सोने की बिक्री से लाभ का इस्तेमाल किया गया था एवं इसका इस्तेमाल आईएमएफ की रियायती ऋण देने की क्षमता को कम आय वाले देशों (एलआईसी) में बढ़ावा देने के लिए किया गया था।

रिर्जव-ट्रांश आईएमएफ के साथ किसी सदस्य देश के कोटे का एक घटक है जो सोने या विदेशी मुद्रा के रूप में है। किसी भी सदस्य देश के लिए, कुल कोटा में से, 25% का भुगतान विदेशी मुद्रा या सोने के रूप में किया जाना चाहिए। इसलिए इसे रिर्जव-ट्रांश या स्वर्ण-ट्रांश कहा जाता है। शेष 75% घरेलू मुद्राओं में हो सकता है एवं इसे क्रेडिट-ट्रांश कहते हैं।

(आपको यह जानने की जरूरत है कि यह आईएमएफ प्रणाली है।)

78.भारत के संविधान के भाग IV में अंतर्विष्ट प्रावधान के संदर्भ में, निम्नलिखित कथनों में से कौन-सा/से सही है/हैं ?

  1. वे न्यायालयों द्वारा प्रवर्तनीय होंगे।
  2. वे किसी भी न्यायालय द्वारा प्रवर्तनीय नहीं होंगे।
  3. इस भाग में अधिकथित सिद्धान्त राज्य के द्वारा कानून बनाने को प्रभावित करेंगे।

नीचे दिए गए कूट का प्रयोग कर सही उत्तर चुनिए :

  1. केवल 1
  2. केवल 2
  3. केवल 1 और 3
  4. केवल 2 और 3

उत्तर (d). भारतीय संविधान में भाग प्ट राज्य नीति के निर्देशक सिद्धांतों से संबंधित है। ये भारत के राज्य को संचालित करने वाले संघीय संस्थानों को दिए गए दिशा-निर्देश या सिद्धांत हैं, जिन्हें कानूनों एवं नीतियों को बनाते समय ध्यान में रखा जाना चाहिए। अतः, 3 सही है, एवं इसलिए, विकल्प (a) एवं (b) गलत हैं।

भारत के संविधान के भाग IV में निहित ये प्रावधान किसी भी अदालत द्वारा प्रत्यक्ष रूप से लागू करने योग्य नहीं हैं (इसलिए 1 भी गलत है, इसलिए विकल्प (b) भी गलत है), लेकिन सिद्धांतों को भारत के शासन में महत्वपूर्ण मार्गदर्शक पद माना जाता है, कानून बनाते समय इन्हें लागू करना राज्य का कर्तव्य है। इनका उद्देश्य भारत में न्यायपूर्ण समाज की स्थापना करना है। उत्तर (d) - केवल 2 एवं 3, है।

(यदि आप जानते हैं कि 1 गलत है, अतः (a) एवं (c) समाप्त हो जाते हैं। 3 निश्चित रूप से सही है, उत्तर होना चाहिए (d)

79.निम्नलिखित कथनों पर विचार कीजिए :

  1. भारत के संविधान के अनुसार, कोई भी ऐसा व्यक्ति जो मतदान के लिए योग्य है, किसी राज्य में छह माह के लिए मंत्री बनाया जा सकता है तब भी जब कि वह उस राज्य के विधान-मंडल का सदस्य नहीं है।
  2. लोक प्रतिनिधित्व अधिनियम, 1951 के अनुसार, कोई भी ऐसा व्यक्ति जो दांडिक अपराध के अन्तर्गत दोषी पाया गया है और जिसे पाँच वर्ष के लिए कारावास का दंड दिया गया है, चुनाव लड़ने के लिए स्थायी तौर पर निरर्हत हो जाता है भले ही वह कारावास से मुक्त हो चुका हो।

उपर्युक्त कथनों में से कौन-सा/से सही है/हैं?

  1. केवल 1
  2. केवल 2
  3. 1 और 2 दोनो
  4. न तो 1, न ही 2

. उत्तर (d). 1 एवं 2 दोनों गलत हैं। भारतीय गणराज्य में, किसी व्यक्ति को विधान सभा का सदस्य बनने के लिए 25 वर्ष से कम एवं विधान परिषद का सदस्य होने के लिए 30 वर्ष से कम नहीं होना चाहिए (भारतीय संविधान के अनुच्छेद 173 के अनुसार)। मतदान की आयु 18 वर्ष है। इसलिए, मतदान करने के योग्य व्यक्ति को 25 वर्ष से कम आयु का होने पर मंत्री नहीं बनाया जा सकता है। बाकी कथन हमारी चर्चा के लिए अप्रासंगिक है।

भाग II के अनुसार, जनप्रतिनिधित्व अधिनियम, 1951 के पैरा 8 के अनुसार, एक व्यक्ति को चुनाव लड़ने से अयोग्य घोषित किया जाएगा, यदि दोषी व्यक्ति को -

  1. केवल जुर्माना किया गया हो, इस तरह की सजा की तारीख से छह साल की अवधि के लिए,
  2. कारावास दिया गया हो, इस तरह की सजा की तारीख से और उसकी रिहाई के बाद से छह साल की अवधि के लिए अयोग्य घोषित किया जाएगा।

अब तक, आपराधिक सजा के आधार पर, भारतीय चुनावों में ‘स्थायी अयोग्यता’ की कोई अवधारणा नहीं है।

(यदि आप ‘वोट देने योग्य’ और ‘स्थायी अयोग्यता’ टैग को तुरंत देख सकते हैं, तो सवाल आसान हो जाता है)

स्त्रोत http://legislative.gov.in/sites/default/files/04_representation%20of%20the%20people%20act%2C%201951.pdf

80.निम्नलिखित कथनों पर विचार कीजिए :

  1. भारत का राष्ट्रपति ऐसे स्थान पर जिसे वह ठीक समझे, संसद का सत्र आहूत (आव्हान) कर सकता है ।
  2. भारत का संविधान एक वर्ष में संसद के तीन सत्रों का प्रावधान करता है, किंतु सभी तीन सत्रों का चलाया जाना अनिवार्य नहीं है।
  3. एक वर्ष में दिनों की कोई न्यूनतम संख्या निर्धारित नहीं है, जब संसद का चलना आवश्यक हो।

उपर्युक्त कथनों में से कौन-सा/से सही है/हैं?

  1. केवल 1
  2. केवल 2
  3. केवल 1 और 2
  4. केवल 2 और 3

उत्तर (c). संविधान का अनुच्छेद 85 (1) राष्ट्रपति को संसद के प्रत्येक सदन को ऐसे समय एवं स्थान पर बैठक के लिए आह्वान का अधिकार देता है, लेकिन इसके दो सत्रों के मध्य 6 माह से अधिक का अंतर नही हो चाहिए। अतः, 1 सही है। इसलिए विकल्प (b) एवं (d) गलत हैं। अनुच्छेद 85 के लिए संसद के दो सत्रों के बीच छह महीने से अधिक का अंतर नहीं होना चाहिए। जिस अवधि के दौरान सदन अपने व्यवसाय का संचालन करने के लिए बैठक करता है उसे सत्र कहा जाता है। भारत में आमतौर पर बजट सत्र (जनवरी/फरवरी से मई), मानसून सत्र (जुलाई से अगस्त/सितंबर) एवं शीतकालीन सत्र (नवंबर से दिसंबर) तीन सत्र होते हैं। लेकिन संसद के तीन सत्रों का कोई विशेष उल्लेख नहीं है, इसलिए 2 गलत है।

हाँ। संसद में एक वर्ष में मिलने के लिए न्यूनतम दिनों की कोई संख्या नहीं है - वास्तव में, उन दिनों की संख्या जिनमें संसद मिलती है, वर्षों से कम हो रही है। संसद के पहले दो दशकों के दौरान, लोकसभा साल में औसतन 120 से अधिक दिनों के लिए मिलती थी। यह संख्या पिछले दशक में लगभग 70 दिनों के लिए नीचे आ गया है। अतः 3 सही है। अतः विकल्प (c) सही है।

अनुच्छेद 85 -

संसद के सत्र, विप्लव एवं विघटन

  1. राष्ट्रपति समय-समय पर संसद के प्रत्येक सदन को ऐसे समय एवं स्थान पर मिलने के लिए बुलाएगा, जैसा वह उचित समझता हो, लेकिन छह महीने उसके एक सत्र में बैठने एवं उसके पहले बैठने के लिए नियुक्त तारीख के बीच हस्तक्षेप नहीं करेगा। अगले सत्र
  2. राष्ट्रपति समय-समय पर हो सकता है
  3. सदनों या सदन को फिर से लागू करना
  4. लोक सभा को भंग करना

(चूंकि 2 निश्चित रूप से गलत है, दो विकल्पों – (b) एवं (d) को हटाया जा सकता है )

81.कार्बन नैनोट्यूबों के संदर्भ में, निम्नलिखित कथनों पर विचार कीजिए :

  1. इनको मानव शरीर में औषधियों और प्रतिजनों के वाहकों के रूप में प्रयुक्त किया जा सकता है।
  2. इनको मानव शरीर के क्षतिग्रस्त भाग के लिए कृत्रिम रक्त कोशिकाओं के रूप में बनाया जा सकता है।
  3. इनका जैव-रासायनिक संवेदकों में उपयोग किया जा सकता है।
  4. कार्बन नैनोट्यूब जैव-निम्नीकरणीय (biodegradable) होती है।

उपर्युक्त कथनों में से कौन-सा/से सही है/हैं?

  1. (b) केवल 1 और 2
  2. केवल 2, 3 और 4
  3. केवल 1, 3 और 4
  4. 1, 2, 3 और 4

उत्तर (d). 1, 2, 3 एवं 4 के सभी कथन सत्य हैं। कार्बन नैनोट्यूब (CNT) कार्बन के एलोट्रोप हैं, जो ग्रेफाइट से बने होते हैं एवं व्यास में नैनोमीटर एवं लंबाई में कई मिलीमीटर के साथ बेलनाकार ट्यूबों के रूप् निर्मित होते हैं। उनके प्रभावशाली संरचनात्मक, यांत्रिक एवं इलेक्ट्रॉनिक गुण उनके छोटे आकार एवं द्रव्यमान, उनके मजबूत यांत्रिक शक्ति एवं उनके उच्च विद्युत एवं तापीय चालकता के कारण हैं। CNTs को उनके उच्च सतह क्षेत्र के कारण फार्मेसी एवं चिकित्सा में सफलतापूर्वक लागू किया गया है जो कई प्रकार के चिकित्सीय एवं नैदानिक एजेंटों (ड्रग्स, जीन, टीके, एंटीबॉडी, जैवसंवेदको, आदि) के साथ संयोजन करने में सक्षम है। वे शरीर द्वारा चयापचय के बिना कोशिकाओं में सीधे दवा वितरण के लिए एक उत्कृष्ट वाहन साबित हुए हैं। तब ब्छज् के अन्य अनुप्रयोगों को न केवल दवा एवं जीन उपचारों के लिए, बल्कि ऊतक पुनर्जनन, जैवसंवेदको, दवाओं एवं प्रदूषकों के निष्कर्षण एवं विश्लेषण के लिए भी किया जाता है। कार्बन नैनोट्यूब कई अन्य सामग्रियों जैसे ऊतक पुर्नरचना के लिए प्राकृतिक एवं सिंथेटिक पॉलिमर में सबसे अच्छा ऊतक इंजीनियरिंग उम्मीदवार हो सकता है क्योंकि यह नैनोमटेरस जैवसंश्लेषण योग्य है, एवं अंग पुनर्जनन को बढ़ाने के लिए बायोमोलेक्यूल्स के साथ कार्यात्मक हो सकता है। यह भी प्रदर्शित किया गया है कि कार्यात्मक कार्बन नैनोट्यूब को ऑक्सीडेटिव एंजाइम द्वारा जैव निम्नीकृत किया जा सकता है। इसलिए, सभी कथन सही हैं।

(विकल्प देखें - कथन 1 निश्चित रूप से सही है, इसलिए विकल्प (इद्ध समाप्त हो जाता है (केवल 2, 3 एवं 4)। फिर, कथन 2 सही है, इसलिए विकल्प (c) भी समाप्त हो जाता है (केवल 1, 3 एवं 4)। सही उत्तर विकल्प (a) या (d) में से कुछ होगा। अगर हमें 3 या 4 में से कोई एक सही मिल सकता है, तो उत्तर (d) होना चाहिए।

82.निम्नलिखित गतिविधियों पर विचार कीजिए :

  1. खेत में फसल पर पीड़कनाशी छिड़कना
  2. सक्रिय ज्वालामुखियों के मुखों का निरीक्षण करना
  3. डी.एन.ए. विश्लेषण के लिए उत्क्षेपण करती हुई व्हेलो के श्वास के नमूने एकत्र करना।

तकनीकी के वर्तमान स्तर पर, उपर्युक्त गतिविधियों में से किसे, ड्रोन के प्रयोग से सफलतापूर्वक किया जा सकता है ?

  1. केवल 1 और 2
  2. केवल 2 और 3
  3. केवल 1 और 3
  4. 1, 2, और 3

उत्तर (d). सभी कथन 1, 2 एवं 3 सही हैं। प्रौद्योगिकी के वर्तमान स्तर (2020) में, तेलंगाना राज्य के विभिन्न हिस्सों से रिपोर्ट किए गए कृषि क्षेत्रों पर कीटनाशक का छिड़काव करने के लिए ड्रोन का व्यापक उपयोग किया गया है। कृषि एवं किसान कल्याण मंत्रालय ने कहा कि यह 1968 के कीटनाशक अधिनियम के अनुसार अवैध था।

पोट्सडैम में जर्मन रिसर्च सेंटर फॉर जिओसाइंस जीएफजेड के एडगर जोर्न के शोधकर्ता अब ग्वाटेमाला के सांता मारिया ज्वालामुखी में ऑप्टिकल एवं थर्मल इमेजिंग कैमरों के साथ बार-बार सर्वेक्षण उड़ानों की एक श्रृंखला के परिणाम पेश कर रहे हैं। ड्रोन का उपयोग लावा गुंबद, लावा के एक चिपचिपे प्लग का निरीक्षण करने के लिए किया गया था। अध्ययन ‘साईटीफिक रिपोर्ट’पत्रिका में प्रकाशित हुआ था।

पानी की सतह के पास तैरने वाली व्हेल एवं डॉल्फिन का फिल्मांकन करने के लिए वर्षों से ड्रोन का उपयोग किया जाता रहा है, लेकिन वुड्स होल ओशनोग्राफिक इंस्टीट्यूट (WHOI) द्वारा किए गए एक नए अध्ययन में, शोधकर्ताओं ने एक मानवरहित हवाई वाहन का इस्तेमाल किया। उन्होंने, इससे मैसाचुसेट्स के तट से स्टेलवगेन बैंक नेशनल मरीन सैंक्चुअरी में हम्पबैक व्हेल से सांस के नमूने एकत्र किए थे।

स्त्रोत - https://www.futurefarming.com/Tools-data/Articles/2018/9/Drone-spraying-and-spreading-becoming-reality-335322E/

https://www.sciencedaily.com/releases/2020/05/200525115649.htm

https://www.cnet.com/news/drone-flies-through-whales-spouts-to-sample-their-breath/

83."यह प्रयोग तीन ऐसे अंतरिक्षयानों को काम में लाएगा जो एक समबहु त्रिभुज की आकृति में उड़ान भरेंगे, जिसमें प्रत्येक भुजा एक मिलियन किलोमीटर लम्बी है और यानों के बीच लेज़र चमक रहीं होंगी।" कथित प्रयोग किसे संदर्भित करता है ?

  1. वॉयेजर-2
  2. न्यू हॉरायज़न्स
  3. LISA पाथफाइंडर
  4. इवोल्वड LISA

उत्तर (d). ईवोल्वड लेजर इंटरफेरमोटर स्पेस एंटीना (ईवोल्वड LISA) पहली बार अंतरिक्ष से गुरुत्वाकर्षण ब्रह्मांड की खोज करने के उद्देश्य से एक मिशन है। एलिसा मिशन में एक ‘मदर’ एवं दो ‘डॉटर’ स्पेसक्राफ्ट शामिल हैं। ये सूर्य की त्रिकोणीय विन्यास में परिक्रमा करेंगे, जैसा कि चित्र में दिखाया गया है। तीन उपग्रह एक इंटरफेरोमीटर का निर्माण करेंगे, जिसमें दो डॉटर स्पेसक्राफ्ट एक लाख किमी लंबे लेजर बीम से मदर से जुड़े होंगे। यह इंटरफेरोमीटर 0.1 मेगाहर्ट्ज से 1 हर्ट्ज तक की आवृत्ति में गुरुत्वाकर्षण तरंगों का पता लगाने में सक्षम होगा। इस तरह की आवृत्ति अंतराल हाथ की लंबाई की सीमाओं के कारण एवं स्थलीय गुरुत्वाकर्षण ढाल शोर के कारण पृथ्वी पर सुलभ नहीं है - इस अर्थ में, eLISA। जमीन आधारित गुरुत्वाकर्षण-तरंग डिटेक्टरों के प्रयासों का पूरक होगा।

स्त्रोत - http://blogs.cardiff.ac.uk/physicsoutreach/gravitational-physics-tutorial/evolved-laser-interferometer-space-antenna-elisa/

(विकल्प देखें - विकल्प (a) निश्चित रूप से गलत है, क्योंकि हम जानते हैं कि वायेजर -2 गहरे ब्रह्मांड की खोज के लिए एक अंतरिक्ष यान था एवं पहले से ही सौर प्रणाली को छोड़ चुका है। न्यू होराइजन्स का विकल्प ;इद्ध प्लूटो की खोज के लिए था। दोनों इंटरप्लेनेटरी स्पेसक्राफ्ट हैं। अंतिम विकल्प केवल (c) एवं (d) के बीच में से होगा।)

84.निम्नलिखित कथनों पर विचार कीजिए :

  1. भावी माता-पिता के अंड या शुक्राणु उत्पन्न करने वाली कोशिकाओं में आनुवंशिक परिवर्तन किए जा सकते हैं।
  2. व्यक्ति का जीनोम जन्म से पूर्व प्रारंभिक भ्रूणीय अवस्था में सम्पादित किया जा सकता है।
  3. मानव प्रेरित बहुशक्त स्टेम (pluripotent stem) कोशिकाओं को एक शूकर के भ्रूण में अंतर्वेशित किया जा सकता है।

उपर्युक्त कथनो में से कौन-सा/से सही है/हैं ?

  1. केवल 1
  2. केवल 2 और 3
  3. केवल 2
  4. 1, 2 और 3

उत्तर (d). सभी कथन 1, 2 एवं 3 सही हैं। कथन 1 सही है। अंडे एवं शुक्राणु क्रमशः रोगाणु कोशिकाओं अंडाशय एवं वृषण द्वारा निर्मित होते हैं। इन रोगाणु कोशिकाओं में आनुवांशिक परिवर्तन, किसी भी अन्य दैहिक कोशिका की तरह, असंख्य तकनीकों द्वारा किया जा सकता हैं, उनमें अन्य सहायक तकनीकों जैसे विकिरण, रासायनिक या कुछ जैविक एजेंट (जो अक्सर अवांछनीय परिवर्तन पैदा करते हैं) के अलावा सबसे अधिक प्रचलित जैव प्रौद्योगिकी है।

कथन 2 सही है। चीनी शोधकर्ताओं ने पहली बार एक मानव भ्रुण के जीन को 2015 में एक प्रयोगशाला (विवादास्पद मामले) में संपादित किया। अमेरिका में वैज्ञानिकों ने प्रारंभिक चरण के मानव भ्रुण के डीएनए में रोग पैदा करने वाले उत्परिवर्तन की मरम्मत की है, जो विरासत में मिली बीमारियों से मुक्त इंजीनियरिंग शिशुओं के करीब एक कदम है।

कथन 3 भी सही है। मानव प्रेरित बहुशक्त स्टेम (आईपीएस) कोशिकाएं पुनर्योजी चिकित्सा के लिए महान चिकित्सीय वादा रखती हैं। मानव आईपीएस कोशिकाएं अनिश्चित काल तक फैल सकती हैं एवं एक वयस्क शरीर में सभी प्रकार की कोशिकाओं में अंतर कर सकती हैं। हालांकि, मनुष्यों पर उपयोग करने से पहले, पशु मॉडल के साथ आईपीएस सेल-आधारित चिकित्सा की सुरक्षा एवं प्रभावकारिता का परीक्षण करना अनिवार्य है। आसान पहुंच के कारण, कम लागत, एवं उपलब्ध आनुवंशिक एवं आणविक उपकरण, कृन्तकों की एक श्रृंखला, विशेष रूप से चूहों में, पूर्व-नैदानिक परीक्षणों के लिए सबसे लोकप्रिय पशु मॉडल रहा है। हालांकि, कई उदाहरणों में, कृंतक मॉडल विकास एवं शरीर विज्ञान में महत्वपूर्ण अंतर के कारण मानव स्थितियों को सही ढंग से प्रतिबिंबित नहीं कर सकते हैं। शुकर, अंग आकार, शरीर विज्ञान एवं शरीर रचना विज्ञान में कृन्तकों की तुलना में मनुष्यों के समान हैं, एवं इस प्रकार सुअर आईपीएस सेल डेरिवेटिव का उपयोग करके ऑटोलॉगस एवं/या होमोलॉगस प्रत्यारोपण पुनर्योजी चिकित्सा के लिए एक बेहतर मॉडल का प्रतिनिधित्व करते है। इसके अलावा, अंर्तप्रजातिय ब्लास्टोसिस्ट पूरकता के विकास के साथ, शुकरों में उत्पन्न मानव अंगों को भविष्य में प्रत्यारोपण के लिए दुनिया भर में मानव अंगों की कमी को हल करने में मदद मिल सकती है।

https://stemcellres.biomedcentral.com/articles/10.1186/s13287-019-1303-0

(विकल्पों को देखें - कथन 1 निश्चित रूप से सही है, एवं हाल के दिनों में कई बड़े अख़बारों की रिपोर्टिंग के कारण जाना जाता है। इसलिए विकल्प (b) एवं (c) से इंकार किया जाता है, एवं केवल (b) या (d) सही हो सकते है। यदि आप 2 या 3 में से किसी एक को सही साबित कर सकते हैं, अतः उत्तर विकल्प (d) होना चाहिए।)

85.भारत के न्यूमोकोकल संयुग्मी वैक्सीन (Pneumococcal Conjugate Vaccine) के उपयोग का क्या महत्व है ?

  1. ये वैक्सीन न्यूमोनिया और साथ ही तानिकाशोथ और सेप्सिस के विरूद्ध प्रभावी है।
  2. उन प्रतिजैविकायों पर निर्भरता कम की जा सकती हैजो औषध-प्रतिरोधी जीवाणुओं के विरूद्ध प्रभावी नहीं है।
  3. इन वैक्सीन के कोई गौण प्रभाव (side effects) नहीं है और न ही ये वैक्सीन कोई प्रत्यूर्जता सम्बन्धी अभिक्रियाएँ (allergic reactions) करती है।

नीचे दिए गए कूट का प्रयोग कर सही उत्तर चुनिए :

  1. केवल 1
  2. केवल 1 और 2
  3. केवल 3
  4. 1, 2 और 3

उत्तर (b). कथन 1 एवं 2 सही हैं, लेकिन 3 नहीं है। कथन 1 सही है। न्यूमोकोकल कंजुगेट वैक्सीन (पीसीवी) बच्चों एवं वयस्कों दोनों को न्यूमोकोकल बीमारी से बचा सकती है। न्यूमोकोकल बीमारी बैक्टीरिया के कारण होती है जो निकट संपर्क के माध्यम से एक व्यक्ति से दूसरे व्यक्ति में फैल सकती है। यह कान के संक्रमण का कारण बन सकता है, एवं यह फेफड़ों (निमोनिया), रक्त (जीवाणु), मस्तिष्क एवं रीढ़ की हड्डी को ढंकने (मैनिंजाइटिस) के एवं अधिक गंभीर संक्रमण (सेप्सिस) को जन्म दे सकता है।

कथन 2 सही है। दूसरी पीढ़ी के न्यूमोकोकल संयुग्मित टीके जैसे कि विस्तारित सीवीवी कवरेज जैसे कि पीसीवी 10 (10-वैलेंट) एवं पीसीवी 13 (13-वैलेंट) ने प्रत्यक्ष सुरक्षा में वृद्धि की है एवं इन टीकों से आच्छादित न्यूमोकॉकल सीरोटाइप को समान प्रभावकारिता के साथ कवर किया गया है। PCV7 की शुरुआत के बाद बनाया गया, PCV 13 ने एंटीबायोटिक का उपयोग भी कम कर दिया एवं समानांतर में एंटीबायोटिक गैर-अतिसंवेदनशील उपभेदों के प्रसार में भी कमी आई।

कथन 3 सही नहीं है। श्रृंखला में गंभीर गौण प्रभाव उम्र के अनुसार भिन्न होते हैं। बच्चों के बीच बताई गई सबसे आम समस्याएं उनींदापन, भूख की अस्थायी हानि, लालिमा या कोमलता थी, जहां टीका लगाया गया हैं वहाँ सूजन, हल्का या उच्च बुखार, चिड़चिड़ा व्यवहार इत्यादि था। वयस्कों ने दर्द, जहां टीका लगाया गया था वहाँ लालिमा एवं सूजन की सूचना दी है इसके अलावा हल्का बुखार, थकान, सिरदर्द, ठंड लगना या मांसपेशियों में दर्द अन्य प्रभाव थे।

स्त्रोत- http://www.cdc.gov/vaccinesafety/ https://www.ncbi.nlm.nih.gov/pmc/articles/PMC6183139/

(विकल्पों पर गौर करें - कथन 3 आपको अजीब लगता है - किसी टीके का कोई दुष्प्रभाव कैसे नही हो सकता है एवं कोई एलर्जी नहीं हो सकती है? यदि यह स्पष्ट है, अतः विकल्प (c) एवं (d) को सीधे खारिज कर दिया जाता है)

86. भारत में, "पब्लिक की इंफ्रास्ट्रक्चर" (Public Key Infrastructure) पदबंध किसके प्रसंग में प्रयुक्त किया जाता है ?

  1. डिजिटल सुरक्षा आधारभूत संरचना
  2. खाद्य सुरक्षा आधारभूत संरचना
  3. स्वास्थ्य देखभाल और शिक्षा आधारभूत संरचना
  4. दूरसंचार और परिवहन आधारभूत संरचना

उत्तर (a). पब्लिक की इन्फ्रास्ट्रक्चर (पीकेआई) डिजिटल दुनिया में उपयोगकर्ताओं एवं उपकरणों को प्रमाणित करने की एक तकनीक है। मूल विचार यह है कि एक या एक से अधिक विश्वसनीय पक्षों ने डिजिटल रूप से दस्तावेजों को यह प्रमाणित किया है कि एक विशेष क्रिप्टोग्राफिक कुंजी किसी विशेष उपयोगकर्ता या डिवाइस से संबंधित है। कुंजी को तब डिजिटल नेटवर्क में उपयोगकर्ता के लिए एक पहचान के रूप में उपयोग किया जा सकता है। जिन उपयोगकर्ताओं एवं उपकरणों के पास कुंजी होती है, उन्हें अक्सर केवल निकाय कहा जाता है। सामान्य तौर पर, कुछ भी एक कुंजी के साथ जुड़ा हो सकता है जिसे वह अपनी पहचान के रूप में उपयोग कर सकता है। एक उपयोगकर्ता या डिवाइस के अलावा, यह एक प्रोग्राम, प्रक्रिया, निर्माता, घटक या कुछ एवं हो सकता है। पीकेआई का उद्देश्य एक इकाई के साथ कुंजी को सुरक्षित रूप से जोड़ना है।

(डिजिटल इंडिया पर किए कवरेज को देखते हुए यह प्रश्न आसान था)

87.निम्नलिखित कथनों में से कौन-से पादप और प्राणि कोशिकाओं के बीच सामान्य अंतर के बारे में सही है ?

  1. पादप कोशिकाओं में प्लैज़्मा डिल्ली नहीं होती जबकि इसके विपरीत प्राणि कोशिकाओं में वे होती है।
  2. परिपक्व पादप कोशिका में एक वृहत् रसधानी होती है जबकि प्राणि कोशिका में अनेक छोटी रसधानियाँ होती है।

नीचे दिए गए कूट का प्रयोग कर सही उत्तर चुनिए :

  1. केवल 1 और 2
  2. केवल 2 और 3
  3. केवल 1 और 3
  4. 1, 2 और 3

उत्तर (c). कथन 1 सही है। पादप कोशिकाओं, फंगल कोशिकाओं एवं बैक्टीरिया कोशिकाओं में कोशिका भित्ति होती है। प्लांट सेल की दीवारें सेल्यूलोज से बनी होती हैं, एवं सेल की दीवार कोशिका झिल्ली को घेर लेती है। यह पादप कोशिका को अपनी विशिष्ट आयताकार आकृति प्रदान करता है। पशु कोशिकाओं में केवल एक कोशिका झिल्ली होती है, लेकिन कोशिका भित्ति नहीं होती है।

कथन 2 गलत है। सभी प्रकार की कोशिकाओं में प्लाज्मा मेम्ब्रेन (जिसे सेल मेम्ब्रेन भी कहा जाता है) है।

कथन 3 सत्य है। पादप कोशिकाओं में एक बड़ा, केंद्रीय रिक्तिका होता है जो कचरे को संग्रहीत करता है। पशु कोशिकाओं में या अतः रिक्तिकाएँ नहीं होती हैं या कई छोटे रिक्तिकाएँ होती हैं।

(यह एक आसान प्रश्न था यदि आप जानते थे कि 2 गलत था, अतः एक शॉट में तीन विकल्पों (a), (b) एवं (d) को खारिज किया जाता है)

88.निम्नलिखित में से कौन-से कारण/कारक बैंजीन प्रदूषण उत्पन्न करते हैं ?

  1. स्वचालित वाहन (automobile) द्वारा निष्कासित पदार्थ
  2. तम्बाकू का धुआँ
  3. लकड़ी का जलना
  4. रोगन किए गए लकड़ी के फर्नीचर का उपयोग
  5. पॉलियूरिथेन से निर्मित उतपादों का उपयोग

नीचे दिए गए कूट का प्रयोग कर सही उत्तर चुनिए :

  1. केवल 1, 2 और 3
  2. केवल 2 और 4
  3. केवल 1, 3, और 4
  4. 1, 2, 3, 4 और 5

उत्तर (d). सभी कथन सही हैं। बेंजीन युक्त हवा में सांस लेने से लोग बैंजीन से सर्वाधिक संपर्क में आते हैं। बेंजीन को गैसोलीन जैसे स्रोत के संपर्क के दौरान त्वचा के माध्यम से भी अवशोषित किया जा सकता है, लेकिन क्योंकि तरल बेंजीन जल्दी से वाष्पित हो जाता है, यह कम आम है। बेंजीन बनाने या उपयोग करने वाले उद्योगों में श्रमिकों को इस रसायन के संपर्क में लाया जा सकता है। इनमें रबर उद्योग, तेल शोधन, रासायनिक संयंत्र, जूता निर्माता एवं गैसोलीन से संबंधित उद्योग शामिल हैं। बेंजीन का उपयोग कुछ प्रकार के स्नेहक, रंजक, डिटर्जेंट, ड्रग्स एवं कीटनाशक बनाने के लिए भी किया जाता है। अन्य लोग जो काम पर बेंजीन के संपर्क में आ सकते हैं, उनमें इस्पात श्रमिक, प्रिंटर, लैब तकनीशियन, गैस स्टेशन के कर्मचारी एवं अग्निशामक शामिल हैं। संघीय नियम कार्यस्थल में बेंजीन के संपर्क को सीमित करते हैं। (नीचे देखें)।

लोगों को गैसोलीन के धुएं, ऑटोमोबाइल निकास, कुछ कारखानों से उत्सर्जन, एवं कुछ उद्योगों के अपशिष्ट जल से पर्यावरण में बेंजीन के संपर्क में लाया जा सकता है। बेंजीन आमतौर पर शहरी एवं ग्रामीण दोनों क्षेत्रों में हवा में पाया जाता है, लेकिन स्तर आमतौर पर बहुत कम हैं। गैसोलीन, गोंद, सॉल्वैंट्स, पेंट, एवं कला की आपूर्ति के साथ बंद जगहांे में लोगों के लिए इसका खतरा अधिक हो सकता है। भारी यातायात के क्षेत्र, गैस स्टेशन एवं औद्योगिक स्रोतों के पास के क्षेत्रों में भी उच्च वायु स्तर हो सकता है।

सिगरेट धूम्रपान एवं सेकेंड हैंड धूम्रपान बेंजीन के संपर्क के महत्वपूर्ण स्रोत हैं। संयुक्त राज्य अमेरिका में बेंजीन के संपर्क में सिगरेट के धुएं का लगभग आधा हिस्सा है। तंबाकू के धुएं वाले कमरों में बेंजीन का स्तर सामान्य से कई गुना अधिक हो सकता है।

लोगों को दूषित पेयजल एवं कुछ खाद्य पदार्थों में बेंजीन से अवगत कराया जा सकता है (हालांकि स्तर आमतौर पर बहुत कम हैं)। कई वाष्पशील कार्बनिक यौगिकों का मानव स्वास्थ्य एवं पर्यावरण पर विषाक्त प्रभाव पड़ता है। विशेष रूप से बेंजीन, लकड़ी जलाने से उत्सर्जित मुख्य कार्बनिक यौगिकों में से एक, कैंसर पैदा करने की क्षमता है।

पर्यावरण संरक्षण एजेंसी (ईपीए) के अनुसार, वार्निश फर्नीचर सहित कई अलग-अलग चीजें इनडोर वायु प्रदूषण का कारण बन सकती हैं, एवं हमारे स्वास्थ्य पर उनका संचयी प्रभाव पड़ता है।

पॉलीअरेथेनेस को एक आइसोसाइनेट एवं एक पॉलोल के बीच प्रतिक्रिया से बनाया जाता है, सभी बेंजीन के कुछ रूप पर निर्भर करते हैं।

स्त्रोत - https://www.hogarsintoxicos.org/en/riesgos/furniture-wood

(विकल्पों को देखें - यदि आप जानते हैं कि 5 सही है, अतः उत्तर सीधा है)

89.यदि निकट भविष्य में दूसरा वैश्विक वित्तीय संकट होता है, जो निम्नलिखित मेंसे कौन-से कार्य/नीतियाँ, भारत को, सबसे अधिक संभावना के साथ, कुछ उन्मुक्ति प्रदान कर सकती है/हैं ?

  1. अल्पकालीन विदेशी ऋणों पर निर्भर न रहना
  2. कुछ और विदेशी बैंको को प्रारंभ करना
  3. पूँजी खाते में पूर्ण परिवर्तनीयता को बनाए रखना।

नीचे दिए गए कूट का प्रयोग कर सही उत्तर चुनिए :

  1. केवल 1
  2. केवल 1 और 2
  3. केवल 3
  4. 1, 2 और 3

उत्तर (a). कथन 1 सही है। कथन 1 वास्तव में सही है। भारत अल्पकालिक विदेशी उधार पर निर्भर नहीं है, एवं इसने नए उभरते संकट में अचानक भुगतान करने के दबाव को समाप्त कर दिया। अतः 1 भारत के लिए अगले ग्लोबल फाइनेंशियल क्राइसिस (GFC) के दौरान अच्छा होगा यदि यह आता है। इसलिए विकल्प (c) गलत है।

कथन 2 विश्व अर्थव्यवस्था के साथ गहरे एकीकरण के कारण समस्याएं पैदा करेगा।

कथन 3 ‘पूर्ण पूंजी खाता परिवर्तनीयता’ दिखाता है। यह एक अच्छा विचार नहीं है क्योंकि इससे बड़ी पूंजी उड़ान (भारत में या भारत के बाहर) का बड़ा आंदोलन हो सकता है जो सिस्टम को अस्थिर कर सकता है।

(विकल्पों को देखें - यदि 2 गलत है, अतः ;इद्ध एवं ;कद्ध समाप्त हो जाते हैं। यदि 3 गलत है, तो (c) एवं (d) गलत हैं।)

90. यदि आप अपने बैंक के माँग जमा खाते (Demand Deposit Account) से रू. 1,00,000 की नकद राशि निकालते हैं, तो अर्थव्यवस्था में तात्कालिक रूप से मुद्रा की समग्र पूर्ति का इसका क्या प्रभाव पड़ेगा ?

  1. मुद्रा की समग्र पूर्ति में रू. 1,00,000 की कमी आएगी।
  2. मुद्रा की समग्र पूर्ति में रू. 1,00,000 की वृद्धि होगी।
  3. मुद्रा की समग्र पूर्ति में रू. 1,00,000 से अधिक वृद्धि होगी।
  4. मुद्रा की समग्र पूर्ति अपरिवर्तित रहेगी।

उत्तर (d). जैसे ही हम डिमांड डिपॉजिट से पैसा निकालते हैं, हम इसे अपने साथ ‘पब्लिक के साथ करेंसी’ के रूप में रखते हैं। अतः M1 नहीं बदलता है।

आरबीआई के अनुसार - ‘आर्थिक सिद्धांत में अवधारणा के रूप में या व्यवहार में मापा जाता है’ धन की कोई अनोखी परिभाषा नहीं है। धन भुगतान का एक साधन है एवं इस प्रकार एक स्नेहक जो विनिमय की सुविधा देता है। धन मूल्य के भंडार एवं खाते की इकाई के रूप में भी कार्य करता है। हालांकि, वास्तविक दुनिया में, धन मूर्त पारिश्रमिक के साथ-साथ मौद्रिक सेवाएं प्रदान करता है। यह इस कारण से है कि धन का उन गतिविधियों से संबंध होना है जो आर्थिक संस्थाएं अपनाती हैं। इसलिए, तरल वित्तीय परिसंपत्तियों के सेट के रूप में नीतिगत उद्देश्यों के लिए धन को परिभाषित किया जा सकता है, जिसके स्टॉक में भिन्नता समग्र आर्थिक गतिविधि पर प्रभाव डाल सकती है। सांख्यिकीय अवधारणा के रूप में, धन में वित्तीय मध्यस्थों या अन्य जारीकर्ताओं के एक विशेष सेट के कुछ तरल दायित्व शामिल हो सकते हैं। इस प्रकार, अन्य देशों की तरह, भारत में मौद्रिक एवं तरलता उपायों की एक श्रृंखला संकलित की जाती है।”

1977 में मनी सप्लाई (एसडब्ल्यूजी) पर दूसरे वर्किंग ग्रुप की सिफारिशों के बाद, आरबीआई ने रिजर्व मनी M0 के अलावा चार मौद्रिक समुच्चय - M, M2, M3 एवं M4 प्रकाशित किए।

M0 = RBI के साथ सर्कुलेशन बैंकर्स डिपॉजिट में करेंसी *RBI के साथ अन्य ‘डिपॉजिट’

M1 = बैंकिंग प्रणाली के साथ सार्वजनिक, डिमांड डिपॉजिट के साथ मुद्रा, RBI के साथ अन्य जमा

M2 = M1 + डाकघर बचत बैंकों के साथ बचत जमा

M3 = M1 + बैंकिंग प्रणाली के साथ समय जमा

M4 = M3 + राष्ट्रीय बचत प्रमाणपत्र को छोड़कर डाकघर बचत बैंकों के साथ सभी जमा

अतः M0 = मौद्रिक आधार, M1 = संकीर्ण धन, एवं M2 एवं M3 = व्यापक धन।

समुच्चय में से, M1 एवं M3 का बड़े पैमाने पर नीतिगत उद्देश्यों एवं शैक्षणिक अभ्यासों में उपयोग किया जाता है। जबकि M1 में सार्वजनिक क्षेत्र के साथ मुद्रा शामिल है, जो कि RBI सहित बैंकिंग क्षेत्र के साथ गैर-ब्याज असर जमा करता है, M3 बैंकिंग क्षेत्र की पूरी बैलेंस शीट पर कब्जा रखता है। M2 एवं M4 जिसमें डाकघर बचत बैंक जमा शामिल हैं बहुत व्यापक रूप से उपयोग नहीं किए जाते हैं।

भारतीय अर्थशास्त्री शब्द ‘मनी’, ‘मनी सप्लाई’ एवं ‘मनी स्टॉक’ का पर्यायवाची रूप से उपयोग करते हैं। RBI मौद्रिक समुच्चय के संकलन में धन आपूर्ति को मापने की सरल योग प्रक्रिया का अनुसरण कर रहा है। RBI अब M1, M2, M3 एवं M4 पर डेटा प्रकाशित करता है।

डिमांड डिपॉजिट एक बैंक या अन्य वित्तीय संस्थान के साथ एक खाता है जो जमाकर्ता को चेतावनी के बिना या सात दिनों से कम समय के नोटिस के साथ अपने धन को खाते से निकालने की अनुमति देता है। डिमांड डिपॉजिट ड1 मनी सप्लाई का एक प्रमुख घटक है। इसमें बचत एवं वर्तमान ए/सी शामिल हैं।

मुद्रा के लिए किसी भी बढ़ी हुई मांग संभवतः पैसे की आपूर्ति को अनुबंधित करने का कारण बनेगी क्योंकि मुद्रा को वापस लेने से मुद्रा कम हो जाती है, क्योंकि गुणक प्रभाव के कारण, वापस ली गई राशि से अधिक से धन की आपूर्ति कम हो जाएगी।

मनी मल्टीप्लायर आरबीआई (M0) के साथ ब्रॉड मनी (M3) एवं रिजर्व मनी का अनुपात है।

(इस प्रश्न को समझने के लिए मूल परिभाषाओं को स्पष्ट होना आवश्यक है)


91.भारत के सांस्कृतिक इतिहास के संदर्भ में, ‘परामिता’ शब्द का सही विवरण निम्नलिखित में से कौन-सा है ?

  1. सूत्र पद्धति में लिखे गए प्राचीनतम धर्मशास्त्र पाठ
  2. वेदों के प्राधिकार को अस्वीकार करने वाले दार्शनिक सम्प्रदाय
  3. परिपूर्णताएँ जिनकी प्राप्ति से बोधिसत्व पथ प्रशस् हुआ
  4. आरम्भिक मध्यकालीन दक्षिण भारत की शक्तिशाली व्यापारी श्रेणियाँ

उत्तर (c). ये वे परिपूर्णताएँ हैं जिनकी प्राप्ति बोधिसत्व पथ के लिए हुई थी।

महायान बौद्ध धर्म में, परामिता (महान वाहन), बोधिसत्वो द्वारा बोधित्व प्राप्त करने के लिए किसी भी परिपूर्णता के अभ्यास को कहा जाता है। छह परिपूर्णताएँ, उदारता (दान-परामिता), नैतिकता (सीला- परामिता ), दृढ़ता (शांति- परामिता), ताकत (वीर्या- परामिता), ध्यान, या एकाग्रता (ध्यान-परामिता), एवं ज्ञान (प्राण- परामिता) हैं। कुछ सूचिया परिपूर्णताओं की संख्या 10 तक मानती हैं, जिसमें दूसरों की मदद करने के कौशल (प्राणिधाना-परामिता), प्रबुद्धता उत्पन्न करने के लिए गहन संकल्प (प्राणिधाना-परामिता), 10 शक्तियों की पूर्णता (बाला-परामिता), एवं पारलौकिक ज्ञान अभ्यास (जनन-परामिता) शामिल है।

स्त्रोत - https://www.britannica.com/topic/paramita

(एक तथ्यात्मक प्रश्न)

92.भारतीय इतिहास के संदर्भ में, 1884 का रखमाबाई मुक़दमा किस पर केन्द्रित था ?

  1. महिलाओं का शिक्षा पाने का अधिकार
  2. सहमति की आयु
  3. दांपत्य अधिकारों का प्रत्यास्थापन

नीचे दिए गए कूट का प्रयोग कर सही उत्तर चुनिए :

  1. केवल 1 और 2
  2. केवल 2 और 3
  3. केवल 1 और 3
  4. 1, 2, और 3

उत्तर (b). रखमाबाई (1864-1955) औपनिवेशिक भारत में पहली महिला चिकित्सक थीं (हालाँकि आनंदीबाई जोशी पहली भारतीय महिला चिकित्सक थीं, लेकिन उनकी असामयिक मृत्यु के कारण वह अभ्यास नहीं कर सकीं)। रखमाबाई ने इतिहास में अपनी पहचान उस कानूनी मामले के कारण बनाई, जिसमें वह शामिल थीं, जिसने आयु अधिनियम, 1891 को अधिनियमित करने में योगदान दिया था।

पृष्ठभूमि - रखमाबाई, जयंतीबाई की बेटी थीं जिन्होंने एक डॉक्टर तथा प्रोफेसर डॉ. सखाराम अर्जुन से दोबारा शादी की थी। रखमाबाई की कम उम्र में शादी हो गई थी। विवाहित होने के बावजूद, रखमाबाई एवं उनके पति - दादाजी कभी एक साथ नहीं रहते थे। रखमाबाई ने अपनी शिक्षा जारी रखी। दादाजी बाद में उसके संपर्क में आए, शायद उनकी माँ के निधन के बाद उन्हें विरासत में मिले पैसों में दिलचस्पी थी। रखमाबाई ने जाने से इनकार कर दिया, अपनी शिक्षा को आगे बढ़ाने के लिए जारी रखा, जो उस समय के समाज के मानदंडों के खिलाफ था। दादाजी ने सखाराम को एक कानूनी नोटिस भेजा, मार्च 1884 में, उन्होंने रखमाबाई को अपने साथ रहने से रोकने का आरोप लगाया।

1885 में, शादी के 12 साल बाद, भीकाजी ने ‘संवैधानिक अधिकारों की बहाली’ की मांग की, जहां सुनवाई एवं निर्णय न्यायमूर्ति रॉबर्ट हिल पिनहेई ने किया था। रखमाबाई ने उस आदमी के साथ रहने से इंकार कर दिया था, जिससे उसकी शादी एक बच्चे के रूप में हुई थी, क्योंकि शादी में उसका कोई मर्जी शामिल नहीं थी। ब्रिटिश मिसाल इस मामले में निहित नहीं हो सकती है, क्योंकि ब्रिटिश कानून का अर्थ वयस्कों की सहमति के मामले में लागू किया जाना था। जस्टिस पिनेह ने ब्रिटिश कानून में इस सीमा को पाया एवं हिंदू कानून में इस प्रकृति के किसी मामले को नहीं पाया। अतः इस मामले में उन्होंने फैसला दिया कि रखमाबाई का विवा बचपन में कर दिया गया था तथा इसमें उनकी मर्जी शामिल नही था इसलिए उन्हें मजबूर नहीं किया जा सकता है।

स्त्रोत - https://feminisminindia.com/2017/08/22/rukhmabai-essay/

(एक तथ्यात्मक प्रश्न)

93.निम्नलिखित में से किस कारण से भारत में बीसवीं शताब्दी के आरम्भ में नील की खेती का ह्रास हुआ ?

  1. नील के उत्पादकों के अत्याचारी आचरण के प्रति काश्तकारों का विरोध।
  2. नई खेलों के कारण विश्व बाज़ार में इसका अलाभकर होना।
  3. नील की खेती का राष्ट्रीय नेताओं द्वारा विरोध किया जाना
  4. उत्पादकों के ऊपर सरकार का नियंत्रण

उत्तर (b). उन्नीसवीं शताब्दी के अंत में नील के व्यापार पर भारत के एकाधिकार को जर्मन रसायनज्ञ के कृत्रिम नील के आविष्कार द्वारा कड़ी चुनौती पेश की गई। उसके बाद भारत के नील उद्योग में तेजी से गिरावट आई है एवं निर्यात व्यापार इतने छोटे आयामों में सिकुड़ गया है कि इसका पूर्ण विलुप्त होना अपरिहार्य था।

(इतिहास से एक तथ्यात्मक प्रश्न)

94.वेलज़ली ने कलकत्ता में फ़ोर्ट विलियम कॉलेज की स्थापना किस लिए की थी ?

  1. उसे लंदन में स्थित बोर्ड ऑफ़ डायरेक्टर्स ने ऐसा करने के लिऐ कहा था।
  2. वह भारत में प्राच्य ज्ञान के प्रति अभिरूचि पुनः जाग्रत करना चाहता था।
  3. वह विलियम कैरी तथा उसके सहयोगियों को रोज़गार प्रदान करना चाहता था।
  4. वह ब्रिटिश नागरिकों को भारत में प्रशासन हेतु प्रशिक्षित करना चाहता था।

उत्तर (d). सिविल सेवा ईस्ट इंडिया कंपनी की रीढ़ थी। सिविल सेवकों को स्थानीय रूप से प्रशिक्षित करने का प्रारंभिक प्रयास वेलेजली द्वारा किया गया था। फोर्ट विलियम के परिसर के भीतर, उन्होंने 10 जुलाई 1800 को फोर्ट विलियम्स कॉलेज की स्थापना की। यह विचार ब्रिटिश शासन को ‘शासन’में बेहतर समन्वय के लिए पूर्वी संस्कृति, परंपरा, कानून एवं प्रशासन को समझाने के लिए था।

(विकल्प (b) से भ्रमित न हों)

95.भारत के इतिहास के संदर्भ में, "ऊलगुलान" अथवा महान उपद्रव निम्नलिखित में से किस घटना का विवरण था ?

  1. 1857 के विद्रोह का
  2. 1921 के मापिला विद्रोह का
  3. 1859-60 के नील विद्रोह का
  4. 1899-1990 के बिरसा मुंडा विद्रोह का

उत्तर (d). मुंडा विद्रोह भारतीय उपमहाद्वीप में उन्नीसवीं शताब्दी के प्रमुख विद्रोहियों में से एक है। बिरसा मुंडा ने 1899-1900 में रांची के दक्षिण में इस आंदोलन का नेतृत्व किया। उलगुलान, जिसका अर्थ है ‘द ग्रेट टुमल्ट’, ने मुंडा राज एवं स्वतंत्रता स्थापित करने की मांग की। मुंडाओं ने पारंपरिक रूप से खुनकट्टिदार या मूल रूप से जंगल को साफ होने के रूप में एक तरजीही किराये की दर का आनंद लिया। लेकिन उन्नीसवीं शताब्दी के दौरान उनकी इस खूंटकट्टी भूमि व्यवस्था को व्यापारियों एवं धनपतियों के रूप में आने वाले जागीरदारों एवं ठेकेदारों द्वारा मिटा दिया गया।

(एक सीधा, तथ्यात्मक प्रश्न)

96.प्राचीन भारत के विद्वानो/साहित्यकारों के संदर्भ में, निम्नलिखित कथनों पर विचार कीजिए :

  1. पाणिनि पुष्यमित्र शुंग से संबंधित है।
  2. अमरसिंह हर्षवर्धन से संबंधित है।
  3. कालिदास चन्द्रगुप्त-II से संबंधित है।

उपर्युक्त कथनों में से कौन-सा/से सही है/हैं?

  1. केवल 1 और 2
  2. केवल 2 और 3
  3. केवल 3
  4. 1, 2 और 3

उत्तर (c). चन्द्रगुप्त द्वितीय एक परोपकारी राजा था, जिसके अधीन भारत को शांति एवं सापेक्ष समृद्धि प्राप्त थी, उसने शिक्षा को भी संरक्षण दिया, उनके दरबार के विद्वानों में खगोलशास्त्री वराहमिहिर एवं संस्कृत कवि एवं नाटककार कालीदास थे। पाणिनि के अनुसार, उनका अस्तित्व लगभग 5 वी से 6 ठी शताब्दी ईसापूर्व तक अनुमानित है, जो कि पुष्यमित्र शुंग से 300 से 400 साल पहले की घटना है। पुष्यमित्र मूल रूप से मौर्य साम्राज्य का सेनापति था। 185 ईसा पूर्व में उन्होंने एक सेना की समीक्षा के दौरान अंतिम मौर्य सम्राट, बृहद्रथ मौर्य की हत्या कर दी एवं सम्राट बन गए। संस्कृत व्याकरण संबंधी अमरसिंम्हा का संबंध विक्रमादित्य से था, न कि हर्षवर्धन से।

स्त्रोत - https://www.britannica.com/biography/Chandragupta-II

(एक सीधा, तथ्यात्मक प्रश्न)

97.मानव प्रजनन तकनीकी में अभिनव प्रगति के संदर्भ में, "प्राक्केन्द्रित स्थानान्तरण" (Pronuclear Transfer) का प्रयोग किस लिए होता है ?

  1. इन विट्रो अंड के निषेचन के लिए दाता शुक्राणु का उपयोग
  2. शुक्राणु उत्पन्न करने वाली कोशिकाओं का अनुवंशिक रूपान्तरण
  3. स्टेम (Stem) कोशिकाओं का कार्यात्मक भ्रूणों में विकास
  4. संतान में सूत्रकणिका वाले रोगों का निरोध

उत्तर (d). ‘प्राकेन्द्रिक स्थानांतरण’ की तकनीक में एक युग्मज से दूसरे युग्मक में नाभिक का स्थानांतरण शामिल होता है, एवं इच्छुक पुरुष माता-पिता के शुक्राणु के साथ स्वस्थ दान किए गए अंडे/अंडों (माइटोकॉन्ड्रियल दाता द्वारा प्रदान) के निषेचन की आवश्यकता होती है। जब एक युग्मज की कल्पना की जाती है अतः माइटोकॉन्ड्रिया मां से आता है। यदि मां के माइटोकॉन्ड्रियल डीएनए में कुछ गड़बड़िया होती है तो गड़बड़ियों को इस माइटोकॉन्ड्रिया के माध्यम से बच्चे में स्थानांतरित हो जाती है। इन विट्रो निषेचन में इसे रोकने के लिए, माइटोकॉन्ड्रिया को तीसरे पक्ष से लिया जाता है। ऐसे शिशुओं को ‘तीन अभिभावक बच्चे’ कहा जाता है।

(तथ्यात्मक लेकिन कठिन प्रश्न)

98.विकास की वर्तमान स्थिति में, कृत्रिम बुद्धिमत्ता (Artificial Intelligence), निम्नलिखित में से किस कार्य को प्रभावी रूप से कर सकती है ?

  1. औद्योगिक इकाइयों में विद्युत् की खपत कम करना।
  2. सार्थक लघु कहानियों और गीतों की रचना
  3. रोगों का निदान
  4. टेक्स्ट से स्पीच (Text-toSpeech) में परिवर्तन
  5. विद्युत ऊर्जा का बेतार संचरण

नीचे दिए गए कूट का प्रयोग कर सही उत्तर चुनिए :

  1. केवल 1, 2, 3 और 5
  2. केवल 1, 3, और 4
  3. केवल 2, 4 और 5
  4. 1, 2, 3, और 5

उत्तर (d). सभी कथन सही हैं। इमारतों एवं कारखानों में, AI संख्या, पाठ, चित्र एवं वीडियो के रूप में ऊर्जा की खपत के बारे में जानकारी की निगरानी एवं एकत्र कर सकता है। देखा गया है कि ।प् ऊर्जा उपयोग को प्रबंधित कर सकता है, इसे पीक आवर्स के दौरान कम कर सकता है। बाधाओं की पहचान की जा सकती है एवं उपकरण विफलताओं का पता लगाया जाता है एवं ऐसा होने से पहले ही उसे ठीक किया जा सकता है। AI में भविष्य की समस्याओं की भविष्यवाणी करने एवं अंततः लंबी अवधि में ऊर्जा की खपत को अनुकूलित करने के लिए बड़े पैमाने पर डेटा को संपीड़ित एवं विश्लेषण करने की क्षमता है।

AI पहले से ही हमारे लेखन कौशल को बेहतर बनाने में मदद कर सकता है। उदाहरण के लिए, हेमिंग्वे ऐप 2013 में बनाया गया एक ऑनलाइन लेखन संपादक है। यह सामान्य लेखन समस्याओं को पहचानने एवं पठनीयता बढ़ाने के लिए प्राकृतिक भाषा प्रसंस्करण (एनएलपी) का उपयोग करता है। और, हां, यह पैराग्राफ को फिर से भी लिखता है। आप भी टाइपिंग के बिना लिखने के लिए NLP - संचालित ड्रैगन डिक्टेशन या ऑनलाइन Dictation-io का उपयोग कर सकते हैं।

सही ढंग से रोगों का निदान करने में चिकित्सा प्रशिक्षण के वर्षों लगते हैं। मशीन लर्निंग ने हाल ही में स्वचालित रूप से रोगों के निदान में भारी प्रगति की है। यह उन क्षेत्रों में विशेष रूप से सहायक है जहां नैदानिक जानकारी एक डॉक्टर की जांच पहले से ही डिजीटल है। जैसे, सीटी स्कैन के आधार पर फेफड़ों के कैंसर या स्ट्रोक का पता लगाना, इलेक्ट्रोकार्डियोग्राम एवं कार्डियक एमआरआई छवियों के आधार पर अचानक हृदय की मृत्यु या अन्य हृदय रोगों के जोखिम का आकलन करना, त्वचा की छवियों में त्वचा के घावों को वर्गीकृत करना एवं आंखों की छवियों में डायबिटिक रेटिनोपैथी के संकेतक खोजना।

एक टीटीएस उपकरण लिखित पाठ को प्राकृतिक भाषण में परिवर्तित करता है जिसे किसी भी मानव उपयोगकर्ता द्वारा सुना एवं समझा जा सकता है। टेक्स्ट-टू-स्पीच टूल का उदाहरण गूगल क्लाउड टेक्स्ट-टू-स्पीच टूल, अमेजन वेब सर्विसेज एवं अमेजन पोली हैं। इस तकनीक को विभिन्न अंतर्निहित तकनीकों द्वारा सक्षम किया गया है जिसमें मशीन लर्निंग एवं आर्टिफिशियल इंटेलिजेंस जैसे गहन शिक्षण उपकरण शामिल हैं।

वायरलेस पावर ट्रांसफर में मांगों का विस्तार हो रहा है एवं विशेष रूप से उद्योग 4.0 अनुप्रयोगों के साथ अधिक आकर्षक बन रहा है। सर्किटरी के साथ पावर प्रबंधन कई स्त्रोमां जैसे कि पीजो तत्वों या भविष्य के शोधों के लिए इंजन कंपन से ऊर्जा संचयन की अनुमति देता है। नतीजतन, कृत्रिम बुद्धिमान प्रणालियों के लिए बिजली की आपूर्ति करने के लिए सभी युग्मन विधियों का व्यापक रूप से उपयोग किया जाएगा।

(यदि आपको 1 एवं 2 सही मिलते हैं, तो आप विकल्प (b) एवं (c) को हटा सकते है। चूंकि 4 सही है, इसलिए उत्तर (d) होना चाहिए।)

99.दृश्य प्रकाश संचार (VLC) तकनीकी के संदर्भ में, निम्नलिखित में से कौन-से कथन सही है ?

  1. VLC, 375 से 780nm वाली विद्युत-चुम्बकीय स्पेक्ट्रमी तरंगदैर्घ्यों का उपयोग करती है।
  2. VLC को दीर्घ-परासी प्रकाशी बेतार संचार के रूप में जाना जाता है।
  3. VLC ब्ल्यूटूथ की तुलना में डेटा की विशाल मात्रा को अधिक तेजी से प्रेषित कर सकता है।
  4. टस्ब् में विद्युत्-चुम्बकीय व्यतिकरण नहीं होता है।

नीचे दिए गए कूट का प्रयोग कर सही उत्तर चुनिए :

  1. केवल 1, 2 और 3
  2. केवल 1, 2 और 4
  3. केवल 1, 3 और 4
  4. केवल 2, 3 और 4

उत्तर (c). कथन 1, 3 एवं 4 सही हैं, लेकिन 2 नहीं है। 2011 में, हेराल्ड हास ने लाइट फिडेलिटी (लाई-फाई) शब्द का प्रचलन शुरू किया। लाई-फाई एक उच्च गति द्वि-दिशात्मक है जो पूरी तरह से जुड़ा हुआ है, दृश्यमान वायरलेस वायरलेस संचार प्रणाली है एवं वाई-फाई के अनुरूप है, जो संचार के लिए रेडियो आवृत्ति का उपयोग करता है। वाई-फाई सिग्नल में अन्य आरएफ संकेतों के साथ हस्तक्षेप की समस्या है जैसे कि विमान में पायलट नेविगेशनल उपकरण सिग्नल के साथ इसका हस्तक्षेप। इसलिए, उन क्षेत्रों में जो विद्युत चुम्बकीय विकिरण (जैसे विमान) के प्रति संवेदनशील हैं, लाई-फाई एक बेहतर समाधान हो सकता है।

कथन 1, 3 एवं 4 सही हैं, लेकिन 2 नहीं है। 2011 में, हेराल्ड हास ने लाइट फिडेलिटी (लाई-फाई) शब्द का प्रचलन शुरू किया। लाई-फाई एक उच्च गति द्वि-दिशात्मक है जो पूरी तरह से जुड़ा हुआ है, दृश्यमान वायरलेस वायरलेस संचार प्रणाली है एवं वाई-फाई के अनुरूप है, जो संचार के लिए रेडियो आवृत्ति का उपयोग करता है। वाई-फाई सिग्नल में अन्य आरएफ संकेतों के साथ हस्तक्षेप की समस्या है जैसे कि विमान में पायलट नेविगेशनल उपकरण सिग्नल के साथ इसका हस्तक्षेप। इसलिए, उन क्षेत्रों में जो विद्युत चुम्बकीय विकिरण (जैसे विमान) के प्रति संवेदनशील हैं, लाई-फाई एक बेहतर समाधान हो सकता है।

दृश्य प्रकाश संचार (VLC) एक डेटा संचार संस्करण है जो 400 एवं 800 THz (780-375 एनटी) के बीच दृश्य प्रकाश का उपयोग करता है। वीएलसी ऑप्टिकल वायरलेस संचार प्रौद्योगिकियों का एक सबसेट है।

डेटा के प्रसारण में दृश्य प्रकाश का उपयोग प्रौद्योगिकियों पर कई अलग-अलग महत्वपूर्ण फायदे रखता है जो रेडियो आवृत्ति का उपयोग करते हैं। इसका सबसे बड़ा लाभ पूरे दिखाई देने वाले प्रकाश स्पेक्ट्रम का आकार है, जो पूरे रेडियो स्पेक्ट्रम की तुलना में 10,000 गुना बड़ा है, जिसके अति प्रयोग के कारण भी भीड़भाड़ है। एवं वीएलसी सुनिश्चित करने के लिए ब्लूटूथ की तुलना में बहुत तेजी से संचारित कर सकता है (इसलिए 3 सही है)।

ओडब्ल्यूसी सिस्टम जो दृश्य बैंड (390-750 एनएम) में काम कर रहे हैं, उन्हें आमतौर पर दृश्य प्रकाश संचार (वीएलसी) कहा जाता है।

ऑप्टिकल वायरलेस कम्युनिकेशन (OWC) – OWC एक सामान्य शब्द है जिसका उपयोग केबलों (उदाहरण के लिए, फाइबर ऑप्टिक केबल) के बिना सभी प्रकार के ऑप्टिकल संचार को संदर्भित करने के लिए किया जाता है। इसका अर्थ है कि दृश्य प्रकाश संचार, लाई-फाई, मुक्त स्थान ऑप्टिकल संचार एवं अवरक्त रिमोट कंट्रोल सभी OWC के अंतर्गत आते हैं।

स्त्रोत - https://www.sciencedirect.com/science/article/pii/S2352864816300335

(कथन 1 निश्चित रूप से सही है, इसलिए विकल्प (d) समाप्त हो जाता है। कथन 3 सही है, इसलिए (b) भी निरस्त। कथन 4 सही है)

100."ब्लॉकचेन तकनीकी" के संदर्भ में, निम्नलिखित कथनों पर विचार कीजिए :

  1. यह एक सार्वजनिक खाता है जिसका हर कोई निरीक्षण कर सकता है, परन्तु जिसे कोई भी एक उपभोक्ता नियंत्रित नहीं करता।
  2. ब्लॉकचेन की संरचना और अभिकल्प ऐसा है कि इसका समूचा डेटा केवल क्रिप्टोकरेंसी के विषय में है।
  3. ब्लॉकचेन के आधारभूत वैशिष्ट्यों पर आधारित अनुप्रयोगों को बिना किसी व्यक्ति की अनुमति के विकसित किया जा सकता है।

उपर्युक्त में से कौन-सा/से कथन सही है/हैं? उपर्युक्त कथनों में से कौन-सा/ से सही है/है ?p>

  1. केवल 1
  2. केवल 1 और 2
  3. केवल 2
  4. केवल 1 और 3

उत्तर (d). ‘ब्लॉकचेन’एक विकेंद्रीकृत बहीखाता है जो विश्वास पर काम नहीं करता है, किंतु एक गणितीय संरचना है जो पारदर्शी रूप से सभी प्रतिभागियों को दिखाई देती है। ब्लॉकचेन तकनीक में लगभग हर उद्योग में लगभग अंतहीन मात्रा में अनुप्रयोग हैं। तकनीक को वित्त में धोखाधड़ी को ट्रैक करने के लिए लागू किया जा सकता है, स्वास्थ्य पेशेवरों के बीच रोगी चिकित्सा रिकॉर्ड को सुरक्षित रूप से साझा करने एवं यहां तक कि कलाकारों के लिए व्यापार एवं संगीत अधिकारों में बौद्धिक संपदा को ट्रैक करने के लिए भी तकनीक उपयोगी है। इसलिए कथन 1 सही है एवं 2 नहीं है। उत्तर ;ंद्ध या ;कद्ध हो सकता हैं। अनुमोदन या विश्वास के बिना एप्लिकेशन को नेटवर्क में जोड़ा जा सकता है। यह क्रिप्टोकरेंसी एवं अंततः ब्लॉकचेन में एक महत्वपूर्ण बहस है। अतः कथन 3 सही है।

(पहले कथन 2 को गलत के रूप में पहचानें, क्योंकि ब्लॉकचेन अनुप्रयोगों की एक बड़ी श्रृंखला के लिए हो सकता है। अतः , (a) एवं (d) हमारे संभावित उत्तर हैं। चूंकि 3 सही है, उत्तर है (d)








Questions and Detailed Solutions are being continuously updated ... refresh and check. Comment and let us know your experience, answers and solutions too!